You are on page 1of 344

MA THEMA TICAL

AND

ASTRONOMICAL TABLES,

POR THB USB OJ'

STUDENTS OF MATHEMATICS,

PRACTICAL ASTRONOMERS, SURVEYORS, ENGINEERS, AND NAVIGATORS;

WITH

AN INTRODUCTION,

CONTAIJI'ING

. THE EXPLANATI~N AND USE OF THE· TABLE~,

ILLU8TUTED BY'

NUMEROUS· PItOBLBlIIS AMD EXAAIPLES.

BY WILLIAM GALBRAITH, M.A.,

TBACHBR OJ' MATHBMATICS IN BDINBURGH.

--

..

• __ J -

... • 0. W ~ : .... ... __

EDINBURGH:

PUBLI8RIID BY

OLIVER 4; BOYD, TWEEDDALE-COURT;



GEO. B. WHITTAKER, AND J. W. NOBlE & CO., LONDON.

18!7.

DEL

Dig;tized by Google

I _

, ,

. - ...

r· · .. l. ,

-r-'~~~ ';,' -s- '. , .

.~ ...

'.

., .'

. '.

'. ,

, '. e ,

OLIVEIL. BOYD, PILINTE.S,

Digitized by Google

TO

SIR GEORGE CLERK, OF PENNYCUICK,

BART., M.P., F.R.S.,

ONE OF THE LORDS COMMISSIONERS OF THE ADMIRALTY,

&c:.,&c:.,&c:.

THE following Work, which you have allowed me the honour of inscribing to you, is intended to promote the purposes of useful instruction, and the advancement of practical science; and it is therefore confined to subjects having a direct utility in the business of life.

THOUGH I am aware that no patronage can materially influence the success of a Work of this nature, which must depend upon its merits alone; yet I have been eelieitous to inscribe it to you, in the hope, that practical men, in search of useful knowledge, may be induced to consult a Book sanctioned by a name intimately connected with many recent scientific improvements; and I confidently trust, that a reference to the volume itself will prove that your obliging permission has not been undeservedly bestowed.

I have the honour to be,

SIR,

With the utmost respect,

, Your most obedient servant,

WILLIAM GALBRAITH.

EDIN1IURGH, NOfJ., 1826.

Digitized by Google

Digitized by Google

PREFACE.

THE appication of the mathematical sciences to practical pur. poses·has of late made great advances in accuracy aud preciaion. The perfection also which astronomieal and geodetical operations have reached, and the extreme delieaey of construction to which instruments have been carried, require correspondent improvementa in the methods of computation and reduction; and, there. fore, convenient tables of moderate expense must be of great value to thoee engaged either in the detalls of practice, or the business of instruction.

There are two classes of tables chiefly in use; . one 'either large and expensive, or attached to expensive works, and which therefore can' with dilHculty· be procured by the generality of purchasen; the other 10 limited and defective as to be totally unfit for· constant reference. It has been my study to hold a middle course between these two extremes. By making such additions to the usual tables as to render their application more easy, without greatly increasing their bulk; by selecting the most useful from larger eolleetions ; by supplying some new tables, and simplifying the practical rules, several very' laborious processes have been rendered more simple and precise, while the requisite accuracy for the nicest purposes has been strictly preserved.

In most of our initiatory works for popular instruction, the processes and examples are unfortunately conducted in such a manner as to be comparatively of little advantage in actual prae. tice, and, consequently, what has been learned in youth, must, in a great degree, be forgotten in manhood, while new methods are then to be acquired.

Digitized by Google

vi

PREFACE.

To remedy this inconvenience, I have selected some of the most approved modes of treating the problems frequently required by Astronomers, Na ... igators, and Engineers, from the works of persons celebrated for their successful application of the exact sciences to the niceties of modem practice.

I have therefore taken many of the Astronomical Rules and Examples from the works of Maskelyne, Pond, and Brinkley; and such as relate to other tbpics from those of Captains Kater, Hall, Sabine, and Parry. To Captain Hall I am under great obligations, not only for access to his original papers, but also for his friendly advice reietm te the application of these methods to practice.

To Mr Ivory I am indebted for his very accurate Table of Astronomiaal Befraeca..,. wbich 1: have endeaeoured tt) improve by eatpanding'acl'adding proportianal parts to "he subsidiary tahles, thereby facilitating itw practical applicatloD. .

. ' Betides labouring to improVe many of the' onli.ary Tahles, I lJave·added. several which anvn8w, _ieS" for the purpose of eimplifying some operatiODB'and reodering others.ore acCu¥ate.

,The eX'plaaatimt wali it is boped1 be found full and eq»Jioit, ~ally towards ~ beginning. The explanation of INIbI8 tab'M1l which follow others, analogous in structW'e or llI'gumentw, is sometimes less full, as it is presumed thOle pt"oviou.ly given are *ell u~. For examplt, the note to Table X·XV., at the .bottom. ,of page 91-, ean hardly be- inQ!lligille to • mere praetiall mlm who has.little _thellNltical know1.dge; but til the method of takiag out the quantities'f'rom Table V., in .batever-quadnmt !Of the cirole, or diviaicm of 24 bonrs, they al'lnituNd,· i8 so t\aUy ftplained lMfore,·it was dtougbt unn8Oel8ary'to repeat the same minutiaie a second time. Still,. bowewr, there may be some r-a 'Which require to be expanded, Hl order to he ~ rada.y 1Mdent.ood, &8 well as others which might, petMps with propriety, be abridged.

The Imroduetion is divided ilttD three puts,' followed' by a eopieus explanation of the general tables, which may M called • fourth;

" In the first I have shortly described the nature, and 'in"'8hgated the more simple series for the computation of Logarithms. I have generally, however, only given the more important mles in words at length, without investigation, so as to Q.e.Creadi1y com-

Digitized by oogle

~ lty penons who haY8 aequed a knowledge of the elemeBtATy priaciples of matheaatics. In fact, the demonait ... boas CBIl:_y beuDdentood by those.who _e.btaiDed a toler. able knowledge of the elements. of geometry and,.~ &ad, since the generality of bookseontainillg thelle eomprehM alto the usual investigations in trigonometry, it was thought advisable to omit tilem. If,.. example, a student Ihould ptrclmee Lepdre's Elementa of Geometry in order to iltudy that acience, lit wilI-noo it tooontam,allo. very elegant iDVestigatiOD8 of almo&t all the useful propenies in Plane and Spherical Trigoaometl'y. On this account, I have only given the demonatratioos of thoee propositionaless commonly inserted in the usual treatiaea.

On the Barometric Measurement of Altitudes, I have given four different methods. The third is in a great degree new, and by the original subsidiary uWee, oalcWated expreuly, for this purpose, it will be found easy and accurate.

TJie second part colltaiae Spherieal Trigooometry, with a great variety of its IOO8t useful application.. As the rules and examples are either new or selected from the best writers on the subject, it .is hoped this section will prove intel'eatiag to students of Aatronomy and Navigation, siace it eentains a number of the usual methods and examples practised by the most distinguished men of seieaee of t'he day.

The ttUrd part contaiDs a variety of Rules and Formu1re for the ueeof Survey.ora, Enginee~ Navigw&tors, and practieal Astronomers. Those for geodetical purposes are selected chiefly for their general utility, andeomprehead a sutlicieDt number for usual pmctice,-an idea which was suggested to me by some Of my more advaaced pupils who have been employed in goyernment surveys. They were first collected in the form of notes aud tl'aDIcribed into tbeir albulll8, to be used when they were engaged in geodetical, accurate military or marine surveying; and as they may prove generally useful to that class of Students, I have arranged them in as natural an order as possible.

The ingenuity and skill of Captain Kater having devised the most beautiful simplifications of the problem of determining the 'figure of the earth by means of the pendulum, and brought the experiment within the reach of our more active and intelligent military and naval officers, I have added the necessary rules and formula for that purpose, in order to initiate, as far asrposaibl1e,

Digitized by uoog e

viii

PREFACE.

our Cadets and Midabipmen in these iatereltiug nsearcbee; as such higher objects of pursuit, not only invigorate their faculties, but inspire them with enthusiasm for the attainDJell~ of prof. Nonal renown.

The fourth part contains the necelS8l'Y Explaoation. of the Tables.

I have thus endeavoured to collect, into as small a space as' possible, the greatest quantity of useful matter naturally ~ nected with the subjects treated in the work; but with what s':!<:eeM I must allow the, p1bli? to detenline.

WILLIAM GALBRAITH.

I,' I

EnUrBVaGH, Nooembef', 1826.

• • ..t ",,' .,

: " .

0' " : ...

• ', .I'l

, .' •••• ,_, • .{. ' ... 1.

, {

Digitized by Google

CONTENTS

or

I'NT'R 0 Due T I'ON.

. I'ajpI.

P".or·l .. PBOPZRTlBIt at LOGARITHMS I

. CoIIItra.et:ictJr of ~thm •••••••••• f •• iii ••••••• ~ ••••••• 'tl ••••••••••••••• 4

. TrigvDometrierJ Lines, eaIled Sinetl. &e........ 8

-lIf1lIttp1es and Powen of AlI:I.................. It

PLAlrIB TB.IGONOIIE7R.Y •••••••• _"., ,..................... 16

lu AWN'; .10 s.iliIItII. ia.,N~n............................. t4

lea ApplicaDon to the l\(en.)U&tion of H~ IIIId. DiltqceI....... t6

_Applil ei to .. D 2 .. ·IIti_ot ..... ·ancl alRe.

~ i'~and a TaWeolllteir .Df.eMIIl'a _ U, 44

a-.etdc Ml&IU.I'eIDMt of Altitudea •••• '"1! ... _....................... '"

n. sPHERICAL TRIGONOMETltY. &c.

Deftnitiona, Principles, and General PToperties....................... 116 Solution ot Spherical Triangles, with their Stereograpbic PTojection 6S

Napier', Rule of the CillllUlu l"aft..................................... 64

Mublyne's Rules for determining the Latitude and Longitude, from the Right Ascension, Declination, and the Obliquity of

the Ecliptic, &.c.. 68

8olutioa of Oblique-Angled Spherical Trianglel. 73

• Oa finding the Latitude by Observation................................ 81 On Finding the Longitude by ObllernUon.

I. By LuJllll'll 81

t. ~ ~ _. __ ••. _ 104

Equation to Equal Altitudes.......... 107

3. By Occultations 114

4. By th. M:o()n~1 TIlLlmh _ ,119

or tile Tmn.!it Instrument _ _ 'lSI

.. To tD.ke a -T{1!.DSlt ~ lSI

l~eU.oil,of T!lblliatillg II Ttlllltlit .,. 1St

. To bring II Tmn&it Instrument into the Meridian 133

To deterllline t!:t e EnOT IU!d Rate of a Clock or CtllODOlDeter by the

.. _ 'rr .. ~rit Instramenr _" " - .. : 136

nr, JII:N8UllATION, SURVEYING, AND F01UIUL.f:. &:c.

lIIe111liration of SUrfaces... 139

lfeDIUl'IItion of SOlid............. 142

Digitized by Google

x

CONTENTS.

.....

PAa" IlL .I.and SuryeyiDg ••••••••••••••••••••••••• •••••••••••••••••••••••••••••••••• 1"

LeveDing ••••••••••••••••••••••••••••••• '" .:............................... 146

RULES ~d FORMULlE ••••••• _..................................... 147

The belt Form of TriaDgIeI................................. 147

To reduce Aaglea to the Cellae of the Statioa 147

To compute the Spherical Exceu 146

To reduce a Measured BIlle lit any Height to the Level of the Sea. 141 To determble the Horizontal Refraction by Rule or Ponnube ••• ••• 149 To find the Angle made by a given Liae with the MerWan •••••••• 149 To deterUlble the Ellipticity of the Earth by the Menreaumt of

Arca ••••••••••••••••••••••••••••••••••••••••••••••••••••••••••••••••••••• 160

To determine a Degree of Latitude............ •••••• ••• ••• ••• ••• ...... 152 To deeermille a Degree of Loagitude........ •••••••• ••••••. •••••• •••••• 161

To determine an ObUquo Dr,gree... ~ ,. ••• ••• ••• 163

Speci6e (harity : 163

To detemUDe the Specific Gravity of Air, Dry, 1I&t~ with Moisture, and according .. the IICtUl State of the AlIQoIphere ••• 164

To detenniIle die Speciic (hayitiell in _0........................... 166

T. ~, ~ ... 01 ~e 1\1l4IJ&qey.r the A.tU1oaphere on

the Pendulum •••••• •••••• .•• ••• ••• ••• ••• ••• ••• ••• ••• ••• ••• ••• ••• ...... 166

CornetioII of PendulUIDI ribrating in Circular .Area... ••••••• 166

. . CcI~q"'.qf.VibratiOD Cpr :QlWylolJq , ", .. , 166

.... '''t.t',.''." ~ ......••. , .. for Ex}HPlajoD- " •.• " •• "" , ••••••••• 1&7

....•. " " for Height abtlve the SII& , •••• sr ••••••••••• 167

DetezmiA..aon of me ~gth o.f the Pend~'l1D at qijf~t Points

on the Earth', Surfac:e , ••••••••.••••••• , •• 168

Determiutionehhe FiguJe'of the Eanla by .the Peac1alam 169

~of the EJIIPWt ancI. Frencla l'atbd_ .. ,. .:.......... 180

Velocity ofSound I60, 161

Velocity of the Dieebuweot WMeI-pi,... ~ ~ .. 161, 162

FaR in a Rher eauMd by ObttIuctioa U tU SUeanl.... 162

. - Toa..-pol ihi,."..., ...._ •• I " " 182

Strength ofTim1!et.~ ,., , , I66, 167, 166

.' ,

CONTENTS

011

EXPLANATION OF THE TAB.L;t:8. &c.·

=-~

r. )files of Longitude at any Latitude.. 1 1

II. Logarithlllll of Numbers ~... 1 1

Logarithmic Arithmetic ••••••••••••••••••••. '" ••••••• ••• 6

III. Abglts which every Point &ad -Qaarter Pbibe of the

CompaBs makes ",ith the Meridian .••••• ...... ••• ••• 7 17

IV. Logarithmic Sinel, .tc. to every Point 'ana Quarter

Point ofthe' c.npa..~ : ',.......... 7 17

V. I..ogarithmic Sinee, Tangents, &c. te Degrees......... 7 18

VI. Natural Siues, 'I'allgente, Secants,._ VmineB, to

. every Degree of the Quadrant .

VII. Meridional 'Parts to every Degree of the Quadrant ••••

. VIII. Traverse Table ' .

11 63 11 64 11 64

Digitized by Google

~_·I

... ' .. ~~

.JX. Di~~ Logarithms......................... 12 86

X. ProportiolUll LogarithlDll :............... 13 68

XI. Dip of the Horizoa............... 13 84

XII. Dip M diJl'enDt DuIallCel................. 13 84

XIII. Cometlon of the Sun's Altitude M Sea 14 84

XIV. CIII1'.KCion ofa SW'. Altitvcle........................... 14 84 XV. SuP'. SsnitUameter, &0................................. 14 86 XVI. SuR', Parallu In Altitude............................... 14 86 XVII. Mean Refractions by Mr Ivory.......................... 14 88

XVIII.}

X~ Subsidiary to XVII..................................... 14 89

XXI. Aup1fttaUon of the 1\1000'. 8emidiamtter in Alti.

tude, tud Z.D........................... 18 90

XXII. Reduction of the Moen's Parallax on the 8phet'Oid... 16 90 nllI -. Logarithmll 01 the Earth's Radii en the Spheroicl...... 16 91

XXIV. Recinetion of die Latitude ;.... 18 91

XXV. For determining the Latitude by the Pole SUr........ 17 91 XXVL Augmeotaden of the Moon '_ Semidiamecer "1 the

Nonageaimal............................................. 18 92

'Xln'II. BcJ.uation of Second DilFerences fw 111 MId '4 bonra." 19 93

XXVIII. Reduction to the Meridian... 21 94

'XXIX. Reduction ., either Selstice.............................. 24 96

XXX. To change Mean Solar into Sidereal 'rime........ 28 98

xxxr, To change 8icleteal into Mean flolar Time..... 28 98

XXXII. To con'fert Meen 'rime into Parts of the Eftuator 29 97

XXXIII. Lengths 01 Circular Area... 29 97

~IV. 'To XlNIII. Fnr eomputing the c-tieD. 01 the

Filled Stars .

. ·!l[Ux" Mean Obliquity of the Ecllptic ..

L., Aad LI. Coneetieu of the ObUllu!ty .

1.11. And LI~I. ~lar ancl Luaar Nlltations 01 the Equi.

DOllell lD TlIIle •••••••••.••••••.••••.••••••••••••••••••••••

LIV. Right Ascensions and Declinations of Stars for 1828. ..

LV. Decimal Numbers for each Day in the Yeer .

tVI. Sun's R.A. for 1828 ..

LVII. Sun'a Declination for 1828 .

LVIII. Equation of Time for 1828 ..

Ux. Comction of Longitude by Chronometera .

LX. Latitudee and Lon8i\Udea of Placea ..

LXI. To con'fert Space into Time .

LXII. To con'fert Time into Space .

LXIII. Useful Numbers in Calculation ..

LXIV. And LXV. To lind the Time and Height of High

Water ..

LXVI. And L.~VII. Tablee of Equation of Tbird and

Fourth DilFerencee ..

LXVIII. Table to fIld the Latitude by tbe Pole Star .

7' .
"
~l
...... !
'::
-
o·'.j
:.' ..
t,t,j
• w.
«...~.
, .. , I I';.i 'q

IIISCELLANBOUS TABLES IN THE INTR()])(]CTION.

29 98 32108 32103

32103 82104 32104 33106 8S 108 33 107 34107 86108 36109 36109 38110

38 111

36112 87112

T AJlLZ . ,I. Sigal of Tftsaeometriall LineI.... 12

.. ' II. Multiples and Po" .. of Arca 12, 13, 14, 18

H , Ill. Heaaaree of Forti "...... 44

IV. Depaeuioo of Memuy in Glue Tubes... 48

I:.' V. Elastic Force of Aqueous Vapour (Dalton) 48

.• ' VI. Loprithma 01 the Bulk of G .. at different Tempera-

7

tura ••••••••••••••••••••••••••••••••••••••••• ".............. 49

Digitized by Google

TABu: VII. Logarithm. of the E!i!c:t of Latitude GO Batometrie

AItitudea ••••••••••••.••••••••••• , ••••• .., Ie

VIII. Comct:ionof the Oblique Semidismeter in LuIlU'll by

Dr Young lot, un

IX. Equation of Seroud Dlft'erence for Three ROlIn or for

60' and 100" .

X. Correction of Apparent Time dependil1lJ upon the Equa. tion of Second DifFerence and the variation of the

Diatance between the Moon and the Sun, ,.,r a fixed

Star, in Three HOuTII ..

Xl. Of the Decimal Fractions ora Day .

Xli. Decimal ·Parts of an Honr ..

XIII. To convert Decimals of Time into Degreea at the rate

of fifteen Degrees to an Hour .

XIV. Variation of tbe81Jn's R.A. and D. in one Second for

each Month in 'the Year ~.

XV. Areas of Circular Segments .

XVf Polygons ..

XV n. Regular Bodies .

:leVIn. Table A. for Correcting the Number of OadUationa

for the Arc of Vibration.... 156

XIX. Tables of Specific Gravity : 183, HU, 166

XX. Expa01lions of Solids and Liquids...... 166

XXI. Table for computing the Strength of Timber 186

XXII. Table for Correcting Lunan for SphemidalFiJUft of

the Earth. Explanation of TabI8l .

·XXUI. Table for ftnding the Latitude by the Pole Star ..

Digitized by Google

In

102 113 US

113

138 141 142 143

, .,

... t •.

INTRODUCTION .

. r. ;

r 1

I

PART I.

OF LOGARITHMIC AND TRIGONOMETBICAL TABLES.

-. SECTION I.

: .. ' OJ the Propertiel qf Logaritlnru.

1. ;:LoaAJlITRHS are a series of numbers, originally invented by Baron Napier, for ·tbe. purpoae of facilitating arithmetical calculations. T_ end is attained by their enabling us to perform the operations of·mulliplication by addition, of division by subtraction, ef involutiOh'by maltiplieation,and oftbe extraction of roots by division.-

~: It is evident that any two series of numbers, the one being in ~~m~cid arid the other in geometrical progression, possess these ~es; 'thus, for example, let the

.A:r. series be U 1 2 3 4 5}

Geo. series I 10 100 1000 10,000 100,000 &C.

Now, if we add any two numbers in the arithmetical series, such

as 2 and 3, which are equal to 5, and multiply the corresponding numbers under them, 100 and 1000, we have 100,000, the number immediately under 5, which was obtained by the addition of 2 to 3. Hence, then, it is clear that, if tables of this kind, sufficiently extensive, were formed, by a reference to them, the operation of multiplication could be performed by means of addition.

In like manner, we perform division by subtraction, for, if from 5 we take 3, the remainder is 2, under which we get 100, that is 100,000, the number under 5, divided by 1000, that under 3, gives 100 as a quotient.

Roots are readily determined in a similar way; thus, 4, in the arithmetical series divided by 2 gives 2, under wliich, in the geometrical series,. is 100, that is, the second, or square root of 10,000 the number under 4, is 100, the number under 2, and so on.

Napier called the first series the ltJgarith1R8 of the corresponding numbers in the second.

3. Since the two series may be assumed at pleasure, we may have as many different systems of logarithms as we choose.

4. The series in art. 2 being adapted to the common denary scale of arithmetic, is, on the whole, the most convenient for general purposes, though other systems have, in particular cases, their peculiar

advantages, '

On considering these series, it appears that the logarithm of 1 is

• The identity of this proceIII with that performed upon the exponents of quantities in the corresponding operations of alaehra, Will be obvIOUS to thoee who have acquired the rudiments of that branch of matDematiCII.

(A) Digitized by Gbogle

INTRODUCTION.

\

0, and that of 10 is 1, and hence the logarithms of all numbers between 1 and 10 are greater than 0 and less than I, that is, they are fractions. In the same manner, between 10 and 100 they are greater than 1 and less than 2, that is, they are 1 with some fraction annexed, and so on. The ",hole numbers or integers in the logarithmic series are hence easily obtained, being allM)I8.a unit less than the number of figures in the Integral part of the corresponding natural number. On this account it is customary, in the common printed tables, to put down only the fractional part in the form of a decimal, the comJ?uter supplying the whole number or integer under the name of ,nde.2:.

5. In order to generalize, let us assume the two following series:

~, ,.; , ";', r"", &c. • . (1)

11, !I', !I', !I''', &c. (2)

in which r IS some given number greater or less than unity, and .2:, .'1:', 111", .2:"', &c. any variable quantities chosen in such a manner that ,.. '9, r'" :y',";' :!I", r'" !I", &e., then the several exponents, .2:, e', :£", :r''', &c. of the senes (1) are called the logarithms of the corresponding terms in the series (2).

Thus if!l, !I, !I', !I"', &c. be a series of numbers such that,.. :y, ~'_!l,.,.a- y". r"'''=y''', &c., then ol'=log'!I, :£' 10g.!I', "'-log. !I", .r"=log. !I", &c.

6. For the purpose of adapting the series (1) to the seriel of natural numbers 1, 2, 3, &c. the given number r must be greater thm unity, the first index .2: must be equal to 0, and the several indices ol", '111", {I"', &c. must continaaHy increase. For, since by the principles of algebra, .2:°=1, whatever r may be, this seriet will inerease from 1 to irifinity J and by properly adjU8tin~ the nlues of Il', 31", 1£"', &c. it is evident that the eevcral quantities r"', rill, r"', &c. may be made to coincide :with the numbers 2, 3, 4, &C. For example, let· r~IO; then, sittee 100=1, find 101::10, the indices of 10, whIch 'WOuld give 10", 1()C1J, 10&'/', &c. equal to the numbers 2, S; 4, &c., must be fractions between 0 and I. If we take the number 3 we hIM! 10'l::3.16 nearly, from which 'We infer that a fraction (or') somewhat less than t or ()O5, being made the index of (r) 10, would give lor=3. This fraction is found by aloulation to be '47712; hence 10'477U=3; therefore, when r=IO, the logarithm of 8 i •

. 47712. ' .

_ In like manner, if we allSume the Ill1mbe!l" 5~ whose logarithm i. to be found in place of that of 3, we have lolt_4.64 whence a fraction, z<"Y somewhet greater than I, or .666 being made the index 01'

exponent of 10, would give 1000tsJ' =5. This fraction more accu .. rately com:puted is found to be .69897, that is, when r=1O the logarithm of 5 IS .69897.

7. From this it appears, that the value of the logarithm of any given number depends upon the value of the number r, and that by assuming it equal to different numbers, as many different systems of logarithms may be formed as we please.

In every system, however, since r°=:I, the logarithm of 1 must be O. This constant quantity r from the powers of which the natural numbers are formed, is called the radix or base of the system to which it belongs.

a. In the general equation v" ~, (art. 5.). let us make .2: vary and observe the correspondent variations of .'1'

Digitized by Google

LOGJ.lUTBMIC T~LES.

3

If r is greater than 1, on making .=0, we have y:::1; when ~=1 then y=r or the logarithm of the base is=l; in proportion as Jl in .. creases from 0 to infinity, .'1 will increase from 1 towards r, and afterwards to infinity, 80 that if we suppose z to pass through all the in-

termediate values, in fallowing tke lmr1 qf CO,,=' !J will increase

also mthe same manner, though much more . y.

If we put for e, negative values, we ahall have !J=r-lt, or 1 9=,..'

Here we see, in like manner, that the more z increases the

1 d h' .

more !J or,.. eereases, so t at m proportion as z augments, nega-

tivelY!J takes all possible values less than 1 as far as:O,'in which ca_ z becomes infinite. This was the proposition which Napier made to Briggs on their celebrated meeting at Edinburgh, when eonvereing

on the propriety of changing the logarithmic scale. .

If r i. less than 1 we shall make r= ~, b being greater than 1 aud

we have !J=! or !J=lr , according as z is positive or negative. We fall here upon the same case, with this difFereuce, that z is positive when!J is less than 1, and negative wheny is greater than 1. This propoBal Briggs made to Napier, but immediatly abandoned it on Napier nggesting that mentioned above, which was finally adopted.

If r=l, we have 9=1 whatever tE may be. . . We may then say generally, that provided r i& not 1lDity~ there can always be found a value for e, which renders ,.. equal to aay given number y. The constant use that is made of the properties of the equation s=r requires the denominations of its parts to be fix,;; ed in order to avoid circumlocution. Hence as before remarked, :r is called the logarithm of the number !J, the invariable number r i. called the base and, finally, the logarithm of a number, the power to which the base must be raised in order to produce that number.

With regard to the base r it is arbitrary, and when we write :r log. y to show that z is the logarithm of the number y or that !/_r". the base r is alway understood, because when once chosen it IS supposed to remain fixed. If it should be changed the new base ought to be indicated.

9. From these principles are derived several properties.

10. In every system of logarithms, the logarithm of 1 is 0 and that of the base r is 1.

2<'. If the base r is greater than 1, the logarithms of numbers greater than 1 are positive, the others are negative. The contrary takes place if r is less than 1.

SO. The composition of a table of logarithms consists in determining all the values of z when !J is made successively equal to 1, ·2> 3. &c. in the equation y=r" .

If we suppose rc=~ on making

z=O, C> 2(, 3(, &c. nc

We find 1/=1, 1", 1", ~5, &c. . . • ~n

The logarithms therefore increase in progression by differences, while the numbers increase in progression by the product or quotient, according as ~ is an integer or a fraction.

The ratios are the arbitrary numbers ( and ",. regard the systems of values of z and .'1 which

We may, therefore, satisfy the equation

Digitized by Google

INTRODUCTION.

9:=1"', .. cluaed in these two progreaaiona, which COincidel with what has been already said in art. (2.)

, 10. We shall now demonstrate algebraically the varioul propertiu of logarithms.

Let N and "be any two numbers belonging to the aeries (1); and,

for example, let N=r" and n::r', then N n=r"X""~, but, hy art. 5~ the logarithm of,.".- is z+.r=log. r"+log. ";:=log. N+'log.

II.

In like manner, if n, n', n" be any set of numbers in the series (1) it might be shown that the logarithm of n X n' X n", &c.=log. n+log. n'+log. n", &c., from which we infer that the logarithm of the product of any number of factors is equal to the sum of their logarithms.

11. Again N =; i but the logarithm of ""_'=Z-;e'i therefore; nr

the logarithm of N =z-z'-log. r"-log."" log. N-Iog. "i hence

r

it appears, that tlte logarithm of the quotient of any two numbers is equal to the difference of their logarithms i and that the logarithm

of a fraction (~) is equal to the logarithm of its numerator minus, the logarithm of its denominator.

If N be less than ft, then log. N-log. " ie negative; therefore. the logarithms of all proper fractions are negative.

12. Let N-r" be raised to the mtllpower, then N"'=,.-; but the logarithm of ,.- is=mz, hence the logarithm of N"':;mz=,,, log. r"

=m log. N; for the same reason, since:; N' Niii-' r;', the logarithm

of "'/N=~=log. N; from which we infer, that the logarithm oftbe

"J m m

m'" power of any number is found by multiplying its logarithm by m, and that of the mtll root of any number, by dividing its logarithm bym.

SECTION II.

Of the Con.ttruction qf Tables qf Logarit/tftu.

13. Let r" express generally any term of the series, (1), and let N be the corresponding number, then r"=N. Hence to find the logarithm of N is merely to solve the equation r"= N where a: is the unknown quantity. In order to accomplish this purpose let r=1 + b and N =1 + n, then extract the 1/" root of each side of this equation,

and we obtain (1 + b )~=( 1 + n );, which by expansion gives

l+i(b)+i (i-I) (;')+i (i-I) (i-2) (:.~) +&c.= 1+;(n)+; (i-I) (i)+; (~-I) (t-2) (;~)+&c.

Now suppose !I to be indefinitely great with respect to z and 1, then will =. and ! vanish in reference to -1, -2, &c., 10 that

.'1 s

z 1 .' h z .1

--1 and --1 Will eac become equal to -1; --2, --2, each

Y Y' !I s

Digitized by Google

LOGA.B.lTBMIC TABLES.

Ii

equal to _g, &e. &c.~ hence rejeeting 1 fioomeach tide of the eqaation we have

~ (6-1 611+1 6'-1 bC+&c.)=! (n-l n"+l n3-1 nC+&c.)

9 9

n-! nll+l n3-1 nC+&c. henee s, the log. (I+n) b-! bll+l 6s_.l hf+&c.

but n=N-l and 6=r-I, therefore, by substitution, the above expression becomes

(N-I) -l (N-I)II+~ (N-lt-1 (N-I)c+&c. (r-I) -§ (r-I)II+! (r-I)3-:l (r_l)f+&c.

. 1

14. Let (r-I) -l (r-l (r-l)i1+l (r-l)5-1 (r_l)f+&C.=

1 II

jj=M.

This quantity M, which evidently depends. upon the base r, is called the modulus of the particular system of logarithms to which it belongs. As it is obvious the series n -bn'+l nS-l n4+1 n&_ &c. will not converge when 11 is any whole number greater than uni_ ty, before proceeding to the calculation of the logarithms of any particular system, it will be proper to show the manner in which the value of z in the last article may be expressed in a converging aeries. This may be effected by means of the following process in which M is substituted for the quantity

1 thus,

(r-l) 4 (r_I)i1+~ (r-l)s-:l (r_l)f+&c.;

Log. (1+11) =M (n4 nll+j ns-t n4+~_n6-&c.). (3)·

In the above for n put -n, and then

Log. (1-11) =M (-n-§ rll-l n°-1I1·--} n"-&c.) . . (4) Subtract (4) from (3), then log. (1 + n)-log. (l-n)::::t1og.

!+:=2 M (n+l n'+1 n'+lIl7+&c.) (5)

I+n N-I

Let N = I-n' then n= N + n' hence

Log. N::2 M{ (~+!) +1 (~+'!)" +! (~+!)' +&c-} . (6)

A . 11th ) +n N h b bs· • .

gamet n=2 N-l' en I-n =2 N-l' ence y su titution In

formula . . . .. ... (5)

N (I 1 ) )

Log·N 1 =2M 2N-I+3 (2N· 1) + 5(2N-I)5 +&c. ~r

( 1 1 1

Log. N-log. (N-I) =2 M 2N-l + 3(-~N-I)5 +5(2N 1)5

) ( 1 1 1

+&c. ; and log. N=2M N-I +3 (N_I)3 + 5 (N 1)5 +&c.)

+ log. (N-I) . . . .. (7)

. I+n N+I 1

Laatly, if 1 n=N-' then n=2N+l and log. (N+I) =

(111·

2M 2N+l+3(2N+I)S+5(N+f)5+&c)+log. N ..

(8)

• Dr means of this f'onnula the logarithm of a quantity exceeding unity by a very BmaD I'rac:rion may be readily found.

Digitized by Google

8iDce the log. of 1=0, thU last aerie. whleh efJIlY8l'geI .,., rapidly, will give the logarithms of all the natural numbers, with facility in suceession, To these theorms might have been added others still more convenient, but they are sufficient for ordinary cases.

15. Before proceeding to compute a table of logarithms, lODle value must be assigned to 1\1' Since the value of r i. arbitrary, let

it be so assumed that (r-I)~n-I)/+~ (r-I),--&c. or M shall be equal to 1, that adopted by apier. Taking series (8) we have

since •

Log. 1 =0 (art. 6.)

(1 1 ] )

2=2 a+S4+5.3&+&c. to 8 terms

(1 1 1 )

3=2 5+3.53+gs+&c. + log. 2

4 =2 log. 2 (art. ]2) . .

(1 1 1 )

6=2 9+3.~+5,96+&c. +log4

6=log. 2+1og. 3 (art. 10) . .

(1 1 1

7=2 13+3(13)3+5(13)&+&c+log.6

8 = 3 log. 2 (art. 12) . . . 9=2 log. 3 (art. 12)

10 = log. 2 + by 3 (art. 10) &c.

In this manner the Napierean logarithms of all the natural numbers may be found. As their accuracy. however, depends. UpOR t.hoee immediately preceding, being derived successively from each other, it would be necessary to check the computations in the actual. construction of a table of logarithms by some independent formula, such as (6), though this in large numbers would be rather inconvenient from its slow convergency.,

16. To find the value of r. the base, in this system recourse must be had to the series (3) art. (14). If log. (l-n) or log. N be put =1 and M=I, we have I' n-in+!n3-!,,4;.,'&c.; reverting this

series, and] +u, or N=I+I+lz!+2~ P+2.~.4. [4. &c. Now let 1=1, then the number whose logarithm is 1, that is, the' base

1 1

r=I+l+i+2.3+2.3.4 +, &c. -2.7182818. To prevent confu-

sion, however, we shall always designate the base or radix of this system by R, retaining r for that of the common logarithms. Hence

R=2·718,281,82846. '

These are also called hyperbolic logarithms from their application to the quadrature of the hyperbola j but this designation is improper, as any system may be similarly employed.

17. When we have the logarithm of a number N for any ,articular value of r, the base, we can readily obtain the logarithm 0 thesame number in every other system. Since, art. (5). when the base is r we have rY=N, we shan likewise have RX=N when the base is R, in which :& is different from X, therefore, RX= r«.

INTBOOOCTIOK.

=~.693147S

=1.0986123

=1.3862944 =1·6094379

=1.7917595

=1.9459101 = 2.0794415 =2.1972246 =2.3005851

Digitized by Google

LOGAIU'l'BIIIC TABLES.

f

Now takmg the logaridDe....,. to the .,._ wbGee .... ia r, ..

/ l. RX=L,..

bat l.,w=z by hypotheais. and I.Rx =X loR, art. (12), whence X loR::.,

or X=~.But if R is the base, X will be the logarIthm of N in the

1. n.

system having that balle, and deaigna~ this by L.N to distinguish

it from the other, we shall have L.N=~:R . . (12) consequently we obtain the logarithm of N in the second system, by dividing its logarithm taken in the first system by the logarithm Of the base of the second system. Again from formula (12) we get

L.N X l.R=l, N . . . (13)

Hence in every system the logarithm of any number is the product of its Napierean logarithm by the logarithm of R, called the modulus.

Alao since ~.~=l. R, there exiBtl between I. N and L.N a conatant ratio represented by I.R .

Since we have by formula (12) L.N=~.~, as N=IO, then art (15)

I 1

2.3025851 = M' or M = 2.305!5851 = 0.4342944819, and 2 M =

0.8685800638 • • • • (14.)

18. It is now easy to construct a table of common 10garitIuM woo.e base r=10, for by fomrula (IS) we·have I.N=l.R)( L.N, bllt l.R=M = 0.4342944849; conaequently I.N =0.434W74B19 X L.N. It therefore only is necessary to suJ.titUte this value for M in any of the eeriee furmerly give.for the camplltatiOO of.the Na~ .. garithms to obtaill the common; thus, if in lelia (8) iOr 2 M we .. bstitute its value 0.86858896 we &ban have

. (I 1 1 )

log. (N+I)=0.86858896 2N+I + 3(2N+I)3+ 5(2N+I)6 + &c.

+ log. N, and making N successively 1, 2, 3, &c.

Log. 1 = . . . . . 0.0000000

(1 1 I )

2='86858896 8+34"+5.86+' &c.

'I 1 I

3 ='86858896( 5 + 3'53 + 56 +, &c. ) + log. 2 =0.4771213

4 = 2 log. 2. . . . . =0.6020600

(1 1 1 )

5 =-80068896 "9 + 3.93 + 5.@ +, &c. + log. 4 =0·6989700

6 = log. 2+10g. 3 •• =0.7781513

(1 1 1 )

7 ='86858896 13 + 3(18)3 + 5(13)5+' &c. +log. 6. =0.8450900

8 = 3 log. 2 . . . . =0.9000900

9=2 log. 3 =0.9549425

10 = • . . . . 1.0000000

19. After Lord Napier had computed bis first tables of loguitlmo it occurred to him that it would be proper to change the radix R=2.7182818 tor=lO, at the laDle time making the logarithms of ~ tegers positive, and those of fractions negative, (art. 8.), 8S more c0nformable to the denary scale notation, and more convenient in practice. It appears that Mr Henry Briggs had also conceived the idea of

Digitized by Google

=0.3010000

a

LNTRODUC'I'lmI.

chulgillg .. radix, aad had compatild: Iosarithms 011.8 plan 11~ what less commodious, by making the logarithms of integers n~ tive, and those of fractions positive, which, upon a personal communicatiOn with Lord Napier, he rejected, and finally adopted his lordship's views, He soon afterwards published the first thousand 10garit6ms of this kind under the title of Loga1'itktnONlfll Chilim Prima."

SECTION III.

" "

Of the Trigonometrical Lines, called Sines, Tangents, ~c.

20. THE Egyptians and Chaldeans began to sludy astronomy at a very early period. As the determination of the relations and distances of the heavenly bodies involve the mensuration of lines and angles, it was necessary to invent some method of ascertaining the value of these quantities, at least in an approximate manner, before any use- • ful results could be obtained. Some of the more elementary propOlitions in geometry must have been discovered in the most remote antiquity, and the inventive genius of the Greeks filled up the general outline. The properties of geometrical figures thus acquired, would, withoutdouht, be applied to the mensuration of several mag~ nitudes, and the distances of various points in space. About six hundred years before the Christian era, Thales measured the height8 of the" pyramids in Egypt by means of their shadows; a method which depends upon the proportionality of the sides of similar triangles. This simple" property forms the basis of modem trigonometry. If, for example, a pole or gnomon be ~et perpendicular to the horizontal plane, it will, In a clear day, when the sun is not vertical, caat a shadow to a given distance, while any other high object, such as a steeple near, it will do the same. If straight lines be conceived to be drawn from the top of these objects to the extremity qf each "of their shadows, it is evident that, unless they are very distant, by this means triangles nearly similar will be formed, whose sides are

proportional; that is, as the shadow of the gnomon is to its height so is the shadow of the object to its height. Now, suppose the length of the ebadow of the ~nomon to be made the radius with which an arc of a circle is described commencing at the bottom of the gnomon, and, as will be afterwards explained, measuring the angle between the horizontal line and the line from the extremity of the shadow to the top of the gnomon, that gnomon will, by the principles of geometry, be a t~ent to the circle. Whence the former proportion becomes as the radius is to the tangent of the angle of elevation, so is the length of the shadow oftheobject to its height. It would thus require the length of the shadow of the pole or gnomon to be measured each time any height was determined. This, however, might be avoided by having the measure of a set of triangles whose sides, to an assumed radius, and a corresponding series of angles, are previously determined by computation. By this means, in such cases, it is only necessary to measure the angle of elevation of the object, at a given point, and its distance from it, and comparing it with one of those computed triangles equiangular to it, to determine, in a manner similar to the former, the height of the object. It is obvious that the same -principles may be applied to objects situated in any plane, whether ..-ertical, horizontal, or oblique.

" Several series of triangles of the kind now mentioned have been

Digitized by Google

TRIGONOMETRICAL TABLES.

actually computed and arranged jn·tables under the designation of tripomecrical tables.

These. were not accomplished at onee, but were the improvemem., of successive ages. Hipparchus, about 150 years before the Christian . era, supposed similar triangles to be inscribed in circles, and employ_ ed in his computation the chords subtending the arcs measuriog them in sexagesimal parts of the radius. Nearly, 300 years at\er_. wards, Ptolemy, in his M1'Y"''>'1I l:1I",.~lf' recomputed the chords but in his Analemma employs the /tall chords instead of the ckorrh approaching. very nearly to the use of sines, afterwards introduced by the Arebians,

Some notions of the tangents, secants, and versed sines, were,· towards the beginning of the tenth century, entertained by the more learned Arabians. About the beginning of. the fifteenth ceRtury the sciences began to be cultivated in Europe, where the greatest pro. gre&s has been made. At that period Miiller invented the ~ents, and shortly after Maurolycus produped his table of secants. I'hese were all in natural numbers to a gIven radius now generally taken at unity, and, therefore, their application was in many cases troublesome. To remove this inconvenience as far as possible, Napier invented his logarithms, which have brought them perhaps to the last dearee of perfection.

Hipparchus, who has been followed by most of the moderns, ell1- ployed·the circle to measure angles. He supposed the whole circumferenee to be divided into 360 equal parts each called a degree. The degree was divided into 60 equal parts called minutes, and the minute into 60 equal psrta called seconds, and the sexagesinlal division was continued, though now the fractions of seconds are more com~l expl'e8led in decima1a, which are more convenient for ealeula-

bOD. .

Whence the semicircle contains 180 degrees and the quadrant 90.

Asfour right angles can be constituted about a point, 90 degrees must be the measure of a right angle. For the purposes of abbrevilJtion a degree is marked with a smalLeircle, a minute with one accent, a second with two accents, &c. Thus 57° 17' 44".806, denotes 5'1 degrees, 17 minutes, 44 seconds, and.806 the decimal, whose value is 006 thousandths of a second. This, being an arc whose length is equal to the radius as will be afterwards explained, is also expressed in degrees and decimal parts of a degree, thus 57°.2957795, a mode of using it, which in IIOm_e cases has its advantages.

The number of. these parts, in either case, contained in the arc f:aetween. the linea OODstiwting the angle, of which arc the angular point i8 the centre, indicates the measure of that angle accordingly.

Hence, if to any number expressed in sexagesimal degrees 0RerU1Ith of itself be added, the 8um will be the same number expressed in the centesimal degrees; and if from any number expressed in centesimal degrees one· tenth of itself be subtracted, the remainder, will be the same number expeessed in sexagesimal degrees.

• The French have lately adopted the centesimal division, which, in many cases, ill preferable to the sexaglfimal. The whole circle is divided into 400 d~ each de. gree into 180 minutes, and the centesimal di vi'!oion is continued. Hence the semicircle containil200 degrees, the quadrant 100, and the ratio of the eentesimal to the sexege •

• imal is as 9 to 10.

To COIIl:ert Wt~al ~es into unWBimaladd .~ of th.e are to itself. .

The convene is effected by lubtracting ll~ of the arc from Itself.. C . I

. .' Digitized by ('lf~oog e

"~J1'IO."

il. If two BtraiSht lines interaect one another in the oatre of a cit~, the ll1'e uf the circutnf\trence intereepted between them i. called

the roMsU1'e of the contained aftgle. w"bate:ver be D

th11 radius of the circle, since the ares are pro~onal to their radii. Thus, the are AB or A'B', 1. the measure of the angle ACB, and is expl'eteed in degree!!, &e.

ill. The complement of an arc is its difference m.. a quadrant, its IUpplement, its difference from a semicircle, and its ezplement, its defect from the whole circumference. Thus if AB be any arc, tho BD is the complement, BE the supplement, and BDEFA the explement.

Tbe same thing holds with l'el'Ud to the angles of whieh t1ae &llCS are the measures, that is, if ACn be any angle, BCD ita difference &om a right angle is called the complement, BeB the IUj)p}em8llt to two right angles, and BOA, meaSured by dle arc BDE:P A, tlhe ekplement or difference from fuur right angles.

!S. The aine of an are, or of an angle of which the arc i. the measute, is a perpendicular let fan from one of its extreini .. upon a rajfillS or diameter passing through the other.

94. The wrIed sine or tJernne of an arc i. that pat of the diameter

intercepted between its sine and the circumference. .

25. The tangent of an arc is a perpendicular to the extremity of the radius at one end of the are, and limited by a straight line drawn from the centre passing through the other.

26. The secant of an arc is the straight line drawn from the centre to the extremity of the tangent.

~. It is usual to express the sine, tangent, and 'OOlJflt of the t'6IfIplement of an arc by the abbreviated terms COline, cotangent, aad

coaecant. .

28. Let ACDE be a circle of which the diameters AD and' CE are at right angles to one another.

Take any arc All, produce tile radius OB, and draw BO, AK perpendicular to AO Or AD, and HB, CI perpendicular to CE; then BB is the D ,ine, BH or 00 the cosme, 2\.G the veraifttl, CH the coverai"e, DG the suvernne, and HE the 8UCOfI~ mae of the arc AB. Also of that arc AK is the tIn$ent, CI the cot(Jngent, OK the secant, and 01 the cOIeN .. t.

29. Since the diameter which biseets an are, also bisects the chord of that arc at right angles, therefore, the sine of an arc is equal to half the chord of twice the arc. Thus BG=i BF-half the chord of the arc BAF, the double of the arc AB.

30. In the right-angled trian~le QGB, BG'+OOilcOB', that is, the squares of the sine and cosme are together equal to the .quare

of the radius. .

31. The triangle OGB being similar to OAK; 00: GB:: OA: AK, or the cosine of an arc is to the sine as radius is to the tangent.

32. Also the triangles OGB, OAK being similar, as before, OG : OB : : OA: OK, the radiue i. a mean proponioDal ~ween the

cosine and the secant. .

33. Sinc, DO : GB : : GB : GA, it follows that the sine is a mean proportional between the versine and suversine.

~. Again. AD: AB : : AB: AG, or the chord of an arc i. a mean proportional between the diameter and verslne, Google

Digitized by ,

TRIGOHOHETIUC4}. TABLES.

11

Cor'.-Since AB8=AD. AG, th~, becaUIe AD is constant, AD" .arietI .. AO, or <t AD)' ~ AAJ, ~bat is, the .,~ of the Ii. qries direetl1. as the venine, or inversely as the coun~: of twice the arc. aG. The triang\,s <)4K 'and leo ar4 eiR1ilar, tJlerefbte At( , AO: :

OC : CI; con~ueritly ttle radius i. a meatlproportiona1 I)ltWeen

the 1angent and cotangent of an aro. .

38. In the application of algebra to geometry, .here the trigonometrical lines are employed, it is necessary to trace their changes in the several quadrants of the ml'cie, since it is obvious that the .-me lines treated of above, may be applied to each. X

In the first quadrant AC, if the sine BG and cosine " l~

GO be supposed poIiti"e, then the sine B'G' on the Di)t;I~~

8IUIle side of the diameter AA', and in the l&Dle direction, still remains posit;,,#!; but the coone OG' A~I--lit-lf:::,:-f baving changed it8 position with respect to the (BlUe 0, or diam~ CC', ~es ~"'"

fa the third quadwmt, the ClO8JD8 AG' aJla siD. .

O'B", h4ving bcdl Oh~ged their positiOll8, ere I d r ii.'

b.&h .galittll. lit the faunh ~ant, the cotine : - .

laay.jng~ed f~ ~tUJl""_, OG i. DctI1rpoftmltJ, wbil. tile line GB""J remaining III in tfae third quadraat, i. ,.,gati... The tupbt1I ADd .ecanc:. depeDdmg up .. the lin. __ ~. AaY8 dMir

.. cleterminecl accordingly.' ,

J'WlIA M!ttcw SG;to 3Ij aDi iDChuive, R beiDg radiua, &e. we Qbtain

I, sin, = (R!I-C08.lI)~ 7. tap. =~

cos.

_R x cos.

-8iiL

R8

2. cos.

a. cot,

3. tan.

9 ....

cos.

R· 10. cosec. - sin

sin8 11. versine =-R - +coe. cos I

.6. couc. :: (R8+dot.8)i l~. COY81 .. ;;;.+jn.

If radius be supposed unity, then

sl!!.

1. ain. = (l--coa.·)~ 7. tan. _

cos.

4.cot.

6.aec.

cos.
~ cos. = (l-sin.t)~ 8. cot. - ...--
UD.
S.tan. = (sec.·-I)' 9.~. 1
: -
cOa.
4. cot. =(coaec.·-l~ l~. eotIet. s::= 1
'"'I'-
8lD •
. ,
5. see, = (l+tan.·)~ . 11. "eraine:::: 1 un.
+ cos.
8. coaee. :::: (1 + cot. .)6 co •• '
11. cover8.= --. -
l+un. • JD the"e WGOd-cut, JYI hal been CIII1itted near 1"', which may easi1ybe supplied by tbepftl. ,

Digitized by Google

]2

INTBcmucrlON.

31. Now, since (7) tan: = ~n ; thenitfollowsfioomtbe principles cos.

· of algebra, that when the signs of the sine and cosine are like, the

· sign of the tangent iS1!0sitive, arid when unlike, the sign of the tangent is negative. In like manner, the signs of the cotangent, secant,

· and cosecant may.be determined from formulas (8), (9), and 10).

Table if the Signs if T1-ignometrical Lines.

Quadrants. { Sine. COIIine. Tangent. Cotaogent. Secant.· Coteeant. }

159 + + + + + +

2 610 + - - - +

3 711 - + + - -

4 8 12, &c. - . + - - + -

Of the Multiplu and Poeer« if Arcs .

. 38. In most treatises on geometry, saches Leslie'S, Legendre's, &c.

· the elementary propositions containing the principles of trigonometIy are also given. It is therefore unnecessary to repeat them bere, as It : only puts the' student to the expense of purchasing the same things in two or three different works. We shall only give a few of the re'. suits' most generally. useful; referring totholle works on geom_etry and trigonometry where the requisite information _may be obta.iried.· , If a and· b are'two 'given arcs' of a circle of which the radius is unity, then

. sin. (a+b)=sin. a cos. b+sin. b cos. a (1)

cos. (a+b)=cos. a cos. b-sin. a sin. b (2)

sin. (a-b)=sin. a cos. 6-sin. b cos. a. (3)

cos. (a-b)=cos. a cos. a+sin. b sin. a . (4)

If we divide these equations, the one by the.other.in sueeession, that is, (1) by (2), and (3) by (4), then

(sin. a cos. b + sin. b cos. a tan. a+b)

cos. a cos. b-sin. a sin. b

(5)

sin. a cos. b-sin. b sin. a (6)

tan. (a-b)

cos. a cos. b + sin. 6 sin. a

Dividing the two terms of the second numbers by cos. a cos. b, and

· substituting tan. a and tan. b for their values in terms of the sine and cosine

tan. a + tan. b

tan. (a+b) .

I-tan. a tan. b

( b) __ tan. a-tan. b

tan. a- - b

1 + tan. a tan.

expressions which give the tangent of the sum and of . the difference

of two arcs in terms of the tangents of these arcs. .

If we make a=b in the preceding formulee, they.give

sin. 2 a=2 sin. Ii cos. a, . . .

cos. 2 a=c08.!la-sin.1I a

2 tan. a

tan. 2 a=l_tan.!la

(7)

(8)

(9) (10)

(ll)

, • Those we would more particularly recommend are the treatises of Gregory, .. Woodhouse, Lardner, and Cagttoli. Dr Kelly's Spherics'is'a'vey good treatise for teaching the practice of the stereographic projection of spherical triangle.. '

Digitized by Google

TRlGONOHETlllCAL TABLES.

13

expressions which give the sine, coeioe, and ~ of twice the arc ill terms of the sine. cosine, and taogent of the simple arc.

39. Returning to equations (1), (2), &c. we have by addition and

subtraction

sin. (a+b)+sin. (a-h) = 2 lin. a cos. ,. (12)

cos. (a+b+cos. (a-b) = 2 cos. a cos. b (13)

lin. (a+b)-4;in. (a-b) = 2 sin. b cos. a (14)

cos. (a-b)=cos. (a+b) =2 sin. a sin.b (15)

Let (a+ b)=u, and (a-b) =v, then by addition and subtraction a=! (u+v), b=l (u-v), consequently the preceding formulae become

sin. u+sin. v = 2 sin. i (u+v) cos.! (u-v) (16)

sin. u-sin. v = 2 cos. (u-v) cos. (u+v) (17)

cos. u+cos. v = 2 cos. (u+v) cos. (u-v) (18)

cos. v-cos. u = 2 sin. (u+v) sin. t (u-v) . (19)

expressions which serve to transform the sum or the difference of the sine or cosine into the product, and thus to unite the two terms into one.

If we divide formula (16) by formula (17) they give sin u+sin. v tan. * (,,+v)

sin. u-sin. v tan. t (u-v) . . (20)

If we multiply these equations member bI member, observing to substitute sin. 2 a=2 sin. a cos. a, formula (9), then

sin." u-sin.llv = sin. (u+v) cos. (u+v) (21)

COs-II v-cos.llu = sin. (u+v) cos. (u+v) . (21)

Since sin. 2 a=2 sin. a cos. a, and cos. 2 a=cos.1I a-sin." a.

The second of these equations may be put under the two following

·forms: . .

cos. 2a=I-2 sin.1I a, and cos. 2 a=2 cos. II a-I

l-008.2a 1 + cos. 2a

whence sin.lla = 2 ,and cos.tia 2 (22)

These expressions are used when, for the squares of the sine and cosine, the first power of the cosine of the double arc is substituted.

40. Let 24=11, then a_til formula (22), these formulae become

l-cos. tI 1 + cos. II

sin.II~U=~-, cos·lIiu= 2

and dividing each corresponding number successively, they giTe l-cos.u

tan~lI tU=I+cos. u

l-tan.1I iu

and cos. u l+tan.ti tu. ". (25)

If b in formulae (1), (2) be made 2 a, 3 a, &c. we may obtain mul-

tiple arcs thus: '" .

sin. 3 a=sin. a cos. 2 a + sin. 2 a cos. a

cos. 3 a=cos. a cos. 2 a-sin. a sin. 2a Substituting for sin. 2 a and cos. 2 a, their values, sin. 3 a=3 sin. a cos.lla-sin.3 a

cos. 3 a=--3 cos. a Bin. a + cos.3 a

These may be put under the form

. sin. 3a::cos.3a(3tan. a-tan:'! a) cos. 3 a=cos." a(l--3 tan.1I a)

(23)

(24)

they become

. ~~~

Digitized by Google

I.

JNTltODUCTION.

In general ,. being any integer,

-. { "("_l~.(n-a) 3

sm "a=oos'" a II tan. II- 1..8 tan •• +

".(,,_I).(n-2).(n~).(n-4) 5 }

1.2.3 .•. 5 tan. a ..• &c. •

{ n(n-I) n(_I)(n--~"_S). 4 }

COlI. na=cos.fta 1---r:2 -=-tan. II a+ 1 . 2 . 3. 4~n. aAc. (29)

The coefficients of the difFerent terms are those of the "eA ~wer ()f the binomial. whence t4ese series may be collected under the follow-

ing form: .

1 { }n 1 { }n '

sin. n a=2,J-l cos. tJ+,J=r sin. a -~",'-1 cos.a-,J=t sin. a (30)

cos. n a = t{ cos. a+,J=I sin. a } II + -i- { cos. a-,J-=i sin. a } ft (31) These formulle, by development, will give the two foregoing series.

and are thus easily verified. -

.1. It may be shown- that if:r represent any arc .

• z3 z5 • 1J:7

sm. :r =:r -1.2.3 + L2.3:D -'1.2.3.4.5.6.7 +, &c.

1J:'iI 1J:4 :z:6

cos.:r = 1- VI + 1.2.3.. - 1.2.3.4.5.6 +, &c.

In these expressions the arc II: is supposed to be divided by the radius. which is here ~en for the unit of length. and consequently

•• • • IJ:. sin. 01:

If we WIsh to restore It we must wnte r 10 place of .% and -r-

instead of siq. 01: in the two members of these equations.

These formulle might be carried much farther than can be introduced into this place. Moat oftbem maybe seen by consulting the books already referred to. but above all the aflalyais infi"itoN"n of Euler.

(29)

(32)

• Woodhouse's Trigonometry, third edition, page 24li-Gregory, page 42 and 50.

Digitized by Google

TRlGONO)lETll.ICAL TABLES.

]I

5.

6'.

sin. • = sin. a cos. a = COl. a

2 ain.lla=1-coe.2a 2cos.lla=l+co •. ~a

4 ein.' a=3sin. lI-sin.8 a 4 cos.3a =3 cos. a+cos.3a

8 ain.' a =3-4 eos, 2 a+c08.4a 8 cos.4 a =3+4cos.2a+coB.4a,&c. 42. Having given a short abstract of the more useful formul. relative to multiples and powers of arcs, we shall now proceed &aabe" the method of constructing the tables of sines, tangents, &c.

When the radius of a circle is unity, the semi circumference is 3.1415926536 nearly. Now there are 1000 or 10800' in a semicircle. consequently, if the former be divided by the latter, the result will be O.OOO'J908882, the measure of an arc of one minute, which, as the arc is 80 small, may be considered its sine.

Now, art. 35. 2, cos = (1 - sin.1I)6 consequently cos. l' = 0.9999999577. If these values are subetituted in formulre, (32), and (33), art. 41 the sines and cosines may be obtained through the whole quadrant.

Thus let the arc a=I', and, therefore, sin . .t=O.OOO2908889. Let

a=ljo, then 5 X 3.1~~926536=O.08726646 the length of a or s, and

.t= + 0.08726646

;&3

1.2.3 = - 0.00011076

/IDS

+ 1.2.3.4.5"= + 0.00000004

.tIS 1115

therefore. z-I.2.3+ 1.2.3.4.5' &c.:=O.087]5574= the natural.ine

of 5°, the logarithm of which i. 8.740206, the log. sine the same arc. Thie method i. easy when the arc is small. as the series then converges ve~ rapidly, but it is rather laborious when the arc is large, in which case recouree mult be had to other methods depending upon the properties of multiple arcs. as may be seen in moat of our tre .. a.es on tri$onometry.

As the SIDes are computed. the cosinea of the l18me arcs may be found from art. 41. formula (33). or from art. 35, formula (~), the tan ... puts and cotangents. &om formula (7) and (8), and the seeanta and cosecants from (9) and 10).

SECTION IV.

Of the application qf Tables qf Sines. Tangents, Secants, 4-c. to

p/Dfle Trigcmomdry. ,

CASE I.

43. In any plane triangle it is shewn in our usual treatises, that

the aides are proportional to the sines of their opposite angles, or Tile sine of anyone angle,

Is to the line of another angle i As the side opposite to the first,

Is to the side opposite to the second.

Th~ term. may be taken alternately, inversely, &c.

44. When one of the angles is a right angle, then the pre. ceding rule may either be applied, or a modification of it derlved from the properties which are peculiar to right-angled triangles.

Di9ltizedbyGO gle

INTRODUCTION.

In right-angled triangles, it is usual to call that side subtending the right angle the hypotenuse, and the other sides which contain the right angle the legs, or the one the base and the other the perpendi. cular.

Then if one of the sides of any triangle ABC, be assumed equal to the radius, the names of the other sides must be determined by art. 26, as follows :-

C

A .B Sine C

The names of the sides being thus known when three of the parts of a triangle including a side are given, the rest may be found by the following rules:-

1.- To .find a side.

As the name of the given side,

Is to the name of the required side; So is the given side,

To the required side.

II.-Tofind an angle.

As the side made radius, Is to the other given side, So is radius,

To the name of this side.

Any side may be made radius to find a side, but one of the give"

sides must be made radius to find an an~le. .

In the solution of plane triangles, it ni'ust be recollected that all the angles in any triangle are together equal to two right angles, or 180°. Whence if two of the angles are given, the other may be found by subtracting their sum from 180°; when one angle is given the sum of the other two may be found by subtracting it from 180" ; and if one be right or 90°, the sum of the other two is also 90°, and the one is the complement of the other.

CASE n.

45. In a plane triangle when the two sides and contained angle are given.

I. As the sum of the given sides, Is to their difference;

So is the tangent of half the sum of the opposite angles, To the tangent of half their difference.

Half the difference added to half the sum of those angles gives the greater, and subtracted from half the sum gives the less.

All the angles being now known, the third side may be found by the rules in case I.

OJ', after having found half the sum and half the difference of the angles, the remaining side may be found without determining the actual angles, as proposed by Thacker in 1743, and recommended by Professor Wallace, in the Edinburgh Philosophical Transactions, in the following manner:

Digitized by Google

PLANE TRIGONOMETRY.

II. As the sine· of half the difFerence of the oppoiite angles, Is to the sine of half their sum,

So is the difference of the containing sides ; To the remaining side j or,

Ill. As the cosine of half the difference of the opposite anglea, Is to the cosine of half their sum;

So is the .rum of the containing sides

To the remaining side.

These two methods may be used as a verification to f!ach other, and will be found somewhat more easy in practice than the fiioat method, as several of the quantities may be taken out from the trigonometricel tables at the same time.

Should the sides come out in logarithms from some previous operation, then Gauss' table for finding the logarithm of the sum and difference of numbers from their logarithms, without first determi~ the natural numbers themselves, would be some advantage, though it was not thought sufficient to warrant an insertion of it among the tables.

The following method of resolving this problem is convenient, par_ ticularly when the logarithml of the sides are given.

IV. From the logarithm of the greater of the two gi,ven sides, having its index increased by 10, subtract the logarithm of the Ie. side, the remainder will be the logarithm tangent of an arc, from which, 45° being subtracted, there will be obtained a remainder. To the logarithm tan~ent oftW. remainder add the log. tangent of half the sum of the oppostte angles, the sum, rejecting lOin the index, wiU be the log. tangent of half their difference, from which the angles themsel v .. may be found.

CABB III.

. 4.6. In any plane triangle, when the three sides are given,

I. As the base .

Is to the sum of the sides ;

So is the difFerence of the sides

To the difference of the segments of the base made by a perpendicular upon it, or upon it produced from the opposite angle.

It may perhaps be convenient to call the longest side the base, in order that the perpendicular may ran within the triangle.

When the three sides of a triangle are given, the difference of the segments of the base may thus be found. Then half the- difference added to half the sum, that is, to half the base, will give the greater segment adjacent to the greater side; and half the difference taken from half the sum will give the less. From these the angles may be found by Rule II. § (44).

II. In a plane triangle, as the rectangle under any two sides, is to the rectangle under the excesses of the semiperimeter above those sides; so is the square of the radius to the square of the sine of half their contained angle, as shown in Leslie's Geometry. In practice, this rule, when logarithms are employed, may be stated as follows:

To the arithmetical complements of the logarithms of the two sides containing the required angle, add the logarithms of the differences between those sides and half the sum of the three sides, then half the sum of these four logarithms will be the log. sine of half the required angle.

III. To the arithmetical complements of the sides containing the required angle, add the logarithm of half the sum of the three.sides'l

. Digitized by L.fC1l5>8 e

If

18

:urr:aoDUcrION.

and the ~tIuD of the dil'erence between thi. half lum and the side OPJJ08.l:te the required angle; half the sum of these four logarithms will be the log. ~ of half the requited an~e.

IV. To the arithmetical complement of the logarithm of half the lum of the three sides, add the arithmetical complement of the difference between half the sum of the three sides and the side ~posite the required angle, and the logarithms of the differences between that half SUID and the sides containing the required angle; half the I1IDl of those fom.' logarithms will be the log. ta"gent of half the required angle.

It may be remarked that these three last rules win, in general, be the most commodious in practice, though, in particular eases, each may_~ve ita peculiar advantage when great accuracy is required.

When the required angle aoes not exceed 90", Rule II. may be DIed, when it does, Rule III. may be employed i and in either case Bule IV. will give correct solutions. These observations depend _pon the variation of the trigonometrical lines in certain parts of the CU'cle, as, for example, near 90". the sines vary very slowly, 80 that dt.e true value of an arc cannot be obtained by our ordinary tables, while the tangents alway!! vary by such perceptible quantities as to leave no doubt of the real value of the required arc. These remarks may be easily verified by examining any of our tables extended to 1U or oven places of decimal ..

Of tie Corutruetion qf Tri4nglu.

47. Previous to the numerical solution of any triangle, it is generally ftrIt constructed geometrically. This is accomplished by means of what are termed mathematical instrumenta, consisting of sCales, compasses, &c. contained in a case, at various prices, to suit the convenience of purchasers, Printed descriptiOils of these, &I well .. of many others, are to be found in Jones' edition of Adams' Geometrical and Graphical Essays.

In the construction of plane triangles the &idea are taken from a ecale of e~ual parts, and the angles are laid down by a acale of chorda. or more conveniently by a protrac:tor.

EXAKPLHS.

CASH I.

48. 1. Given the angles and hypotenuse of a right-angled triangle, to find-the base and perpendicular.

Let the hypotenuse AC of the right-angled triangle ABC be 288, and the angle A 39" 22'; it is required to find the

sides AS and BC. LJC

Con.rtruction.-In the indefinite araight line AB h

take any point A, and by a protractor or scale of Q.

chorda, make the angle A equal to 39° 22'; from

an)' convenient scale of equal parts take AC equal .A. C B

to 288, and from C draw CB, perpendicular to AB ;

then ABC will ~ the triangle requir~d. In order to simplify and preserve uniformIty, the angles may, In general, be denoted by the C&})ital letter8 A, B, C, and the opposite sides by the smallleUeN II, b, c. The sides a and c being measured by the same scale from which b W&l taken, will be found to be 182.7 and 222.7.

Digitized by Google

PLANE 'nlGONOMKT"y.

Calcwlatio1l 1. By nat_ DaJQbers, § (48).

To find a.

& _ _z_ B . Ii I. sin.Axb

All..... : am. : : fI : a, or a::-sin. B

1 : 0-684281 : : 288 : ~1 X 988=181.673:=0 To find c.

ADd sin. B : aiD. C, or cos. A :: b : c

1: 0.773103:: 288: 0.773103x988~654=c 9. By logarithms.

It

To find G.

A14 sin. B, 01' radius Is to sin. A 390 22' So is b 288

To G 189.673

10.000000 9.802002 2.469392

2.261674

To find c.

Aaradius .

Is to cos. A 39" 22' So is b988

10.000000 9·888937 2.489399

To c _&3 9.3476'J9

The solutions may be varied by assuming any of the sides for ra.. dius, according to art. (44), and verified by Gunter's scales.

2. Given the angles and one side. to find the hypotenuse and the adler side.

Let the side AB be 758, and the angle C 39" 26' j to find the angle A, and the sides BC and AU

.4III_BC is 921.7, and AC 1198.36. and the ~le A 500 M'. Cmutrvctitm.-From a scale of equal parts make AB equal to 758. &be ~le A 500 34', the complement 01 C, and draw BC at right mgles to AB j produce AC arid BC till they meet in C; then ABC is the ~le required. and tI and b measured on the same scale from w1dcb c was taken will be found to be about 922 and 1193 respectively.

3. Given the hypotenule and one side, to find the angles and other aide.

Let the hypotenuae AC be 544. and the base 464 j to find the angles A, Q and c. and the side Be.

"' __ The augl. A. is IIQ ~', though Cis Q80 au' _Ie .. Coutntetion.-Make AB equal to 464 trom a Kale qf equal paN, aad from B draw Be perpandiou'" to AB. tho m.a the ,*,U(l A • the distance AO equal to 644 delra"i1>4 III arc iltteraecti., BC • C. join AC. and the triangle is constructed. The angle A. Wn, measured by & pr:ea.etor o\' ale 01 RoMs, will be i»ud to be 310 _. consequently C is 5SO 32'. and the aide BC 284 trom the laDle scale by wruch the other aides were laid down.

4. Given the beee and perpendicular, to find the angles and hypo-

tenuae. G I

Digitized by oog e

INTRODUCTION.

Let the base AB be 558, and the, perpendicular Be 456; required the angles A and C and the hypotenuse AC.

Aru.-A 39° 15' 21", and 50" 44' 39", and AC ~.69!. .:

Construction.-Make AB equal to 558, and draw BC perpendicular to AB and equal to 456, join AC, and the triangle is constructed. The angle A will measure 391°, and the hypotenuse will be·about- 721 nearly on the scale of equal parts. The other side may be found by Euclid I. and 47, or Leslie's Geometry II. 10, and 13.

5. Given the angles and one side of an oblique-angled plane trian-

gle, to find the other sides. '

In the triangle ABC, are given the side AC, 532, the angle A 38" 40', C 92° 46', and consequently the angle B 48D 34' ; to find the sides AB and BC.

An8.-AB 708.76, BC 443.34.

Construction.-Draw the indefinite AB, at A C .

make the angle BAC equal to 38<> 40', and from a ~ scal~ of equal parts make AC 532, at C. dra:w CB ~ makmg the angle ACB equal to 92D 46', It WIll cut _

AB in B forming the triangle ABC which was re- .A C B

quired.

6. Given two sides, and an angle opposite one of them, to find the other angles and the third side.

In the triangle ABC are given the side AB 274, AC 306, and the angle B 78° 13'; required the angles A and C, and the third side BC.

Ans.-The angle C is 6r 14', the angle A 48D 33', and the side BC 203.22.

COn8truction.-lUake AB equal to 274, the angle B equal to 7f1J 13', and with an extent equal to AC, 306, intersect the line Be in C; ABC is the triangle required.

If in this triangle the side B be greater than C, there may be two triangles formed, constituting whatis called the ambiguous case, tbatis, itadmits of two solutions, either of which answers the conditions required, unless from some known circumstances one of them must be adopted in preference to the other.

Thus in the oblique-angled triangle ABC there are given AB318, BC 195, and the angle A 32D 40'.

Ans.-The an~le B is 610 50' or llSO 20', the angle C is 859 40' or 29°, and the SIde AB is 360.246 or 175.15.

Construction.-Make AB equal to 318 from any C·

convenient scale of equal parts, the angle A equal ~

to ~ 40', and with the centre B and distance equal _ C a.

to BC 195 describe an arc cutting AC in C or C' ; b a.

ABC or ABC' will be the triangle required. ,

CA.SE II. ,A C B

49. Given two sides and the contained angle, to find the other a,les and the third side.

n the triangle ABC let the side AB be 920 and AC 500, and the contained angle A 36" 5W ; required the angles B and C, and the third side BC.

An8.-B is goo 58' 50", C 113C1 0' 10", and Be ia 600.31.

Digitized by Googk

PLANE 'falGbIlOME'l'S.Y.

il

COII8tnu;tiotl.-Make AB equpl to 920, at the C

point A make the angle BAC equal 96<> 5i', and 6a AC equal to 500 ;. join BC; ABC is the triangle

required.

-A C B

By Calculation, art. 45, I.

As AB+BC 1420. 3.15~

Is to AB-BC 420 2.623249

So is tan.! (B+C) 71q 34' 10' 10.477162

To ~ ... (B-C) '41 35 10 0.948128

C 113 9 10

B 2958 50

As sin. B 290 58' 50"

Is to sin. A 36 52 0

So is AB 500

To BC 600.31 .

Or bl art. 45, II. and III.

As SID. ! (B-C) 410 35' 10" Istosin.i (8+C)71 34 0

So is AB-BC 420

To BC 600.31

As cos. i (B-C) 410 35' 10" Is to cos. t (B+ C) 71 34 0

So is AB+BC 1420

2.788875 9.822001 9·977125 2.623249

2.7783'l3 9.8'13877 9.499963 3.152388

To BC 600.30.. 2.778374

The advantage of these two last methods consists in its being unnecesaary to find the values of the angles C and B to determine Be, and that several of the quantities are found among the tables at the same opening of the book, and if computed both ways they are a check

upon each other. . .

CAJlE III.

50. Given the three sides of a triangle, to find the angles.

In the triangle ABC, there are given AB 800, AC 320, and BC 562; to find the angles. '

Ccmn1'Vction.-Draw the line AB equal to 800 from a scale of equal parts, then from the same scale take an extent

~ual to AC 320, and with the centre A and ~ distance 320 describe an are, in like manner, ~ with the centre B and distance BC 162, intersect. At4----::C----'~B the former arc in C; ABC is the triangle required.

In the solution of this question, if the ~les A or B are first to be determined, then rules II. or IV. § 46, will be found moet convenient and accurate; but if C be wanted first, then if great accuracy is required it would be improper to use rule II., but rule III. or IV. should be employed, 10 &8 to give the anglewidl all the requisite accuracy in nice operations.

Digitized by Google



nl~va.n:ON. By Calculation.

Bv .. B II. '

AD 800

AC 300 ar. co. DC 562 art. co.

8um 1689 Half 84:1

1st difF. 521 log. tel cliff. m log.

Hall 64° l' 54".4 sin. 2

2·716838 2.446604

19.907556 9.958778

Sum

C ]28 3 48 .8 RULB III.

AB 800

AC 320 ar. co. BC 562 ar. co.

Sum 1682
Half 84:1 log.
DifF. 41 log.
Sum
Hall 64° l' 6~'.9 cot.
i 9.924797 1.612784:

19.28269f 9.641847

C 128 3 49 .8

RULB IV.

AD 800 AC 320 BC 562

Sum 1882

Half 84:1 ar. eo. 1st difF. 41 ar. co. td dift: 521 log. 3d clifF. 279 log.

$um

Half 64° l' M".7 tan. 2

10.3)2431

C 128 3 49.4

From these solutions it appears that the first and secend difFer about 1" from each other, while. the aecond and lut only difFer00'8'~4

Digltizell by uU L

PLANt: 'l'l.tOONOMrI'RY.

HIMi tt. .... C been DeIIN!' 180", the finIt and second 8OlutloM might perhapa bave differed more COIleiderably, while the aecond and third woulQ have agreed more nearly. Hence it is clear that the proper raIea, when great nicety is required, must be chosen accordbig to the Datare of the angle.

ELUrPLBI POB EnROllE.

51. 1. What aq),e will one foot lubteDd. at the distance of My

mila? A,...-O".78.

g, The hypotenuae of a right.an,ted triangle 1aeiq M7~ feet, and the acute angle adjacent to the base. 519° DO' 68", w6a are the hue and perpendicular ?

..f,...-The base 4746.064, and the perpendicular, 2723.538.

3. If the bate of a plane triangle be am, and the other two aidee i88 and 192, what i8 the length of the perpendiculAr upon the hue, and the length of the segments of the Laee made by a line bitec:ting the vertical angle?

..fu.-Perp. 139.4274, segments 230.4 and 163.6.

4. There are three town., A, B, C, 10 eituated t.IuIt the bearin_g of B and C from A forme an angle double that of A and C from B, and that of A and B from C double that of A and C m.n B, or the angle opposite b is double of that opposite o, and the circuit round au the three i8 just QlMl hundred mi1ee; what are their relalative distancel from each other in 8ucceeaioD?

A,...-19.8073, 35.6861, and 44.5066 mila

i. In the right-angled triangle right-angled at B, Jiven the bate AB 70, and die sum of the hypotenuse and perpendicular AC ad BC 200, to find the hypotenuse and perpendieul&r, and the remaining angles?

:"u.-The angle ACB is 3'/0 16'. AAC 51° 94', and AC 119.52, and BC 8'1 .•

6. In an oblique-angled triangle ABC let the side BC be 532, the ~le BAC 110" 30', and the sum of the sides AB, AC 637; req1lireel the angles C and B, and the .idea AB and AC?

A,...-The angle C ia 45° 5', B 94° 25', and the side AB4O'J.3 and.

ACJM.7·

7. In the obUque-angled triangle ABC, let the aide DC be 250, the angle BAC 96" 50', also the difference between the sidesAB and AC 106; required the angles ACB and ABC, together with the .des AB and AC'?

AfU.-ACB is 57° 55', ABC 25° 15', and AB 213.4, and AC 107.4- 8. Given the bate 214, the vertical angle 49° 16', and the SUJD of the other two sides 459; to find the sides and remaining angles?

Am.-The acute angle is S3" 44' 48", the obtuse angle is 91° 59' lSI", the side opposite the acute angle is 176.;'5, and the side opposite the obtuee angle is 282.24.5.

9. Given one oftfle sides 252, the opposite angle 20° 46', and $he exceu of the base above the remaining side 86; to find the remaining

aglee and sides. . .

.;Im.-The vertical angle is 94° SiSI' 28", the remaining anale is 55° 51' Si", the base is 507.08, and the other side 421.08.

10. Given the bate 1514, the vertical an,le 75° 24' 50", and the perpeadicular ~.41; required the remainmg aides and angles.

- .du.-The sides are lwa and 1172, and the ang\ee are 56" 4' 5" and 48" 31' 6" respeetively.

Digitized by Google

6i. The vario ... aailiaB. in II&vigMieo ar. CIQly .tke appjieathnt of trigonometry in particular circumstances.

The course is the. angle fORDed· between the meridian and the po.iDton whieh. the ship _s, the distance ia the hypot.eouae, aod the difference of latitude and departure, the legs Of . a .• ~ triangle.

Thus let AB represent the meridian; then if a, , ,

ship sails north-easterly, the line AC is drawn to D B C

the right-hand, making an angle BAC equal to D~---t.::---:7"'c the course. and AC represents the distance. AB. the difference of latitude, and Be the departure. Ii abe saila north~westerly, then'BAD is supposed to be the angle of course shown by the compass, and is generally in points and quarter points, AD the

di.taDce,-AB the difference of latitude and BD ~ , if

the departure. ~in, if the ship sail south easter- :E

)y, AF is the distance, AE the different latitude, EP the departure, and F AE the course. If, however. AE' be the meridian ClifFerence of latitude, E'F' is the difference of longitude, E' AF' is the course, and AF is still the : distance. Hence the course and distance between two ;places can be found, by this method, when their latitudes and longitudes are known. This is commonly called Mercator's sailing.

Parallel, middle latitude, and oblique sailings, may readily be explained on similar principles, though these can only be completely

discussed in regular treatises on navigation. .

See Mackay'S, Norie's, Riddle's, Inman's, or Robertson's Navigation.

ELUlPLES.

1. A ship from latitude 47° 30' N, sailsS. W. by S. 98milesj what latitude is she in, and what departure has she made?

Ans.-DifFerence of lIititude 81.48, departure 54.45 miles, and the latitude come to 4()0 9' N.

~ e . A ship from latitude ~ 3W, N. sails between north and west till her departure is 54 miles, and then finds herself in latitu4e .0 54' N.; what course did she steer, and what distance did she run? ..I Au.--course 32" 22' N. W., and distance 98.18 mile ••

3. Coasting along shore I saw a cape bearing N. E. by N. After standing N. W. 20 miles the same cape bore E.N. E. Required ~ distance of the ship at each station.

An8.-From the first station 33.26, and from the second 35,31 miles.

4. Required the course and distance from Caithness point in Scotland, in latitude 500 46' N. longitude 3° 17' W., to New York in North America, in latitude 41° 5' N. and longitude 74° 15' W.

A",.--Course 68" 32' or W. S. W. nearly, and distance 2899.2 miles.

5. A ship from latitude 6()0 24' N. and longitude 43° W. saila between South and West till she is in latitude 56° 30' N., and has made 226 miles of departure; required her course, distenee, and longitude?

An8.-Course S. E. nearly, distance 325.4 miles. and the longitude of the ship 35° 47' w.

6. Required the course and distance between the IsleCQfMay• in lati.

Digitized by oogle

...... .. II. ~ r 18' w .. ,..ad H .. i" ...... '0"1 .. MI" N. laagitude ". 5S' B ? . . .

A........o..ne 8. 'II" ~' B. ad Dirt. m .....

f. A..., .. thelale.tMayrailed OIl the fDUe1Vu. .... 0CIIIIIWtt.

~_""., .

Go__ 1 DiH.-i>iI:- IAt. I ~ I

N. I s. 1 E. 1 w. 1

40 --·1 28.3 28.3

50 46.2 19.1

20 14.1 14.1

60 49.9 33.3

200 76.5 184.8

15 2.9

20 18.5 76 63.2 60

PLUlB 'l'UQ.ClIIOIIIITBY .

42.2 58.2

14.6

----·1---1

318 95.8 218.4 380.0 . 22 .• 95.8 22.4 -r----

12'J.6 357.6

1-

2" 3'S.

56 12N.

5' 9N.

------- -----

Heaee the ship is about 3 miles south of Helwolalld ¥t.

S.71,E.

Die: ofLat. Lat.left

I

!Lat. in



14.7 7·']

SRCTION V.

,AppIicatimt qf PifJfte Trigortometry to tlee Mercnration qf Heipt, and Distmlce,.

63. One at the moat i~ applications of plane trigonometry is the mensuration of heights and distaueee. TIle lata are 80IDe Of the aide. and angles of a triangle. The aides are measured by rods, lines, tapes, or chains, cossteuceed accordiag to Ute degree of accuracy ..equired ; and the angles are measured by SGme angular instrument, such as the quadrant, sextant, reflecting circle, repeating circle, or tlaeodolitAt. The repeating theodolite is perha,., i. 8eneiotll, the __ - COJlnnient of ell for taking the neceaaary: angles, 8lld the chaitt, properly constructed, the best for measuring the side called the baH, though, to milit.aay mgineers, the small pocket circular box-sextant, or semicircle, as impnwed by Sir Howard Douglas, will be found highl,.::rul, wlwo aecompanied by the box-measuring tape. One of8c calder's 8urvey.ing compasses will also be found ve'100mmOdious in miliwy and nautical surveying. A complete descnption· of tIIeIe inatruments would far exceed our limits, and their use is beat

• Thoee who wish for written descriptions may consult Jones' edition of Adam', o-etrieal and Oraphleal 1i:uays, aJioeady mentioned, Blot', '.l'ratu d_'A-.nmnie l'Itvaiaue, DeJambre's Astronomie, Base du Systeme Metriqve, WOCMIaeue'e, ViDce't, aad Pianon'. Treatises of Astronomy.

Digitized by ({;~ogle

II

~'I<*.

leIImt ... the Miperiutendance of • maatIr. Ia 1lI!IMft},. it ~ be ~ked, that an allowance muet be made for die- hei8bt fIf die eye above the hori.-l p1aae; .. d" when ,the hoM above-IJM!I~ ia inIIiaed te tbe horison, it muet be l'ed1tced to it aGClOIdiDg,U, ~ given inclination, though in nice operations the hue is eelected 10 .. to be, if not exactly, at least nearly level. Then, from a little attention, by dri ring in stakes at moderate distances, and levelling their tops, on which deals properly prepared are laid, an exact horizontal line may be obtained. This truty level line is to be moat carefully measured, allowance being made for the contraction or expansion of the materials of which the chain is composed according to the state of the thermometer; in nice operations reduced to the level of the lea; and such other precautions as the natnre of the case may require , muet be observed, in order to insure the greatest possible accuracy ; lIlIIDy examples of which may be seen in the Trigonometrical Survey of the British islands under the direction of the Board of Ord. nance.- A ·number of the more useful problems connected with trigoaometrical SU1'Veying may be seen in the third volume of Hutton's Course of Mathematics by Dr O. Gregory, in Baron Zach's Work OIl the Attraction of l\'Iountains, in the Base du Systeme de Metrique Declmal"and in Plnseant's Geodesie.

EXAlilPLE I.

To determine the distance of a tower, inaccessible by reason of an intervening river, I measured, on a horizontal plane, the base AB, 500 yards, and at each end took the angle included between the

other end and the tower, which were 50'>56' C

and 760 10' respectively: What is the distance '

of the tower from each end of the base ?

In the annexed figure, AB=500 CAB = 5()<> 56' CBA --: 75° 10', and consequently

Angle C = l00>-(A+B)=53° 54'

Hence, sin. C 580 54 9.907406

Is to AB 500 2.698970

So is sin. A 50° 56' 9.890093

9·907406 9.698970

To BC 480.46

80 Is sin. B 75" 10'

2.681657

9.985280

, To AC 590.2 ., 2.776844' I

The perpendicular or nearest distance C d may, if required, be easily

found thus: :

As radius 10.000m0

Is to AC 5982 2.776844

So is sin; A 5()<> 56' 9.890093

To Cd 464.45 . 2.666937

Remarkl.-These' distances might have been determined without an Uutrument to measure the angles. Thus, suppose that, in the

• Theft are lleYeral methods of ap})roximating to the heights of objects by meaus of mimIII, IbaIlows, staff", geometriw squares, and Gunter'. quadrants I but • ~

are rehhIm ailed where mueli acc:uracy is required, they an: omitted here. '

Digitized by Google

n

PLAlfK~Y.

_tit'II' i of.·.,... AB, andtheline.CA,CII, the,.. ... lEU, AIJ),.AJI,·II11;.BG, Wlelre taken aU equltdOOfeet, .... D.R_~ eel 88, and FG 122 teet, ·the reapeetiY8 ~ te·& rMiu'.t'lCJe feet. of tile exterior aD«leiDAE. i'BG. whieh .. equ)lto.tIMUt"erdeal interior ~1es CAB. CBA. Now, IIinee half the chord is the sine of half i:be:~, we have -Nif=-43=sin.! A=ii" 'SS', and A::DO" 56'. In like manner, 0. "B--61=37° 36', and B=75° W',

which results ..-ee with the former. .' -

Notel.-Tbe number 100 was chosen for the sake of lim~city; ~ any other eonYeDieot numbee ~ be adopted. taking care !o ruvi.iie half tbf measure of the chord by it. . . .: r

Note i..-The same thing may be accomplished when the- siaea ~ the ~s bear any proportion to each other, by findiag_ hom them the angles DAE, FBG. Also the supplements EAB, ABG' of the ~ angles may be found in the same manner, or otherwise

by joimng AS and BE. !

ExurPLli: IL

W8.IlUng to know the breadth of a river, I measured 100 y&l'da in a straight line by the side of it; and at each end of this line I found the angles subtended bl_ the other end, and a tree close by the oppo.o lite side, to be 53° and 79G 12'; what is its perpendicular breadth ?

An.r.-l05.89. . . .

EXAMPLE III.

In order to find the distance between two trees A and B, which could not be Qirectly measured on account of a pool of water which eecupied. the intermediate space. I measured the distance of each frODl a ~d opject C, which were 588 and 672 yards respectively, and then at C took the angle ACB between the two trees 55° 40'. Required their distance.

180" 0' Angle C_ 55 40

A+B 124 00 ! (A+B 62 10

AS Be+AC 1260 3.100371

Is to BC-AC 84 1.924273

80 is tan ... (A+B)6'JO 10' 0" 10.277379

To tan. .. (A-B) 7 11 53

v

c

Angle A 69 2] 53 Angle B . . 54 58 7 ~ &in. A .CK)O ~l' 53"

Is to BC 6'r.l

80 is sin. C 65 40 0

9.971903. 2.821369 9.91685"

To AB 592.96 2.773025·

• EXAIIIPI,E IV.

In the trigonornetri~ survey of Britain, Colonel Mudge found, m.n ~utations dependinf{ on former operations, that the logarithm of the Dumber expressmg the distance between Cheviot and Cross FeU in feet was 5.4654017, .and betweeuCheviot and Wisp Hill .D.~78,· and the: angle contained by these, corrected for

•• 1 ••• .·r ,

• In lODIe of the examples the computations in proportion are pedbrmtd by COIDp8riDs the rifIu of the angles with the-rides, a method somedmee more eaBY to beginners.

Digitized by Google

: IIW ... ., was 6S'" 8W 18/1. RaqaiNd the CJdmo ... -. ... eli....,. lMneea Wilp Hill and eroe. Fell, without fir. In_

.. __ .the siva Bidet in Datural D1IbI.bera. ,

Au_TAe angle at Wisp Hill ia W'/0 14' 4". .

Cross Fell 39 15 (6

The mdllDce of Wisp Hill-nom Cross Fell 235018.6 feet.

EXA.PLE V.

In order to determine the height of a tower, I mea..; C

81sred in a direct line AB 366 feet on a horizontal LJ plane. I then took the angle Cab 87' 30', the height

AD of my instrument being 5 feet. Required BC the

~eigbt of the tower. .

.dnl. sc =~.84.

. Aad Aa 5.00. A..

Height BC =285.84.

EXAlIlPLE VI.

W alltiD.g alOI)~ the side of a river, I observed an obelisk on the op. posite side, which on account of the river was inaccessible, but. whose height I wanted to ascertain. For this pur-

p.' ose t. took at B t. he angle CBD 50° 39' at A the angle ~c CAB 33030', which was distant from B 368 feet.

Required the height of the obelisk and the dis-

tance of the station D from its base. .

SohJtion. -Because the angle CBD=CAB + ACB, CBD~CAB=ACB=50" 39'-33" 30'=17° 9', hence

siD. C: AB: : sin. A: BC; and in the right. angled triaDKle DBC are now given BC and the angle CBD, to find DC and BD, 521 and 427.2 feet respectively.

ELUlPLB VIL

A solution ofthia problem, more easy and commodious in practice, may be obtained thus :-

Let. CD represent any object whose height is to be detennined; at the points A and B observe the angles of elevatiea, and measure the distanee AB, the point. A,B.C, and D being in the same plane.

See preceding figure. .

. For in the tr~les ABC, CBD, am. ACB : AB: : sm. A: BC,

and R: BC:: sin. CBD: CD, from which we have sin. ACB: AB ~ Be : : Bin. A x sin. CBD : BC X CD or sm. ACB X BC X CD.= sin. A X sin. CBD X AB X Be; radius being unity.

H CD Bin. A X sin. eRD X AB __ to· th

ence sin, ACB, ; or, lWIAIDg et.ermsbomo-

geneoUI, and substituting cosec, for --.1_, ~

SlD.

RB X CD = sin. A X sin. CBD X cosec. ACB X AB.

That is, t<l the sines of the observed angles of elevation, add the cosecant or the difference of these angles, and the logarithm of the measured distance; the sum, rejecting 30 from the index, will be ~ height of the object.

Let the angles of elevation be 56- 54', and 33° 20' r~vet.J" and the distance between the stations 100 feet. Required the height C1f &he object.

Digitized by Google

PLAD TIUIfiIOMDIOTRY

{WI,"" .me. I,).IUIOEJIJ

Angles ~f el~vation 33 20 sine 9.7399'15

Difference Distance

Height .

Height of the eye

Height of object.

22 34 cosec. 10.415942

100 feet t.OOOOOO

]18.5 5.5

124.0 feet.· EXAMPLE VIII.



InDrder todetermine the distance of two inaceetsible objects lying in a direct line from the bottom of a tower 00 feet ~b, on the top .which I Wok the aaglea of depression of the two objects; that Of dae moa remote being 24,0 48', and that of the nearelt 58° S6'. . Required their distance.from the tower, and from each other.

A,.8.-139.842 feet.

EXAMPLE IX.

Wanting to know the distance between two boats lyin~ at anchor in a Itr~ht line from a light-house, which is 110 feet hlgh,oJi the top of whIch I took the angle of depression of the farthest, and found ia fA) belS" 26', and that of the nearest 56° 44'. What was their distance i

An.r.-I29.5286 feet.

EXAMPLB X.

. From the top of a hill I observed two mile-atones on a boriaoatal ro.d, which ran straight from its bottom, and took. their respective· augles of deprelaion below the horizontal plane paseing thNugh the place of my eye; that of the nearer miie-Btone was 36° ur,. MUI that of the more distant 15° 9&. . Requind the height of the hill.

"'.-780.17 yards.

EXAMPLB XI.

ID order to find the height of an obelisk standing on the top of a ~ly sloping hill, I lIleasUred from its bottom a diatanee of 40 ~ and then found the an~le formed by the inclined plane, anel a line from the top of the obelisk to centre oftbe instrument, to be' 41 0; mel, after measuring downward in the same direction 00 feet farther, the angle formed as before was only 23° 45'. Wlaat was the heightol' 6e obelisk and the angle of the inClined plane with the horizon?

... -Height 67.628 feet. Inclination flO 541. '. ,

EJUID'LB xn .

. Wishing to know the height of a tower standing on the top of a regularly sloping hill, to the bottom of which I could not approach on account of a ditch around it, at the outside of which I took the an,le formed by the inclined plane, and a line from the centre of the 10" strum~t to the tOJ? of tlie obelisk, and found 'it 41 j but after meamnnt downward m the same sloping direction 54 feet farther. I founa the angle formed in like manner to be 23<> 45'. What was the height of the obelisk itself, and that of its top above the last place of observation, 8upj)OSin~ the angle formed by the inclined plane and the'1Iorizon to be 21- 54}?

A,.8.-51.86 feet the height of the obelisk, and 83.51 above the last place of observation.

I

l

Digitized by Google

UlTAOJilUCJ'!'IQN. EXAllPLB XIII.

Being on a horizontal plane, and wanting to know the height of' a tower on the top of an inaccessible hill, I took the angle of elevation of the top of the hill 40", and of the top of the tower 51° ithen measuring in a direct line 100 feet farther from' the hill, I took in the same vertical plane the angle of elevation of the tower 33° 45. Required the heIght of the tower?

.4111.-46.666 feet.

EUIlPLK XIV.

In order to know the height of a castle ...... ding OD a hill, I took the angle of elevation of the top of the castle above the horizontal plane 500, and of the top of the hill 25° i but could not, as in last 8umpie, measure a aufBeient cliatance direetly from tile CMt1e.'. I thentore measured in an oblique direction 52. yards, makiIIg with the castle an aagle of7il" 10', at the farther endof which the angle, in the same JD&IlDeJ', was 04° 80'. What w." the height of the castle?

.4,."...:.a4.464 feet.

EXAIlPLE XV.

Wanting to ascertain the height of a tower standing upon a hill, the height of the hill, and the honzontal distance from the nearest .plaee of observation, on account of the nature of the ground I proceeded as follows :-

At A I took the angle GCK 3" 38', and GeE 2" 44'; then having set up a staff AC equal in height to the centre of the theodpJite, I measured 1810 feet up the sloping groundABinadirectlinewith the tower, keeping thepoints

K, E, C, B, in the same ver- B .

tical plane. At B I took the G

angle FDC = BAI = 10 54', I.A U; ,

and EDF=l° 32'. Required the height of the tower, the ~ht of' the hill, and the horizontal distance from the first place ·of ob .. n ... · tlon.

1. In the triangle DCE, are given the side DC::z:1810 feet, ~ angle ECD 175° 22', EDC 3° 2&, and DEC 1° 12'; to find CE=D175.89 feet.·

2. In the triangle CKE, the angle K=86° 22', CEK=9'J° 44',.

KCE-OO 54' and CE=5175.89 ; hence EK=81.463 feet. ..

3. In thetriangleCGE, the angle GCE=2° 44', and CE=5175.89;. hence CG=AH=5170 feet; and GE=246.826.

4. In the triangle ABI, AB=I810, the angle BAI=l ° 54'; hence AI=l809 feet, and BI=60.011 feet.

If EK, the height of the tower, were only wanted, it may be found

thus: I

• In calculations where the IIIIme number is used which has been found from ~iOUJ computatiOD, itl log. mould be reserved from the firBt to be used in the next, &c.

Digitized by Google

II

sm..DBC : DC: : ain. CDE : CEczDC sin. CDE. eoeee, DBC. aiD. K: CE (=DC. sin. CDE. cosec. DEC):: sin. KCE: KE, and

RfKE=DC. sin,. CDE. sin. KCE, sec. GCK. coaec. DEC. -

By logariduns.

sin. CDE 30 ~ •

sin. KCE 0" 54'

sec. GCK 3038' cosec. DEC 1° 12'

log. DC 1810

8.777333 8.196102 10·000874 14.67fm3 3.257679

EK 81.463

1.910961

EXAJlPLE XVI.

At the top of a cutle which stood a hill near tae seB-shme, the 8Jl8'le of depression of a ship'. hall at anchor was 4° 52'; at the bottom of the c&stle the angle of depression was 4° 2'. Required the horizontal distance of the vessel, and the height of the snr on which the eastle stands above the level of the sea, the castle itself being 64 feet high.

Afl6.-4373.75, and 308.4 feet respectively. ..

EXAMPLE XVII.

From a window in the lower part of a house, nearly on a level with the bottom of a steeple. I took the angle of elevation of the top of the steeple 4{)0; and from another window 18 feet directly above the former, the same angle of elevation was 37° 30'. Required the

height and distance of the steeple. .

An.r.-210.44, and 250.79 feet respectively.

EuxPLE XVIII.

Suppose A and C to be two sta- 0

tions on sloping ground. 0 an ob- ~

ject on the top of a hill, and the

angles OCA, OAC, measured with ~

a sextant, to be 79° 29' and 63° II' ~ c

respectively; also let the angle of A p

elevation of AO above the horison- .

tal plane be 6D 3&, and that of CO 5° 22'; what are the horizontal distances and height of the object, AC being 410 yards?

In the triangle AOC are given all the angles, and the side AO ; to· find AO and CO. Again, in the triangle AGO right-angled at G, are given the angle OAG and the side AO; to find AG=660.8 and 00=76.4. Lastly, in the triangle COB, right-angled at B, are known CO and the angle OCB; to find CB 600.7, and OB S6A .. and 00- OH=76.4--56.4=20 yards nearly =HG=CP, the differel1Ce ot'the heights of the stations, supposing AP to be horizontal. Nmv m.the right-angled triangle APO are given AC and OP, to lind AP:;:.

{(AC+CP) (AC-CP)}, =.J430x390:;:. iI67700'=409J1 yards. Hence the sides of the horizontal triangle APG are glven. to find tile angles, which may be determined by Case III, Plane Trigonometry, to

be AGP-37° 31' 29", GAP=63° 19' and GPA=79° 9' 91" .

The present may serve as an example of reducing hypotenusal lines to their horizontal measure, and of determining the height of an object above each place of observation in most common cases.

EXAMPLE XIX.

The height of the mountain called the Peak of TenerifFe was found, 7

Digitized by Google



baromMrioalloJ, by the lIIIIthods described in Gre,py" Meehlaic., Vol. I. book~, to be 12,356&et,ori.34 EngIiah miles, anel the.-gle of depresaiolr of dte- hmiHll, u.,. thIe meaD of a' great number of observations, 10 58' 12"; it is required to determine the diameter of the earth, sup~, it to be a perfect spheee.

Aru.-791S:6 raiIeI. .

Let C be the'Cefttre of the earth, the circle BTG a vertical section pt!Aing through the centre, AB the height of the Peak, AT the tangential line drawn from its top to the visible horizon, and AD a line perpendicular to a plumb-line hangblg freely: also, let BE, a tangent to the earth's surface at B. meet the other tangent AT in E. Then, in the triang'e ABE, right-angled at B, there are given BAg the complement of DAT, the angle of depNS-

aion::::::Blr I' 4&', and AB=2.34, hence R : AB: : tan. G. ,

A : BE :: sec. A : AE. But since the triangles .• . ;

ODE, CTE, are right-angled at B and T, have the side CB=C'I:, and CE common, they are (Leslie's Geom. I, 22, or Hu);ton's Geom. theo.34, cor. 2) equal, and therefore BE=ET; hence. AE+B~ =-:AE+ ET=AT. In the tria?gle ATC, right-angled at T, .we.have R: AT:: tan. A : TC, the radws of the earth. The' operatlOll .tJiU1, perfonned occupies but small compass, which may fltj)l be £tI'" ".rtened. Forsincetan.A+sec. A=tan. (A+!comp.~) wel'b .. ' . ~ ~ratiIJg the proportions from which At, BE, and CT a _ deduced, ~! .::. 1 "J ,·i i x .~,(jS:,~ ,

, R9 CT:=:AB tan.(A+!comp. A} tan. A ....

Of, kg. CT=log.AB+log. tan. (A+tcollij). A)+log. tan. 4~.f!

the index. : ~

: 'J'be loguithmie computation is ~ follflVs~- • f\:'~. )4 J-!:'

Depression ) 0 58' 12"

. , . r i. : .! br:p.

<: "'\.t J

Half {59 6

Comp. depress. 88 146 tan.

Sum 89 0 54 tan. Height of Peak 2.34 miles, log.

Earth's semid. 3956.8

2

11.7646438 '0.3692159 .;.,

3.5973«7 . ,

• s , ~ • .. ."1

Diameter 7913.6 " . • .v ,

Distance 136.1 " 2.1338595' 0-

If AT were required, we have only to take radius (10) from the sum of the two last lines, and the remainder, 2.1338595, is the log. of 136.1, the distance sought.·

Note I.-This method of determining the earth's radius, though elegant in theory, is useless in practice, at least where any thing more than an approximation is wanted, by the great irregularity of the horizontal refractions.

Note 2.-When the diameter of the earth is known, and height of the object given, the distance of the visible horizon may be easily fOund; for, Euc. In. 36. AB.AG-ATII.

• See Dr O. GTegory's TrigonometTy.

Digitized by Google

pLAn TJUQOWOJfETBY.

Bt1Ggaritbin8t

AD i.M 14w.

00· 79i3.6



O.3.U~

AB+OO:AG 7915.94 log.

As before 100.1 miles, log.

Nole S.-The depression of the horizon, or the dip, u It is called It Ilea; is the angle DAT contained between the title aim. visiOIe horizon. For if an observer, whose eye is situated at Aont1'1e deck of a Teael, takes the altitude of a celestial objeot with HadleY'B quadrant or sextant, by bringing that object to the surface of the: water at '1'; instead of the true horison AD, tile altitude is evi. derrtly tdo great by the angle DAT=TCA. This mlly be ce'lculll.~d ., the U8Ual tormula! or trigonometry (or that pu'l'pose; bUt !lS it WiU, at any probable' altitude, be a lIlllall qilantitr, those which give the eosine or secant or its value are not suffiCiently co'irect; RIr ..tdeh reason we shall give the {oRowing method:-

(lKl+AB)xAII-ATIf, (Euc. III. 38.)., hence BGxAD+ASIr_AT., or IDC X AB+ABIt=ATII, and AT· being, at any probable' ete ..... don. but a small quantity in comparisoB of AC, it ma)' be safely .. glected i therefore J(2BC X AB)::A.T. But CT(:;..; BC} : R :: AT

[J(tBCXAB)]: tan. C=tan. DAT RJ(WcC.AB)=~R~B •

... . -1 ilR". . dBCbe' k'

~aw !pnce 'BC IS a constant quantity, an mg ta en 10 geDe.io

al at 9S66 mil.,. =i0887889 feet, hence the log. of :" isl~98ll4. IIIld tan. DAT !(I~.98114+1og. AD). Since, in the present case, .. arc may_be substituted for its tangent,. the radius, therefore, beCltDlee fIT' 11' 44".8=200064".8; and"e have log. DAT in second. :1£S.66999 + lbs. AB in feet}.

. ~ dJp is afFe8&ed by terrestrial refraction:. which is very v~e,

rmt )ydi6rent authors it is estimated at difFerent quantitie& Dr ' MaAelyne.8lltimated it at one-tenth of the whole; M. Delambre, oneeleventh, ~ Col. Mudge, one-twelfth. See Dr Hutton's ~,

vol. In. page 138.

b.-Required the dip, the height of the eye being 40 teet, and -.,mnlling the terrestrial refraction at t~.

Constant log. S.60999

Height of eye 40 teet 1.60206 .

5.21005

403".6 log. ~ sub, -h S3 .6

Dipt

3'10=6' 10".

• See aIeo the metW by Lealie in·hIa a-etry .

... Tlle4i.p in JaiDu ... ilequal at da 1Iq_1QOt ohhe IIe.IPt in feet .-df.

Digitized by ((P.1ogle

Note 4.--8ince AB X BG+ABII=ATe, therefore . AT"

AB(AG+AB)=ATII, andAB=BG AB' (1.)

+ .

Now, if AD is the unknown quantity, and being small in comparison

, '...I. ATII

of BG, it may be fo~nd approximately by making, first, AlJ' = BG

nearly, substituting this value of AD' for AB in formula (1.), and

AT II : .

AB=DG+AB' . . . . . • . . (2.)

which will be sufficiently correct for most purposes, If not, ~ operation may be repeated till it is so.

This is useful in determining the height of an object conside~ably distant.

· Now, the mean diameter of the earth is about 7912 miles, or · 1775360 feet =GB, of.which the logarithm is 7.620920, and its arithmetical complement is 2.379080; therefore to twice the log. of AT, in feet add the constant log. 2.379080, the sum, rejectiDf. tensin the index, will give AB', which will be sufficiently correct If AT does not exceed 1000 feet, If more distant, .the operation m~ be rep!&ted. This correction must always be added to heights de~ed geometrically as the usual instruments give their eleva:. ti.oD ouly above the tangent AT.

EXAMPLE XX.

· Given the angles of elevation of any distant object, taken at three places in a horizontal straight line, which does not pass through the pqint directly below the obJect; and the respective distances between the stations: to find the height of the object, and its distance from

either station. , ..

, Let AEC be the horizontal plane; FE the perpendicular height or tb.e object F above that plane; A,. B, C, the three places -ef obsena.yon; FAE.1 FBE, FCE, the respective . Ji JIlgles of elevation, and AB, BC, the given distances. Then, since the .tri-

angles AEF, DEF. CEF, are all right. :M: ~~:::2:§;;;r=F7

ugled at F, the distances AE, BE, CE. .A:

will manifestly be as the cotangents of the ~les of elevation at A, B, and C; and we must determine the pointE, 80 that these Iines may have that ratio.

· Construction. .

To effect this geometrically, we must take BM, or AC produced; equal to BC, BN equal to AB.; 8I\d make

MG : BM (.=BC) :: cot. A: cot. B, and

. BN (=~) : NG : : cot. B: cot. C.

With the lines MN, MG, NG, construct the triangle MNG; and join BG. Draw AE so, that the angle EAB may be equal to MGB; this line will meet BG produced in E, the point in the horizOlll4l plane falling perpendicularly under F.

...... Demon.ttration.

By the similar triangles AEB, GMB, we have

. AE : BE : : MG: MB : : cot. A : cot. B, and BE : BA ( BN): ~ BM : BG.

Therefore the triangles BEC, BGN 1lre similar; consequently

Digitized byGoogl~

. PL.A.NE ~Y.

BE : EC :: BN : NG : : (lOt. B : cot. C. Whente it is obviou8 the AE, BE, CE, are respectively as cOt. A, cot. B, cot. C.

, Calculation.

In the triangle MGN are given all the sides, to find the GMN, equal to the angle AEB. Then, in the triangle MGB, are given two aides, and the contained angle; to find the angle MGB, equal to the angle EAB. Hence, in the triangle AEB are known the side AB, and all the angles; to find AE and BE. And then EF=AE :tan. A=BE . tan. B.

Anal!Jtically.

Let AB=r, BC=s; also let the cotangents of the angles F AE, FBE, FCE, ~e denoted by the letters a, h{ c, respectively •.

Then, putting EF=x, we have, to radius 1, 1: a : : x : az=AE, 1 :.h;: ~ : b=BE, 1 : c : : z : c z=CE; and on AC from E, letting fall the perpendicular ED, we have (Euc. II. 12) ai z'l=bi Zll+ all z'l-b'l z'l_rfl

r·+ 2 r • BD; hence BD 2, . In like manner CD=

bga:£~Slzg_8 BD-BC=BD-.r: whenceBD b'lZfl~'1Z.+88.

, . S

hQ Zll-c'l ~Q + IJ$I ali 1118 _ hI! Xl - r'l

Therefore 21 = 2r . Hence.al";:;

r,lI+r,1I d _j rs(r+s)

,(all-hIl)-1'(h"-cQ)' an z..... ,(ali_hll)-r(b!l_cli)"

I Otherwise thus; ~

If AB and CB be conceived to be bisected in M' and N', and ED & perpendicular uP.OD AC, which are however omitted to avoid complexity in the figure; then, (Leslie's Geometry, II, 21.) AEQ-BESI =AB X ~M'D, and CEfl_BE!l=BC X 2N'D; therefure, AEIf X Be -BEll' X BC:::AB X BC+2M'D, and CEflXAB-BE!lXAB=AIJ xBCX2N']}. Adding equals to equals,andAESI xBC+CE!I XAB -AC)(BE!I=ABxBCxAC; consequentlyAEflxBC+CESI xAB

=AC )('BE8 X AC x AB X BC. •

JlAB=BC. then AEfl+ CEII=2AB"+2BEII, the line EB being drawn from the vertex E of the triangle ACE, to anr point B in the baM. Put AB=D, BC::d, EF=x, and then expressmg algebraically

the fOl'egoiDfJ theorem. . .

; The equation thence resulting is,

dzll cot. IIA+Dz!l cot. flC=(D+d) XII cot. IIB+(D+d) Dd.

HMUle~ transposing all the unknown terms to one side of the equagon. dividing by the sum of the coefficients, and extracting the

j (D+d)Dd

square root .. we shall ha~e z= d eot, II A + D cot. iC-(D + d)cot. tJB"

Thus EF becoming known, the distances AE, BE, CE, are found by multiplying the cotangents of A, B, and C, respectively, by EF.·

Cor.-When D=~ or D+d=2D=2d, the expression becomes

. x=d+.v(~ cot.IIA+I cot. QC---cot. liB), which is pretty wen suited to logarithmic computation. The rule may, in that case, b. thus expressed_Double the logarithm cotangents of the angles of elevation -of the extreme stations, find the natural numbers Answering thereto, and.take half their 81Ull; from which subtract the llatul'lll DUlIlber answering to twice the logarithm contangent of the middle angle of elevation: then half the log. of this remainder subtracted

Digitized by Google

"

..

ilaDI* 118. P£ tlIt ......... .u...a. heel ... the fi ... Und ......... " or the ..eond ami. UUt4;IIaIiop» wID be -,bw. of cite belpt,'fIf ..

,~tot AB=OO f~. BC 7~· teet; aD_$l41 FCE:t:5O'I . 28', .. angle" FBE=4O" 83', and tlie aogle F AE:::3O" ,W; reqUired die ~CC!!I AE, BE, eE, aDd EF. the height of the object.' ", !

)1&I.-AE=1S9.09 feet, BE=llO.84 feet, CE=78.51 teet, ·Mad

EP .... 94.84 feet. '

.9. Let the three angles of elevation be S6" 50', f,u° 241, 'and 140, and the two equal measured distances 84 feet; required the height

9("'e Qbject. ' ..4.........oa,964 feet. ;

", I .



OI.veD the angles of eientio.D at which an ob,kct .8 ljeen"'_ ~ w.~eh ~oint.B in a no:dzolltal plane; to find its position and altitudcto !

;Let A .. B. and C be-the three, _ :

PQints of' observation, and D F :Q , I. ,~,

~e b"ttom of the perpendicu; lilt' {,,·om the. given object ttl dUl h!,"rifontal pl~ne. ]t Is evi~ dent tha>t tbe "horizontal di~ tan~!ill !tD. ~D, and CD are pr0I!0l'tion~ to the cptangents or the vertical angles at ~6 stations A .. n, C~.; let these CO~ tan.gents 'b~ respectively de., PQt~ ~Y tpe L, M, and N. •

DiVIde A..B internally and ;J; e¥;~. ,"ally at; the pOlnts Band F ltl the ,ratiO of L, to ¥ ; and. the lines,DE and.DF }lining jntbe verte~ D mqst bhect in·tema11y and externally the angle •. ,,,lienee EDli' is a right

angle, anti contained in a se~ .'

micifcle;whereforeonEFde... . : ,A'·: r.: C

epribe a 8enUd~cle. I~ the .. same maDDer, dtvJde CR iDtenlally aitd llXterDaliyat G and H" in the ratio of M tJo N and 011 .8 ciesedMa seinicircle. The point D commoll to beth ~clet ... I8eeur

in the'ir inte~sectioD;. ., ' . ,

From this construction the trigonometrical ee1culatienis· ...my: deduced. Fo~ L+M : M : : AB: BE and J.,..:_M: M:: AB: BF;-

BE+BF EF .

~_ce l)E = --2-:;::;; T' 01' redtu8 KE is found. In like

In'~,rlet N + ~I : M : ;' CB : BG, an~ N_.M; M: : ·ca'; aH,.'~' . ,'""'. BG+BH .•. • . " ' ,qaePIlY DJ = _. -2-. In tl e triangle IBK, the 8Jd.e~ ",l$~ ,~d

BK, with. dIeia!. htcl1ldeci-angle, = ABC, are giYen.; and, ~ ..

,I

. ," .

• See Lealia', C2e0meRy, fourth edition, JNI8II '.116. To avoid extencliBg the fi«aJe loG 1Pp,c'" the PO~.l!l .bi.ck IIhoIIJd lie in eontinualiloa of Bl,m ~_ ",. BI!'. iD eontinuation of ]J1l, as 1I1eIhs the ~ee jj)ininB DE aud DY, Je omitt.ec1. .

Digitized by Google

PLA.R.~RY •

... ..p HI and the bIiIIt. IJ( .... foi:nld;; A~ III dfe iidee' fit .~"IDK being ~, the-lID. JKD i .. fbund. Hence, ~ the triangle BDK the wbole angle BIrD and its containing sides.are. ~en;.and, therefore, the hue BD, or the horizontal distanee &em

~ Mati_ B, and consequently ita altitude, Is determined:' ,

It is obviou, that the opposite aemicil'cleB will likewise, by their ~0Il, give, on the other aide, a seceud position D' tor ~t point. In practice, however, this ambiguity could be easily remOTa ' It may be remarked mo, that the point D may fall eitIier with-

J~_ without the triangle. '

J.f. dIeJ. Objec$ be seen at the same elevation ftoom .11 tJH, tllree points. the area of the circlet will evidently become tangents, whieb biaect at right angles the aides of the triangle ABC. The projection 'DIGf'-the object on t1ae horizontal plane, will then be the centre of ~re1e eireum.scribing that triangle; and, therefore, the redius or

distance AD may be found by prop. 18, book VI. Lealie's Geometry,

.. shown in the notes, page 347. '

, If the three points.of observation should lie in the slime atl"aight liae, the centres of the determining circles will occur in that line or ita extenaion: and hence the process of calculation will be greatI,. abridged, and will coincide with the foregoing Y.roposition.

Uatn,u.-Let the angle of'elevation of the object at A,' be 5()'1 45'. tbat at :B ~ 15', mel that at C W 45'; also the aide AD 24 yard~

AC 38, and Be 50. Required ita height? .

. Hence L= cot. 50" 45', M = cot. 500 15', and N = cot. 46" 45'. l'raal the given aid.,. the angle ACB = 9/10 35' 10", ABC = 470 9' "', and HAC 105° 15' 28". Also L = 0.8170343, M= 0.6188188, N=o.~~_I) therefo~ BE = 10.343, .and BE -. 74,928~ ~bence KE --=d.~, and BK _32.2925. In hke manner, BG "",19.846, 8H = ~~. hence DI= 57.9845, and 1B = 38.1385. From these the angle'lKB-~ T/" 11' 24:". and KIB 55° 39' .. 1411 j and the side IK -:-23.677. __Now t1-em the three sides ID,IK, and ED. the angle IKD :::;.J07'" 10' ~;'. To this, by applying the angle IK.B by addit.ien and subtraetlon we obtain the angle BKD'= 1M." 21' 50",

..ad. ~ =-AJ{D = 290 59' ~'. .

) From the sides BK and KD, and the contained angle. B'KD. at libu~d ,tile' _~le KBD = IO'J'> 16' 39", and KDB = 47~ 44' lUi', '.tinm wbicll BD = 21.8065, arid the height of theobje~ 35.2~ yl.l.rd.S'~ t- .8boGld the ~Dt D' be the foot of the perpendicular e, tIi~ angle.

KBD' = 20 29 and KD'B = 10 52' 50". and BD' = 74.876, whence

·(tIe height above D' will be 121 yards. .

'JCf , • •

, .,.' EullKB XXIL '

;.~.. • • ,I', 0tbennae thus:

GiTen the anales of elevation of the object from three poi~tll;in - ~ fIUIJe p~e forJOing a'~angle, of which tb~ aides are known, CO

I' fiDd. the position of the obJect referred perpendicularly to that plM1e -ll6crits altitude above it. . . " '

. "),.,~(iQn..-The perpepd.icW.ar from the object to the plltile

O;-, J', .

, "HI'

Digitized by Google

38

may fall either widrln or! .• ftIIoat. tri- E ' ,~

~It:. ,.m, bo~ CNA 1_ A. B. aQd C tie the poirik of observatipn, apd.. ~, and or the

~les of elevation at these points respective- .. ', r

ly. Join A. B. and C. and on AB .produced. if neces~'y .. make AE equal to AC, and AD to An. JOlQ ED, and upon it construct the

triangle EDF 80 that cotangent ~ : eontan- G

gent ~ :: AE : EF, and cotangent ~: co-

tangent" :: AD : DF. Join AF, and from A ))

B draw JIG; making the angle ABG equal to the angle ATE, and join CG. The point G in which the straight lines BG and A'F attersect each other will be the point at which a perpendicular Ieit' fall from the object would meet the plane, thus ascertain~ itf1W position of the object, from which. and the giTen angles, its alti-' tude may be found. -»,

, Demo1lstralion.-It is obvious that the etraigbt lines dniW'll'1ronr, each of the point,s of observation 'to· the point at which .. perpenc!i-

eular let faU from the obje<.'t meets the plane. ought to be in proporJI tion to the cotangents of the angles of elevation at' these pOJnts reo; spectively. The preposition therefore resolves itself into tbis. ' 'l'Ofind a point in. a plane from which straight lin6 drawn tcJ tb!oee given points in the Brune plane 8hall ha~ to each other a given ratio which follows from the construction just given. .' .

S.oMioll.-In the triangles ABG, AFE, the angles at B and F are equal by construction. and the an~lea BAG is common to botb:i these tWQ triangles are therefore similar. And AG : BG i : A£ ; ~F\

AC ot. ~ .. , .'.' oJ

:: cot. "': cot, fj. Hence EF= xC Again'AG ·:,AE-: t!

. col..·, .. ',:,)

AB : AF or AG : Ac: : AD : AF; and as the ~g1eat'.4. is co." man 0 the two triangles AGe, and ADF; these trialagles.~J conaequently AG : CG : : AD : FD :: cot •• : cat ". w¥nce Fl)::

AB X (!at: ;,. .

--~--~ . . '

. cot.'.

The triangles ADE. ABC having the sides AD, AE of the ene' equal tc? the sides. A.B. AC of the other, and the aagle at A., cqmmon to both; are equal, and the side ED is equal to the side Be:, Therefore in the triangle ADE, the three sides are given, and those of the' triangle FDE are already found; whence the an~les AED and JPED, aud eonsequently the angle AEF may be obtained ; and fl:om. the lingle AEF, with the sides AE snd EF~ the angle AFE or ABG, which is equal to it, may be determined. Then in the triADfIle ABG. having the two angIe. at. A and B. and the side AB the distance, BG may be found, consequently, with it and the &ll81e ~., the height of the object becomes known. . : ~ample.-Let the side AB be 80 feet. BC=119, and AC=140, &tao the angle at A or .=500. that at B or ~-60", at C or oy=:66° • - reguired the height of the object.

, FrOm these EF'=96.329, DF=66.758; the angle 'AED=34° 4&. EDA=8T 6' 23". EAD=8'r 6' 23", EAD=58" 5' 37", GEF=34° 6' 57", AFE. or ABG=700 37' 8", F AE or AGB=4O" ~ 16", BG =55.673; and the height 96.392 feet.

Digitized by Google

PI.ANB ~EtIlY.



" . .8E.uIPr.-. DIlL : 'I : From. conTenient station P, there tould IJe seen tlJree olijects Ii I B, and C, whose distance from each other were' AB=8 nlfiei, AC~ :miles. DC:::. miles; I took the horizontal angles APC=SSO 4,5',

BPC=2'JO 30'. It i. hence' reqUired to determine the respeeeive . distances of my station from eacli object.' Here it will be necewry, .'illustratlve and preparatory to the computation, to describe the

manner of . '

Construction.

, J;>ra". the given triangle ABC trom any cmvenieDt Balle. From.

;tbe paint A draw a line AD to make with AS . C

.JUl mgle .equal to 22° 30', and from B a line BD t.?;make an angle BDA equal to 83° 40'. Let a ckcle be described to paM throUJh their .iDtu- A section D, and through the pomts A and B. tr"---t'-----"J.B

Through C and D draw a Ifr&ight line to meet ~, c:iJ;cle again in ,P, which is the .poiDt required. For drawing P A, PB, the angle APD

is evidently equal to ABD, since it atands on P ._~ .....

til. same arc AD; and, for a like reuon, BPD =BAD. 80 that P u the point where the angles have the auigaecl val... . ... ~ .

, . Computation. .

, In the triangle ABC, all.the sides, are given; to find the ~gles. In the triangle ABD, all the angles are known, and the '1id ~B; to ~d o~ of the other sides AD. Take BAD from lJAC, the remainder, DAC is the angle included between two known sides AD, AC; uom which the angles ADC and ACD may be found. Tbe angle CAP = I8O"-(APC + ACD). Also, Bep =ECA-ACD; and .PJJC=ABC+PBA=ABC+ sup. ADe. Hence, the three required distances are found by these proportions.

Ai sin. APe: AC :: PAC: PC, and:: sin: PCA : PA;' and. lastly, as sin. BPC : BC : : sin. BCP : BP. The operatioo at lmsth

is as under: !,

By BuleILI Case III., we have

Sin. • BAC = J! X : =J iu-!=·25=sin. 14° 28' Sgtl, ~d .

X ..

- 'BAC = 28° 57' 18". . ,'... ,'; I " •... :

jIX5 .

Sin. • ~C = 8 X 4 . lJlO~'39D2847 _un. 9(111 17' l" ••.. ~ •

. , ABC =4& 84' 3". . . " , ., , .. ;

Sin. • ACB = A~:=Jt iJI0-"7~sbt. 5~ 14' 19"1, ~

ACB = 104° 28' 39".

D.4B= ~ 30' CAB=28" 57' 18" DBA 33 45DAB=22 30 0

!fIO° 0' 0" . .J)AC= 6 2118

Sum 5615 DAC= 6 'J/1 18 . ADC+ACD:::;:17332 42

180 0 ~(ADC+ACD)= 86,46.21

-

ADB" 123 45

Digitized by Google

Uft"aODVC'.rIOft.

AD log. •

AC 6 miles, lor. + 10

Arc '. 48" 18' 7" tan.

Subtract 45 0 0

. ReaWncler 3 18 7 tan.

HADC+ACD)=86 46 SlI tan.

i(ADB-ACD) 45 00 17 tan.

O.oeoI686

t=

O·7~98a6 10.1181513

10.OSOl687

Allin. ADB Itso 45' ar. co.

,. to AD . 8 mile.

80 is tin. ABD sso. 45'

8;1611283 U.Ht7IHV/

lO.OOWJIO

ACD::;:; 41 'J"

ACD· 410 7' 4" sin. 9~819678 APC 38 46 () ar .. cOlin. 0.Sl56i610

Sum' 9" ii "

180 0 (}

PAC 105 'J 56 sin. AC 6 miles log.

CPA 7.10199 DlJ1es

0.'178]513

9.98t67.te .0·7781613

0.8513801

PC 1().G525 miles •

ACB=l04° 20' 39"

ACD= 4J 1 4 BCP+BPC=

BCP= 63 21. 35 PBC=

AI sin. BPC 22° SO' 0" ar. co.

II to DC .. miles . •

80 is sin. BCP 63" 21' 35"

l.ol~ 1000 (f ()H 85 51 35'

94825 0.4171603

0.0026600

9.9512694

To PB~~ miles 0.670.197 .

The computation of problems of this kind, however, DULy be .i.little ahorteDed by means of the following

: . - General Int1emglllion. - . . .

Put AC=.II, BC=h, APC=P, BPC=P'. ACD::z:C, and Ia there be taken for unknown quantities PAC=. PSC Yo The triangfes PAC

UidfBCgive' . .'

Sin. APC : sin. CAP:: AC : CP, and'

Sin. BPC : sin. CBP : ;. Be : CP'; that Is;

Q!- p' a sin. z I'D __ ..:I

<:JIll. : SID. Z :: a: . P =~. &IOU

. 81~ •.

Si pt. b & lin. I op

n, !. sm. !J : : : sin. P' == .

asin.z bsin.!J . _.. •

Hence, . P ~P' ; which may be reduced to a 81n. P' 11n.

81n. 8lD •

.e-6 sin. P sin. !J=O.

• See ~Ix TrIgooometrIe, aad o.pr,'. T~.

• Diqitized byL.oogle

uadrilateral ACBP, we have CBf.~~APC-B~C.

AP, or y=360-P-P/_C-.r. . , •

-P-P/-C=R, then we shall have y=a...,.,.and con- , ~qnf:ntlJ.:a sin. pi sin. x-b sin. P (sin. R cos, .-cos. It,' sin. x)=O.

~~*' by sin. e, there l'8Iults, IJ sin. P'-b sin. P' (mh. R C~B. x

~ .. "'.L ... I -" . I SlD.t&

--c:Q8.-B,FO. -

• ~tt.- .. ,.; h cos. x a sin. P'+b sin. P cos. R

wnehce we ave -.-=cot. x= 6' P' R .

SlD. X SlD. SlD. ~

o !hl~, e.~pres8ion eeperated in~ two parts, wellave

••.• ~ .. ,. IJSlOoP cOI.R " •

. :. ; cot. or= b . P' R +~R ; or, '

sin. SlD. sm.

cos. R a Bin. P' )

"'. ~,.' '1'., cot. z = Bin. R (6 sin. P'cos. It + 1, ; 4ft, .. , • ,'1E-.

cot. or = cot. R (6 ~ 8lDp' . pi R'+ I'); or, lastly, '

IHD. eqs. . '. t, (

OJd~~ P' P R __.. R+ at R ,. "I.,

cot'. or - 6 sm. cosec. cosec. "U1'. c.. _. _~ ~.

Hence, :z being thus determined, we get y from tbJI e~~ ~.

R--z; and CP from either of the expressions given a~ove. ","

We shall now apply the foregoing formula to the solution of the

q .... :INt proposed. '. :' ,': 'j,'

t! :.tlc'; ~ II EXA.PLB?lXIV. .,,' " ':,'

Here a = 6 P = 330 45' 0" P AC=:z

b = 4 pi = 22 30 0., PBCFY , ,'. i., - "

___ --ACB = 104 28 39 found by computation

PLANE ~MBTRY.

~ . ", . ' .. ; . r

160 43 39 360 0 0

R= 199 16 21

~ ..

,.

• \ I

" I.

: \'

cot. x =i siI~;, ~:.~os~~. P, cosec. R cot. R+cot. ~,.i qr, " ~ .; ,

,. ( " main. P' ) . :' -.i"

cot. or = cot.,R. b sin. P COB. R+l and using logarithms

we have - a' = 3 log. fMf-1lt19, '.

Il = 2 ar. eo. 9.6989700.

P' = 22° 30' '(1' sin. 9.5828397 .

P r= 330 45' 0" ar, co. S.0.2552610 -

~ }lVb9P~ .CPIio is neg. 199) 6 21 ar, co. C. 0.0250452

1.09458 + 1.00000

log.

cot.R

0.09458 log.

+ 199° 16' 21"

cot. :z 100 8 10

As sin. 33° 45' 0" ar. co.

Is tQ~. z 105 8 119 ,

So is 6

To PC- 10.4251

Whence the rest may be found.

0.0092311

8.9757993

10.456a594

9.43~1587 0.2552610 9.9846660 0.7781513

1.0180783

In using these Iormu1e great attention mult be paid to the signs of the quantities.

EXAMPLE 'XXV.

Suppose the objects A,B,C, are Been from D, and have their distances AB 7i miles, Be 12 miles, and AC 8 miles, the angle BDA 25°, and CDA 19°; it is required to determine the distances DA, DB,DC.

A,.,.-DA 10.0286,])C 16.7857, DB 14.9095 miles.

EXAMPLE XXVI.

8uppose the objects A, B, C, are seen trom D, and have their distances AB 8 miles, DC 12, and AC 71; the angle BDC being 17° 47' 19". Required the distances DA, DC, and DB.

Am.-DB 12, DC 2t.85, and DA 20 miles.

EXAMPLE XXVII.

If, AB be 8, AC 7.2, and Be 12 miles, and the angle ADB U)7°

56' 13". Required the distances DA, DC, and DB. ,

A,.,.-DB 5, DA 4.892, and DC 7 miles.

EXAMPLE XXVIII.

Let the objects A, B, C, be in a straight line t ~d their distaDces AC 3.626, AB 12, and BC 8.374, the angle AJJC being 19". and BDC 25°. Required the distances DA, DC, and ))B..

Ans.-DA 9.~711, DC 10.861, and DB 16.8485.

EXAMPLE XXIX.

Let the objects A, B, C, as seen from D, be within the triangle; and let the distance AB be 6 miles, BC 12, and AC 9, the angle BOO being 123° 45', and ADC 13~ 22'. Required the distances DA, DC, and DB.

A,. ... -DA 1.372, DB 5.523, DC 8.018.

EXAMPLE XXX.

A ship from Bombay in latitude 18" 57' N, sailed S. W. by S. 224 miles. Required the latitude come to, and the departure.

Ans.- The difference of latitllde ial86.2, and the departure 124.4

Latitude ofBombt.y. 18" 57' N.

DifF. of lat. 186 miles = 3 6 S.

Latitude' come to

15 51 N.

EXAMPLE XXXI.

Having oeeaaion to travel through the counties of Kent and 8ur. rey, I perceived the fort built by Lady James, on .shooter's hill, which bore from me N. N. E.; and after going 20 miles in • W. N. W • .ure~on, I perceived the fort again, which now bore N. E. by:& Required my distanee from it at each station .

.04,.,.-29.93 miles, and 36 miles.

EXAMPLE XXXlr.

Digitized by Google

From a Ilhfp 'at sea, I observed a point of land to bear~. bI S., and after aail1D2 12 miles N. E., it bore 8. E. by E. Required the distance of the last place of observation from the point of land.

AM.-26 miles.

PLANE "rlt.IGONOMETRY.

E~LBXXXIlL

Sailing N. N. W .• thu •• of6kaota III u...-.a,·8h ..... 1 di .. covered two light-houses, the northenunOilt of which bore N. N. E. and the other E. by N., and at 10h. 30m. the northernmost light bore E. N. E., and the 'other E. 8. E. The bearing and distance of the lights from each other are required.

CalcullJtioa.-In the triangle ACD are given the side AC equal to 15 miles, the anjtle ADC 3 points, the interval between E by N. and E. 8. E. and' the angle CAD 4 points, the distance between 8. S. E. the opposite point to N. N. W:, and E. S. E. j to find CD = 19.09. Again. in the triangle ABC are given AC as befo,re equal to 15 miles, the angle ABC equal to 4 points, the interval between NoN.£. aad E.N.E. and the angle ACB also 4 points, the interval between the N. N. W. and N. N. E. points; hence the angle CAB is a right aagle j eonsequently, we get BC = 21.21.

:r-.tly, in the triangle BC]} are given the sides CB, CD, equal to 2l.21 arid 19.00 respectively, and the included angle BCD 5 points, the interval between N. N. E. and E. by N. ; to find the angles CDB =trr 30',. CBD =560 15' =5 points, CBE = BeN =2 points, and the distance BD = 19.09.

ELUlPLB XXXIV.

The side AB of a pentagon bein~.I~ toises, the face of the bastion AC 50, the nonnal or perpendicttlar KL 30; it is required to find, by trigonometrical calculation, all the other lines and angles of the fortification, supposing the line of defence AH to be equal to a line drawn from A to D.

. AD 100 A

SolutiOlt_Here 9 = T =9O=AK. ~--::--~r---~

Hence, in the right-angled triangle AKL, AK (90): R : : KL (~: tan. LAK _lao~. Becaule AD i. the side 360" ofa regular pentagon, we bave -"-5" =

'12" .

1S1" = AOB~ and T = 36" = AOK, 0

whence ~,=M' = EAK, and 5¥-1&> 26" = $0 34' = EAC, which- MDgdouMed is 710 8', the BBllent angle PAC or DBJt. Join Be; then will AB~ be a triangle in which are given AB, A£, and their contained'angle- BAC ; to fiIld ABC=6° 48'. Now sin. ABC (6'> 48') : AC (60): : sin. BAC (180 00') : BC = 138.52, equal to the l1ne of defence AH 01':8&. In the triangle 'SCQ, A:BG-ABC=lflb W_.,48' = no 38' =CBG. Because Be = BG, we have

1"-2U0?JWt _ ~~ =840 ll' = CGB. .

Again, because AB and EF' are parallel~ and AH, BG equal j we have the angles BAH,ABG,AHE, andBGF all equal, that is, each equal to ISO 26'.

III the triangle CGH, we have the angle CGB+ BGH = 840 lI' +lB' 2W = 10'J0 3'1' = CGR; t80"-(CGH+ CHG) = 1000-(102" :J1' + ISO 26').= 500 67' = the angle HCG; and the side CH = AH -AC = 133.52-50= 83.52 = CH. Then sin. CGH (102" 37'): eft (83'.52)~: sin. CHG (ISO 26'): the flsnk CG or DH = 2'].062:: sin. BeG (58'" 67') : the curtain OR = 73.323.

Digitized by Google

INTB.OJ)UCTION.

TABLE OF THE MEASURES OF THE PRINCIPAL LINES AND ANGLES IN REGULAR FORTRESSES, FROM FOUR TO

..

TWELVE SIDES INCLUSIVE.

Nam .. of PolygoD&
Nam .. oC Side& and Angl ... ~,p~~ Hepta./Octag. NOIlllg. Decag. Undec. Dodec
---- ----1----
Exterior aide, In toilea 1110. 1BO. ISO. ISO. 180. 1BO. ISO. 1BO. 1110.
RadlUl 01 exterior lide 1!7.3 153.1 lS0.0 107.4 235.1 16.'1.1 !91.! 319.4 347.7
Interior side 115.5 113.9 130.6 136.' 140.0 U!.9 144.3 146.8 148.1
£."diUl 01 Interior side Sl.7 105.4 ise.e 157.0 183.0 iIlS·9 233.4 259.7 !S6.1
aplta! • • 45.6 47.1 49.3 50.5 6!.! 54.! 57.S 59.7 61.1
2;:"* . . lII.5 'l7.0 ao.O 31.0 34.0 36.0 39.0 .t1.0 43.0
rtln • • 78.0 77.1 76.4 7 s. 9 75.3 74.7 73.7 71.4 69.3
FlanIt • • to.3 24.5 21.3 29.! 31.1 33.0 35.8 37-0 38.1
Face • • 60.0 50.0 30.0 50.0 50.0 50.0 50.0 51.0 52.0
Line of defence 13.1.0 104.1 135.1 135.8 136.4 137.2 138.! 138.! 138.t
Demigo'1le 18.7 23.4 ~n.1 30.! 32.4 34.l 35.3 37.4 39.4
-- --,---- -- --
Angle of the Centre 9000' 71<>0' 6000'/510f6' 4500' 40'0' 36°0' 3'l.4'- JJOO 0
~:"Ie 01 the p~on 90 0108 0 1'lO 0 US 34 135 0 140 0 144 0 147 16 150 0
ngle 01 the C n 97 I 9S!I 99 15 99 47 100 21 100 54 101 43 10! 15 lot 46
Angle 01 the Shoulder • III 3115 3 ""r' fl on "H" W sn .. ~ ,n"
Anjle 01 But.. or FlsDk. Angle 6156 7436 83 8 S9 26 9336 9624 97 lj 98 16 98 56
Dlmlnilhed AncJe • • 14 I 1642 1816 19 34 ~O it 21 48 23!6 i4 se !,~ 3!
Exterior FlankIng Angle 151 56114636 143 8 140 5! 138 36136 24 133 8 131 0 1118 56
Breadth 01 FOIl. in Tol .... 15 )16 1718119110 Un--;;- APPENDIX.

BAROMBTRIC :MBABURBIIIBNT OF ALTITUDB8.

Having given a pretty full view of the method of measuring the heights of objects geometrically, we shall here subjoin that of determining them by the barometer, thermometer, and hygrometer.

That the observations may be carefully and properly made, the persons who undertake them should be provided with two portable barometers of the best construction, filled with mercury of the same specific gravity, on which, by means of a vernier properly adapted ·to· the scale, the hei~ht of the mercurial col~s may be read, off to the 500th part of an inch j each barometer being fitted up WIth an attached thermometer, set in the wooden frame in the same m~er as the barometer tube is. The ball of each thermometer would be best . if nearly of the same diameter 88 the barometer tube. Besides these, they must also be provided with two other thermometers detached from the barometers, Of these barometers, one, with its attached and detached thermometers, is to be placed in the shade at the top of the eminence, while the other remains below. Let them continue in their places at least a sufficient time for the detached thermometer to acquire the temperature of the air, that is to say, till the contained fluid is stationary. Then the observer on the emi-. nence must note down the height of the mercurial column in the barometer, 88 well 88 the temperatures exhibited by the attached and detached thermometers j and, at the same time, the other observer must make like observations upon the instruments b e low. It~ in

Digitized by Google

PLANE TIUOOlIOJfETRY.

this manner, three or four set. of obaervation • .be taken, at each station, after short intervals of time, and the mean of the res. farDished by these sets respectively be taken, the prubabjlity of error in the ~true altitude deduced by the following - rules will be much dimiDished. When our third method of computation is adopted, two ofDanieU'. hygrometers must be employed to determine the dew points at each station. If the observations be ~ on several successive days, the position of the instruments ought to be changed at each station alternately, at the aame time comparing each pair of instruments to determine their index error should there be any. It is also advisable to make the observations in serene weather, between 11 and 12 o'clock. For it has been found that the computed heil{hts are too nnall, when the observations have been made near sunrrse or sunset, or when the wind blows fresh from the south; and that, on the contrary, the computed results are too great, when the observations are made about tliree o'clock in a hot summer day, or during a brisk wind from the north or east.-

I. Dr Robi8On" Metlwd.

In this method no tables are required; it will be sufficiently exact for moat purposes, and is not difficult to remember. It was deduced from the following considerations:

1. The height through which we must rise in order to produce any fall of the mercury in the barometer is inversely proportional to the density of the air, that is, to the height of the mercury in the barometer.

2. When the barometer stands at 30 inches, and the air and quick.silver are at the temperature of 32" of Fahrenheit's thermometer, we muat rise through 87 feet to produce a depression lrJ of an inch.

3. But if the air be of a different temperature, this 87 feet must be increased or diminished by about 0.21 of a foot for every degree of difference of the temperature from 32".

4. Every depee of difference of the temperatures of the mercury at the two stations makes a change of 2.833 feet in the elevation.

Hence the following rules: .

I. Take the difference of the barometric heights in tenths of an

inch; call this D. .

II. Multiply the difference d between 32" and the mean temperature of the air by '21, and take the sum or difference of this product and 87 feet. This is the height through which we must rise to cause the barometer to fall from 30 inches to 29.9; and may be called h.

Th 3ODk. the ,_ .... .-:1 eleva' 1

us -- IS approxlllHlWlU tion very near y.

fill

IV. Multiply the difference) of the mercurial temperatures by 2.833 feet, and add this product to the approximated elevation if the u1!.per barometer baa been the warmest; otherwise subtract it; tlJen will the resulting sum or difference be the corrected elevation.

Or, this rule may be expressed by the following formula, where d is the difference between 32" and the mean temperature of the air, D is the difference of barometric heights in tenths of an inch, m is the

,

• One penon may perform the whole operation with one eet ofinltrumenta by making the observations two or three timet! alternately at the top AIId bottom, and taking a mean of the rewlta at each station.

Digitized by Google

.... barOJlletne keight, ~ \be dijl'er.eue between u.e mercurial..... .

pen_a, 8Ild B ill, tile CltneCt elevaDon.' " ,

E ~aJ±O.21d) '±~x2..833.

m

Por M\ example, SDPpeee that the mercuPy in the hlrome_ter at the lower- stati~ was 29.4 inch~, ik temperature 50!' of Fahrenheit's thermometer, a~ the temperattu'e of tile air 4,5<1; the ~~ ef the mercury at the llppe station 26.19 inches, itS temperatuPe 4,6, aM.

t\le tem~u,,& of'the' ah- ago. '

Here D = 994 - S!51.9 = 42.1 1::= 81+ C19XfJoI) = 89.1

". = 1(29.4+ 25.19) = 27;-

30Dl . l' '

____"":;:;:, app~~xl~ e eVlJ,tiOn :;::

'In

Correction for t«:m~. mere. 4 X 2.833 =

Correct elevation in feet Do in fathoms

11. _n,. Hulton's MetIiotl.

4IZl.24

11.83

4111.91 685.32

1. Observe the height of the barometer at the bottom of any height or depth intended to be measured, with the temperature of the quicksilver by means of a thermometer attached to the barometer, and also the temperature of the air in the shade by a detached thermometer.

2. Let the same thing be done also at the top of the said height ox d~, and, at the same time, or as near the same time as may be. And let those altitudes of the barometer be reduced to the same temperature, it it be thought necessary, by correcting either the one or other ; that is, augment the height of the mercll1' in the colder temperature, or diminish that in the warmer, by its lJ1JlIiltb part for every degree of difference of the two. The altitudes 80 corrected being denoted by M andm.

3. Take the difference of'the common logarithms of the two heighta of the barometer, corrected as above, if necessary, cutting ofFS fig:u.res next the right hand for decimals, when the log. tables go to 7 figures, or only 2, when they go to 6, and 80 on; or, in general, remove the decimal point 4 J>1aces more towards the right hand, those on the left are fathoms in whole numbers.

4. Correct the number last found for the difference of temperature of the air as follows.: Take half the sum of the two temperatures for the mean one; and for every degree which this differs from the temperature 31°, tate SO mallY times tJae t'hth pan of the fathoma above found, and add them if the mean temperature be above 31°, but subtract them it' the mean temperature be below 31:°; and the sum or ttift'erence will be the true altitude in fathoms; or, being multiplied

by 8 it will be the altitude in feet. .

Same example.

~-"AT ~I

Mean 42 Dur. 4 II

Barometers.

29.4 lower 25.19 upper

Digitized by Google ' .

PLANE TBoISOIIOMETRY.

As 9000: 4:: sa9.4: .0123

Mean 42 corr. .OUB

Stand 31 M = W.3877 log.

4681656 401~

435 : 11 :; 669.374 : 16.924 Corr. 16.924

'"The altitude sought 686.298 fathoms.

Let the state of the barometers and thermometers be as follows to find the altitude.

Thermomeien II Barometer,.

Detached. I Attached. Lower ~.68

57 57 Upper ~.28

42 43 Altitude 119.897 fathoms.

Metlaod III.

The foregoing m.edlods have been found &0& experience to give results tolerably correct ill ordiaary cire\Wl8tance., though they de. viate considerably &om the trath in peculial' cases. To obviate this, as far as possible, we have given another method, which, it is hoped, will prove very accurate.

tn thia case let B be the height of the Enslith baroule$er at the lower station, b that at the upper. t, the temperature by Fahrenheit at the lower, and t' that at the upper, L the latitude of the place of obaervation,fthe elastic force of vapour at the lower, andf that at the upper, and H the height of the one place above the other in feet, then

.. { ~31

H=60000 (1.315)~(1+0.00268 cos, 2 L) X

l+f' B-tf

(1+ B+b{I+0.6001(t-t,)})log. (b{l+O.OOOl{t::t'H-V")} .(A)

DifF. 11

m= 26.19

log.

Hot' T....at

The 18ctors (1.315) 180 . and 1 +0.00268 COB. 2 L, may be redueed into tables' and, if given in logarithms, they will be very readily applied. ff the centigrade thermometer be used, then

ti"t

B ::::60046.6 {(l.315)200(l+O.~ COB. 9 L) ~

1· lJ:L' 1 B--\f } B

( + B+ b{ (+O.oool8(to-t')) OS-(bh ... o.oool8(t-f) }-tf) ( )

In whicb ease also S, b,f, ahdj' tnay be given in reference to the :french standard nletre.·

'the log. of the constant 60000 feet may be employed with ad. vantage, being 4.778151.

i'f L~place'B constant 16399 ml!t!!~ .o_r 6O~.6 feet, be taked, the oonstant lCJgaritbm would be 4.1ttUMU, and the factor 1+0.00268

doe. S L must be used.t .

. • &e ~iGt'l Trait~ de ilbysiqae, Tome I. p. ~J.

+ A. LaplaGl'& cdnataat ill perhaps the lJIote lIc:curate, it may be used i Digitized by

th C81i1ello ooge

INTRODUCTION.

BAROMETRIC TABLES.

TABLE I.

'rABLB OF TIIB DBPRB8810N OF IIBBCUBY IN GLAS8 TUBB8.

DepmoIoIIo by
DIam. Ivory. Uplaee. I You",.
-
I", J", I", J",
0.05 0.29494 0.2964·
0.10 0.14028 0.13940 0.1424
0.15 0.08628 0.08538 00880
0.20 0.05811 0.05798 0.0589
0.25 0·04075 0.04117 0.0404
0.30 0.02916 0.02965 0.0280
0.35 0.02110 0.02165 0.0196
0.40 0.01534 0.01591 0.0l39
0.45 0.01117 0.01174 0.0100
0.50 0.00835 0.00868 0.0074
0.60 0.00443 0.00462 0.0045
0.70 0.00228 0.00244
0.80 0.00119 0.00128 This table is to be used onl y when two barometers, differing considerably in their internal diameters, are employed.

The expansion of the volume of mercury for 10 Fahr. = 0.000086, more correctly than O.OOOl, though the difference in the nicest barometric observations is almost insensible.

TABLE II.

BB DALTON'8 TABLB OF THB ELASTIC FOBCB OF A.QUBOU. VA.POUR.

Barometer 30 Incite •.

. Temp. Force. Temp. Force. Temp. Force. Temp. Force. Temp.) F_

- ~-----..---

Fahr Inch. of Fahr. Inches of Fahr. Inches of Fahr. Inch. of Fahr. Inch. of

• Mercury. Mercury. )(eI'rn.ry. Mercury. Mercury.

0" 0.064 20 0.129 40 0.263 60 0.524 80 1.000 1 0.066 21 0.134 41 0.273 61 0.542 81 1.040 2 0.068 22 0.139 42 0.283 62 OJj60 82 1.070 3 0.071 23 0.144 43 0.294 63 0.578 83 1.100 4 0.074 24 0.150 44 0.305 64 0.597 84 1.140 5 0.076 25 0.156 45 0.316 65 0.616 85 1.170 6 0.079 ~6 0.162 46 0.328 66 0.635 86 1.210 7 0.082 27 0.168 47 0.339 67 0.655 87 1.240 8 0.085 28 0.174 48 0.351 68 0.676 88 1.280 9 0.087 29 0.180 49 0.363 69 0.698 89 1.320

10 0.000 30 0.186 50 0.375 70 0.721 90 1.360 II 0.093 31 0.193 51 0.388 71 0.745 91 1.400 12 0.006 32 0.200 52 0.401 72 0.770 92 1.440 13 0.100 33 0.207 53 0.415 73 0.796 93 1.480 14 0.104 34 0.214 54 0.429 74 0.823 94 1.530 15 0.108 35 0.221 55 0.443 75 0.851 95 1.580 16 0.112 36 0.229 56 0.458 76 0.880 96 1.630 17 O.l1Q 37 0.237 57 0.474 77 0.910 97 1.680 18 0.120 38 0.245 58 0.490 78 0.940 98 1.740 19' 0.124 39 0.254 59 0.507 79 0.971 __ 99 }.800

Digitized by~UU~L\::

PLANE TRIGONOMETRY.

TABLE III.

LOGARITHMS OF THE BULK OF GAS,

From the formula 1:0 X log. 0.1883027, in which z is the number or degrees above 32" Fahrenheit.

iI'-,. Leg. Balk. Temp. LOII.B. Temp. Lot. B. IT-p. Lot. II.
'"- - - -
00 [975413 25° 1.994622 50" 0.013800 75° 0.033039
1 .976181 26 .995390 51 0.014599 76 0.03380'1
2 .976950 27 .996158 59 0.015367 77 0.034567
3 .977718 28 .9969'J7 53 0.016135 78 0.085344
4 .978486 2D .997005 541 0.016904 79 0.036112
5 .979265 30 .998463 55 0..0.17672 80 0.036881
6 .980023 81 .999232 56 0.018440 81 o.O.'~7649
7 .980791 32 0.000000 57 I 0.019209 82 0.038418
8 .981560 33 0.000768 58 0.019977 83 0.039186
9 .982328 84 0.001537 59 0.020745 84 0.039954
10 .989096 35 0.()()'..B)5 00 0.021514 85 0.040723
. 11 .983865 36 0.003073 61 0.022282 86 0.041491
12 .984633 37 0.003842 62 0.023050 87 0.042'l59
13 .935401 38 0.004610 68 I 0.023819 88 0.048028
14 .986170 39 0.005378 64 I 0.0.24587 89 0.043796
15 .986938 40 0.006147 65 I 0.025356 90 0.044564
16 .987706 41 0.006915 66 I 0.026124 91 0.045333
17 .988475 42 0.007683 67 I 0.026892 92 0.046101
18 .989243 43 0.008452 68 I 0.027661 98 0.046869
19 .989911 44 0.009220. 69 0.028429 94 0.047638
20 .990780 45 0.009989 70 0.029197 95 0.048406
21 .991548 46 0.0.10757 71 0.029966 96 0.049174
22 .992317 47 0.011526 72 0.030734 97 0.049943
23 .993085 48 O.oI2294 73 0.031502 98 0.050711
24 .993858 49 O.oI3062 74 0.0322'T1 99 0..051489 P. P.·1 ·2 '3 '4 '5 ·6 '7 '8 .g

to tenths 77 153 238 307 ~ 461 538 615 691

TABLE IV.

LOGARITHMIO VALUES OF 1 +0.00268 ooe, 2L.

Lat. Lot- Lat., Lor· Lat. Log. La!. Lo,.
00 0.001162 ISO 0.001045 2~ 0.000716 39" 0.000242
1 0.001162 14 0.001027 27 I 0.000684 40- ().()()()202
2 0.001100 15 0.001007 28 0..000651 41 0.000162
3 0.001156 16 0.000986 29 I 0.000617 ~ 0.000122
4 0.001l51 17 0.000064 30 i 0.000582 43 0.000081
Ii 0.001145 18 0.00094L_ 31 . 0.000546 44 0..000041
6 O.OOIl38 19 0.000016 32 0..000510 45 0..000000
7 0.001l29 20- 0.000891 33 0.000473 46 9.999959
8 0.001118 21 0.000864 34 0.000434 47 9.999919
9 O.OOIl06 22 0..000836 35 0..000398 48 9.999878
10 0.001093 23 0.000808 36 0.000360 49 9.999838
11 ".001078 24 0.000778 37 0.000321 50. 9.999798
12 0.001062 25 0.000747 38 0.000281 51 9.009758
- 50

INTRODUCTION.

TABLE IV.-Continued.

La!.i Log. La!. . . Log. La.!. Log. La!. Log •
52° 9.999719 62q 9.999349 ~72° 9.999059 82" 9.998882
53 9.999679 63 9.999316 73 9.999036 83 9.998871
54 9.999640 64 9.999284 74 9.999014 84 9.998862
55 9.999602 65 9.999253 75 9.998993 85 9.998855
56 9.999566 66 9.999222 76 9.998973 86 9·998849
57 9.999527 67 9.999192 77 9.998955 87 9.998844
58 9.999490 68 9.999164 78 9.998938 88 9.998840
59 9.999454 69 9.999136 79 9.998922 89 9.998838
60 9.999418 70 9.999109 80 9.998907 90 9.998838
61 9.999383 7] 9.999084 81 9.998894 EXAMPLE I.

To determine the height of Arthur's Seat above the sea at Leith by the following observations, the height by levelling being 802.66 feet.

Bar.

Leith Pier 29.567 Arthur's Seat 28.704

Att. ther.

55.25 5].75

Det, ther,

54°.0 50.5

Dew point.

500.0 48.5

f=0.375 f'=0·'357

Fah. ther. 54°.0 50.5

3.5 .. f+f'=0.732 28.704 X 0.0001 X 3.5=0.010 nearly, and

b=28·704+0.0]0 =28.714

Sum

104 .. 5 Constant log. of 60000 feet

4.778151 0.015367 153 38

Half 52.25 log. B . . .

B=29.567, B-tf =29.567-0.062=29.505 log. lA69895 b =28.714, b-if'=28.714-0.059=28.655 log. 1.457201

Difference

.012694log.2.]03462 138

0.006181 206 25

H =799.32 feet H'=802.66

2.902721 )0

Defect =3.34 feet 11

The operation, when Laplace's constant is used, would be-as follows:

• The t and t' in the denominators of the fractions in the formula should have been .. and.", the temperatures of the attached, to distinguish them from those of the detached thermometers.

Digitized by Google

Excess = 0.46 foot, or 5t inches EXAMPLE II.

Required the height of the Peak of Snowden above Caernarvon quay from the following set of observations?

Bar. Att. Ther.

Caernarvon Quay 29.984 56.5

Snowden Peak 26.271 42.75

Constaht logarithm .

Correction for latitude 53° 4'

55°.25 + 43" Q

2 _59 .125 log. B.

EXAHPLE III.

Captain Sabine found the height of a hill at Spitzbergen, determined geometrically, to be 1644 feet; required its height barometri-

cally from the following set of observations? '

Observed height of the barometer at the bottom.

In. In •

Barometer, (diam.oftube 0.30) 29.6735

Reduction to 3~ F. . -0.0200

Capacity . . -0.0561

Capillary action (Young) + 0.0280 Index . -. +0.1960

PLANE TRIGONOMETRY.

Laplace's constant log. in feet

1 + 0.00268 cos. 2 L for 56° Mean temperature 52.25 log. B

Difference oflogs of corrected altitudes, log.

1.01256 log.

H=803.12 H'=802.66

B--l.f =29.920 log. 1.475962

b'-!f'=26.262 1.419328,

Difference, . 0.056634 log.

f+f' 0.656

1+ B+b =1 + 56.182 = 1.01168 log.

H _ 3561.2 feet H' = 3555.4

Excess 5.8

I I

I

.

r

!

True height

+0.1479 29.8214

51

4·780646 9.999566 0.015367,

153 38 2.103462 138 0.005181 206 25

2.904782 70

12

Det. Ther.

55.25 43.00

Dew Point. . 50c.25 41.00

4.'lOO646 ' 9.999679

0.013062 77 15 4 31 2."753047

0.004751 247 33

3.551592

Attached ther. 39°.75

Detached 34 .90

Dew point 34 .00

By Daniell's' hygro-

meter.

Digitized by Google

62

INTRODUCTION.

Observed height of the barometer, &C. at the top.

In.

Barometer, (diam. of tube 0.15) 28.0075 Attached there 3&'.4

Reduction to 32" F. -0.0105 Detached . 35.4

Capacity . • -0.0445 Dew point DL 35 .5

Capillary action (Young) + 0.0880

+0.0330

True height

28.0405

Constant lo,rarithm . •

Correction ror latitude about 80° •

B-lf=29.8214-0.0357=29.7857 log. 1.474008 b -If'=28.0405-O.0375=28.0030 log. 1.447204

Difference 0.026806 log.

34.9+35.5

Mean temperature 2 = 35.2 log. B

I + f+f'_ 0.214+0.225 1 +0.439 -100759 I

B+b- 57.00 57.00- . og.

.

Bar. H = 1635 G~. H = 1644

4.780646 9·998907

66 2.428135 0.002305 153

0.003029

247

3.213488

Difference - 9 feet

By another set of observations.

In. In.

Barometer, at bottom 29.8304, at top 28.0624 corrected.

Attached thermometer 39°.4. 35°.2

Detached 35 .4 34 .2

Dew point 35 .4 34 .2

Constant logarithm • 4.780646

Correction for latitude ago •• 9.998707

B-lf =29.8304-0.0374=29.7930 log. 1.47411ii

b -!f'=28.0624-0.0360=28.0264 log. 1.447569

Difference 0.026550 log.

. 35,4+34.2

Mean temperature of the air 2 34.8 log. B

r+J' _ .225+'216_

1+ B+b _1+ 57.89 _1.00762 log.

Bar. H = 1618.3 Geo. H = 1644.0

-

Diff. - 25·7

By the first set 01- experiments H = 1635 feet

By the second ~. • 1618

Difference

17

2.424065 0.00]537 615 O.OO3O'J9 268

3.209067 979

88

Digitized by Google

PLANE TRWONOMETRY.

Captain Sabine thinks there is some error in the secood set of experiments, arisingfioom the circumstance. that Mr Foster. his assistant, was obliged to hold the instruments to prevent their agitation by the wind.

It is proper to remark, that Captain Sabine finds 1644.58 for the first and 1630.66 for the second set of observations, as stated in the Philosophical Transactions of the Royal Society of London. but the particular formula he used is not mentioned. The usual formulee given by Roy, Shuckburgh, and Laplace may give the height more near the geometrical method in certain cases, such as in a mean state of the atmosphere, than that which we have given. though there is no doubt but that the circumstances which have induced us to give a new method, involving considerations not usually attended to in such measurements, are more conformable-to the laws of nature, and will in time become more accurate as those branches of physical science on which they depend are rendered more perfect.

-The dew point is supposed to be found br Daniell's hygrometer.

If that instrument is not at hand, the dew point may be found by two good thermometers, one of which has its ball covered with moistened tiuue-paper. as proposed by Mr Anderson, Rector of the Academy of Perth. who also gives a formula for the barometric measurement of altitudes. in which in some of the corrections I have been anticipated.

Let F. the elastic force of vapour by Dalton's table be thus reduced tof according to the difference between the naked and covered

thermometers. thenf=F- 0.02: xp =F-O.00092~t xp. in which ~ t is the difference between the temperatures of the thermometers, and

p the barometric pressure. .

Now let ~ be the elastic force at the dew point. then

) f F-O.OOOO2p~t

~= 1+0.002084,(/-1') T+O.OO2f(t;-t') nearly _ (1) Here t', the temperature of the dew-point is unknown, but may be determined, first approximately from the numerator of the formula. and then substituted in the denominator, and a second approximation obtained. which will generally be sufficiently correct.

To exemplify this, let the thermometer with the dry ball show 60°F,

and that covered with moistened tissue paper 51t

T-t or ~t. . . . . 8l

Now if the barometer be at 30.4 inches we have from the numerator of formula (1) f =0.524-0.00092 X 8i X 30.4-0.524-0.238= 0.286. Thisf corresponds, by the table o't Dalton to 42~ nearly, 'Which being substituted for t' in the denominator of the formula

. 0.286 0.286 0 2756 hich fi all .

gives ,= 1 +0.0021(60-42) - 1.0378 =. w n y gives

1'-41 °.3, the dew point. This is ~rhaps one of the best methods of determining the point of deposition, as the instruments are not, like the l)ygrometers of Deluc and Saussure, liable to be deteriorated by time, and besides. may still answer other purposes which none of tlie usual hygrometers can.

Cor.-From the same principles. may be derived a formula &0 determine the weight of moisture in 100 cubic inches of air or

Digitized b: Google

54

INTRODUCTION.

W 0.6854 ~ h f' . Wh 01156 d

= 1+0.0021(t~-32) at t e reezmg pomt. en ~='''''' an

t'=41.3 we get from the expression W=0.1837 grains when the air is completely saturated with humidity. But when the temperatures

are 60° and 41° the W = 1 + 0.~~~41 ) = 0.1767 grains in 100 cubic inches. Perhaps this method may be conveniently compared with Mr Daniell's, to show their relative accuracy and consistency ..

It may be added, that l\Ir Dalton states from experiments at moderate heights, that an elevation of 240 feet ~ives a depression of 10 temperature Fah. and an elevation of390feet gIVes a depression of 1 ° F. of the dew point. Hence, if t be the temperature and D the dew point

aH aH

a t= 240 ,andaD=300'

Method IV.

For ordinary heights, such as those usually met with in Britain, the following method, requiring no tables, which is somewhat simpler and more easily recollected than Dr Robison'S, is subjoined.

Let B be the barometric altitude at the lower situation, and b that at the upper corrected for the difference of temperature in the usual manner, the atmosphere being in its mean state with regard to

aqueous vapour, &c. .

Then H=13100 (B+b~~B-b) { 1 + O.OO245C ~ t' 3~)} in feet.

Bar. in. Att. Ther. Det. Ther.

E.x.-Leith Pier 29.567 55°l 54°

Arthur's Seat 28.704 51 i 50 i

28.704 X 0.0001 X 3.5=0.010, and 28.704+0.010=28.714=b

(54+50~ 0) .

1 + 2 -32 X 0.00245 = I + 2O:t X 0.00245=1.04961, hence

58.281 X 0.853

H=13100 X 29.567 X 28.714 X 1.04961 805 feet.

Height by levelling 803

Difference

2 feet.

EXAMPLES FOR EXERCISE.

1. If the base of an oblique-angled plane triangle be 40, and the other two sides 20 and 30, what is the length of the perpendicular ?

Ans.-14.52369. .

2. If the base of a plane triangle be 40, and the other two sides 20 and 30, what are the segments of the base made by a line bisecting

the vertical angle? Ans.-24 and 16.

3. The hypotenuse of a right-angled triangle is 19630040. and one of the legs 19630000; required the two acute angles?

Ans.--6' 56".4, and 89° 53' 3".6. . 4. If the sides of a plane triangle be in proportion to each other M the numbers ·h }, and i; what are the angles?

AlIs.-1l7° 16' 46", 36° 20' 10", and 26° 23' 4".

5. At the Observatory on the top of the Calton-hill, 350 feet above the sea at Leith, the angle of depression of the horizon. marked by

Digitized by Google

PLANE TRIGONOMETRY.

55

the sea down the frith of Forth was IS' 12" byobservation. Now supposing the effect of refraction to be one-twelfth part of the whole, this must be increased by one-eleventh of itself, or the true depression would be 19' 51".28. Required the earth's diameter?

Ans.-7946 miles.

6. Suppose the height of Melville's Monument, in St Andrew's Square, Edinburgh, to be 60 feet, and that the figure placed upon the top of it is ] 2 feet high, at what distance from the monument may the statue be viewed under an angle of 3°, and what is the greatest angle under which it can be seen?

Ans.-It will be seen, under an angle of 3°, at the distance of 208.23, or 20.75 feet, and the greatest angle under which it can be seen from a point in the horizontal plane is 5Q 13'.

7. It is required to find the distances from the Edystone lighthouse to Plymouth, Start Point, and the Lizard respectively from the

following data : .

{Plymouth to Lizard 6O}

The distances from Lizard to Start Point 70 miles .

. - . Start Point to Plymouth 20

Plymouth } {North

Lizard bears from Edystone W. S. W.

Start Point E. by N.

{Lizard 53.04l

. Ans.-From Ec1ystone to Plymouth 14.33 { miles.

Start 17.36,

8. Barometers. Thermometers. Required the Altitude.

Lower 29.45 Attach. I Detach. Ans.-409.61 fathoms,

Upper 26.S2 38 31 or 2458 feet, by Hut-

41 35 ton's method.

EXAMPLES BY THE FRENCH MEASURES,

OtiOei'y.... I .~eJglit .or I _Attached _.uetaChed _~w Latitude.
Humboldt. JSan.ilu.ete.c. ·Ibt:rwuwetcJ". Tbowuxnd.er. Point.
~ndiu, ~'509818:2O"'O <en'. IS0.75 cent. 16°.0 cent. 5°0N.
e. Oc. 0.76294425.3 25 .30 20.0 H=3543m
I
Chimb. .377275'100.0 cent. ~ 1°.6cent. 0°.0 cent. ]°45' N.
Pac.Oc. 0.762000:25.3 +25.3 20.0 H=5925m Calculation of tile last Example by Method III.

Constant 18393 metres log. 11.85 1185

(1.375) Too = _. - X 0.138303

100

Latitude 1° 45' log. B-lrf-0.7591l4 log. 1.880307 b -!I' =0.377471 log 1.576884

Difference 0.3034231og.

f+ l' _ 0.022373

1+ B+'b-1+1.l36585 1.019686

4264653 O.oI6389 0.001161

1.482048 0.008467

H=59254 metres . 3·7727]8 Or 19441 English feet, the height of Chimborazo above the level of the Pacific Ocean.

Digitized by Google

56

PART II.

SPHERICAL TRIGONOMETRY.

SECTION I.

Definitions, Pri"ciples, and General Properties.

1. Spherical Trigonometry is that branch of mathematics by which we are enabled; in all cases, where three of the six parts of a triangle formed by arcs of great circles in the surface of a sphere are given, to compute or determine the other three.

2. In plane trigonometry the knowledge of the three angles is not sufficient for ascertaining the sides; for' in that case the relation« only of the three sides can be obtained, and not their value; whereas, in &pherical trigonometry, when the sides are circular arcs, whose value depends on their proportion to the whole circle, that is, on the number of degrees they contain, the sides may always be determined when the three angles are known. Among other remarkable differences between plane and spherical triangles are,

(1.) That in the former, two known angles always determine the third; while in the latter they never do.

. (2.) The surface of a plane triangle cannot be determined from a knowledge of the angles alone; while that of a spherical triangle always can.

3. A sphere or gllJbe is a round body formed by the revolution of a semicircle about its diameter, which remains fixed.

4. The centre of the sphere is the same with that of the revolving semicircle.

5. The azis of the sphere is the straight line about which the semicircle revolves.

PROPOSITION I.

6. If a sphere be cut by a plane, the section will be a circle.

Let the sphere AEBF be cut by the plane ADB; then will the

section ADB be a circle. Draw the chord, or E

diameter of the section AB, perpendicular to the .seetion ADB, and through the centre C draw the axis of the sphere ECGF, which will (Eue, III. 3.) bisect the chord AB in the point G. Also, join CA, CB; and draw CD, GD, to any point D in the perimeter of the section ADD.

Then, because CG is perpendicular to the plane '

ADD, it must be perpendicular both to GA and 'F

GD. Hence CGA and CGD are two right-angled mangles, having Digitized byGQogle

J

SPHERICAL TBIGONO:METBY.

the paopeadieular CO common, and the hypotenuse CA equal to the hypotenuse CD, being both radii of the same sphere; therefore their third lides GA, OD, are also equal. In like manner, it may be shown, that any other line drawn from 0 to the eireamfer; ence of the section ADB, is equal to GA, or GB; and eonsequently that section is a circle .

. Cor.-If a ~here be cut by a plane through the centre, the aeetion i •• circle, hsvmg the same centre with the sphere, and equal to the circle by the revolution of the half of which the sphere was deacribed. :ror all the straight lines drawn from the centre to the surface of the sphere are equal to the radius of the generatfug semicircle. Thererore the common section of the spherical surface, and of a plane: puaing through its centre, i. a line lying in one plane, having all its points equallr distant from the centre of the sphere, and is consequently the circumference of a circle, having for its centre the centre of the sphere, and for its radius, the radius of the sphere, that is, of the semicircle by which. the sphere is described. It is therefore equal to the circle of which that semicircle is a part.

7. Any circle formed from the section of a sphere, by a plane through its centre, is called a great circle of the sphere.

Cor.-All great circles of the sphere are equal; and any two of them bisect each other.

The]' are all equal, because they have all the same radii, as has just been shewn, and any two ot them bisect one another; for, as· they have the same centre, their common section is a diameter of both, and therefore bisects both."

8. The pole of a ~eat circle of the sphere is a point in the surface of th~ sphere equidistant from every part of the circumference of

that circle, •

, 9. A spherical angle is an angle on the surface of a sphere contained by the arcs Of two great circles which intersect each other, and is the same as the inclination of the planes of, or tangents at the point ot intersection to, these great circles.

10. A spherical triangle is a figure on the surface of a sphere fonned b:r the intersection of three arcs of great circles, each Of which ia less than a semicircle.

11. A right-anlfled spherical triangle has one right-angle; the' sides about the rIght-angle are called legs, and that opposite the. right-angle is called the h!lpotenrue.

)~ A quadrantal spherical triangle has one side equal to a qu~ drant, or 90°.

13. An oblique~gled spherical triangle haa none of its angles

• ht.

ng14. Spherical triangles are also called equilateral, isosceles, or 600- lene, according aa they have three sides equal, two sides equal, or all

the three sides unequal. . .

15. Two arcs; or anB:les, when compared together, are said to be Glike, or of the lame apection, when both are less, or both are greater than 9O'f. But when one is less, and the other greater than goo, they are said to be unlike, or o~ diff.erent a.ff_eclion~ or characters, .

16. Every spherical triangle has three sides and three angles,

58 Il(TBODUCTION.

IUJd ii any three of theee six paris be giftll, the other tar ... ."._ •

found. •

11. A ltme is a part of the lurt'aoe of. sphere ton~ by -

I&Imicircumferenoea of two great ci.rc1el. - ,

18. A ... tAl eircle of the sphere is that whoM .,laDe duet 1l~ ,...

through the centre of the spnere. .

1~. The 81P8ll circles of the sphere do bot fall under the tonBideration of 81)herical trigonometry, but such only as baTe the ~. centl'e with the sphere itself. And hence it is that eph.rkal trigoDObletry i8 of 10 much Ole in practical 8IItro1iomy, tbe ap~ heavens &8IumiDg the shape of a concave sphere who. cet1Re II dle aame as the Centre of the earth.

20. The Me. of a spherical triangle are all &rca or great ~j whieh, by their inteneetiOh on the surfsee of a ~ert1, t'JOIl8lit11te that triangle.

21. If ABDG be a great circ14! of the

sphere "hose tenth! i. C and PCP' a dia- p

meter of the sphere ~rpendicula.r to ita plane, the pointa P, P are the poles of that circle. And if the small circle abc d be

perpendicular to PP', we call P, .p' the .A . »

poles of that small circle also.

22. The great circles PAP', PGP', pa!!'8- ing ~B.gh the poles P, P' ot' the great circle G, are called m:ondtlrle.t to that circle.

PROPOSITION U.

23. If two arcs of circles meet each other they make two ang~, whiab. are together equal to two right-angles.

Let the arc AB meet the arc CD in the point ~

B; then will the two angles ABC, ABD be equal E

to. two right-angles. For, suppose the are BE

~ be perpendicular to CD, then the angles .

DC, EDD are right-angles.

And since the angle EBD is equal to the angles . B D

BBA, ABD, the three angles, EBC. EBA, ABD.,

are equal to the two right-angles. .

But the two angles, EBC, EBA, are equal to the angle ABC; whence the two angles, ABC, ABD, are also equal to two right.. angles.

PROPOSITION III.

24. tr two arcs 'of a circle intersect each other •. the verti~, 0'"'

opposite angles, will be equal. . .

Le. t the two arcs, AB, CD, intersect each other iny All)

E, then will the angle AEC be equal to DEB, and '

~Oto~a .

For since the are A~ meets the arc CD, the . .

an~les AEC, AED are together equal to two B

right-angles, (Prop. n.) C

And because the arc DE meets the arc AB, the angles.DEB. DEA are also equal to two right-angles.

. Taking away from each the common angle AED J and the re-

Digitized by Google

SPHE~C.u. TlUGONOMETRY.

thtRi1l1&' 'ft~, Aile wIll be equal to DEB. III ,the l&JDe manner It may lJjCed that the angle AED is equal to CEB.

CtW.- e if any number 0' arcs at cUodea mteneet each other,

all the angles formed about the point 01 inteneetion are together

~ te tOO, right .. aaglee. .

PllOPOSITION IV.

25. The arc of a great circle, between th~ pole and the drcu,m. ference of anether great circle, is a quadrant.

Let ABC be a great circle, and P its pole j if PC, an arc of • great circle, pass through P and meet ABC in C, the arc PC ia.

qi!d.: circle, of which PC is an arc, meet ABC again in A, and let AC be the common section of the planes of these great circles, which will pass through B, the centre of the s_Ehere: Join P A, PC.

Because AP=PC, (def.), and equal straight lines in the same circle, cut off' equal arcs, the arc AP = the arc PC j but Al'C is a 1!IenUcircle, therefore the arcs AP, PC, are ~ach otthem quadrants.

COT. 1. If PE be drawn. the an,le AEP is a right-angle j and.

PE, being at right.angles to every line it meets with in the plane of the circle ABC. is at right-angles to that plane. Therefore the straight line ,drawn from the pole of any great circle to the centr~ t)f the sphere is at right-angles to the plane of that eirele j and, con,versely, a Itraight line drawn from the centre of the sphere perpen. tticular to the plane of any great circle, meets the surface or the sphere in the pole of that circle.

COT. 2. The circle APC has two poles, as has been shewn in art. 21., one on each side of its plane, which are the extremities of. diameter of the sphere perpendicular to the plane APC; and no other point. but these can be poles of the circle APe.

PROP08I'J;'JON V.

26. If the pole of a great circle be the same with the intel-section of other two circles, the arc of the first circle intercepted between 1;he other two, is the measqre of the spherical angle which the same two circles make with one another. .

Let the great circles AP, BP, on the surface of the sphere of

'Which the centre is 0, intersect each other fn p

P, and let AB be an arc of another great circle of the pole 88 P, AB is the meaeure of the B1>herical angle APB.

Jom PO,AO, BO ; since P is the pole of AB. P A, PB are quadrants, and the angles AI?-~~r----:;;ll,C

POA, POB are right; therefore the angle AO B is the inclination of the planes of the circles

P A, PB, and is equal to tile spherieal angle .

APB i but the arc AB measures the angle P

AOB. therefore it also measures the spherical ~le A.PB. J

Cor. If two arcs of great circles, P A., PC, which intersect ~ch other in P, be each of them quadrants, P will be the pole of the

Digitized by Google

60

U'lROl)lJCTION.

great circle w.htcbPlltSael! ~o11lh .A and B. the eat\"ellakiea of tbMe ares. For since the arcs P A and PB are quadran .... , the aaglee Po.A, PP;S are l'ight-angles, and PO is theref8l'e parp8Ddicular. to the' ~e AOB, tliat is, to the plane of the great circle which flU .. through A and B. The point A. theref~, is the pole pf.the gNat

circle which passes through A and B. . .

P&OP08ITION Vt.

. ~7. An angle made b, any two great circles of the spbere is t!qu.a1 to the an~le of inclmation of the planes of these circles.

Let BAE be a spherical angle made by two great circles CB,A.

CEA ; then will this angle be equal to the angle B .

of inclination of the planes of those circles. For, ~B

take the arcs AB, AE, each equal to 90°. or a n E

quadrant, and throul{h the pomts B, E draw

the art: of the great circle :BE, and from D, the .

centre of the sphere, draw nB, DE. .A. F» . C

Then, because AB, AF are q.uadrants,

A and C are the poles of the circle of which BE is a part, and the lines DB,. DE. aeeeaeh perpendicular to the common section AC; consequently BDE is the angle of inclination. of the planes CRA. CEA. But since DB DE are equal, being radii of the ~ sphere, the angle BnE. which is measured by the arc BE, is equal to the angle RAE, which is me.asured by the same arc.

And if FH be drawn in the plane CBA, and FG in the plane CEA.ea.ch perpendicular to the common section AC, the angle BFG, which is equal to the angle BDE, will allO be equal to the nngte BAlt

Cor. The angle :BAE made by two great circles of the sphere BA, EA, is equal to the angle n A m, formed by two tangents drawn frofD the angular point A. one in each plane, these tangents being ,each perpendicular to the diameter AC.

PaoPOSITION VII.

~ The distance of the poles of any two great circles or the 8fhere is equal to the angle of inclination of the planes of those

CIrcles. .

. Let AEB, CED be two great circles, and P, P' their poles; then

will the arc PP' be equal to the angle of their p ~

inclination AOC or BOD.

For, since P is the pole of the circle AEB. and P' of CEO, the arc .P A will be equal to PC, being each quadrants, or 900; and if PC, AI?-':.d-....;:ciIk---F'o:.---:1I which is COmmon to each, be taken away, the remaining are, PP', which is the distance of two poles, is «:<J,ual to CA, the measure of the angle of inchnation AOC.

PROPOSITION VIII.

29. The circ~mferenct; of a secondar, is a~ right angles ~ the cir-

cumfe~e~ce of ItS great CI~cle .at the pomt of intersection, ' .

The direction of the eireumferenee of a great circle 'at any poiat

Digitized by Google

-

I

! !

SPHERICAL TRIGONOMETRY.

'1

being the l8Dle 88 the diauleter of ita tangent at that point, the angle OBT, (fipre JlI'Op. V.), is a righ~angle, BT beiag a tangent to BP at the point B. POB is also a right-angle, and the arc PB is in the plane POB, therefore the direction of the circumference PB at B must be parallel to PO. But PO is _perpendicular to the circle ABC; therefore the circle PBP' is at B perpendicular to the circle ABC; hence the arc PB at B is at right-angles to AB at B. For

the same reason PAB is al80 a right-angle. • .

Cor. I.-If a. great circle, PBP', be perpendicular to ABC, and BP, BP' be taken each equal to a quadrant, or 90°, P, P' are the poles of the circle ABC.

Cor. 2.-Ir any two great circles, PAP', PBP', be perpendicular to the cIrcle ABC, they meet at the poles P, PI of that cirCle.

POOPGIIITION IX.

30. In an isosCeles spherical triangle the angles at the base are equal, Let ABE (figure prop. VI.) be a spherical triangle, having the ~ AB equal to the side AE, the spherical angles ABE, ABE are equal.Cor. I.-Hence, if two of the angles of a triangle be equal, the sides opposite to them are likewise equal.

Cw. 9.-A perpendicular drawn from the vertex of an isosceles spherical triangle to the base, bisects both the base and the vertical angle, except when the two sides are quadrants; in which case there are an indefinite number of perpendiculars. .

POOPOSITION X.

31. If the three sides or one spherical triangle be equal to the three sides of another, each to each, the angles 'Which are opposite the equal sides are equal.

PROPOSITION XI. .

. .,

. ~ If two sides and the included angle of one spherical triangle be equal to two sides and the included angle in another, these two tr ingles are equal.

PROPOSITION XII.

33. If from the angles of a spherical triangle, as poles, there be described on the surface of the sphere three arcs of great circles, which, by their intersections, form another spherical triangle, each side of this new triangle will be the supplement of the measure of the angle .which is at its :pole, and the measure of each of its a~le8 the supplement to that SIde of the primitive triangle to which It is opposite. '

PROPOSITION XIII.

34. If the tlmee angles of one spherical triangle be equal to the three angles of another, each to each, the' sides which are opposite to the equal angles are equal.

PaoPOSITIoN XIV.

35. If a side and two adjacent angles of one spherical triangle he equal to a side and two adjacent angles of another, each to each} their remaining sides anel angles will be equal.

• The demonatratiolla, which may be seen in Playfalr'. 01' Legendre's Geometry, are CllDitted, .. the,: would swell this Work too much, but may perhape appear in • m_ complete treatise on trigonometry. that baa been long meditated.

Digitized by Google

PaoJlO8ITION XV.

36. The sum of any two sides of a spherical triangle [s gi'eatel' than the third side; and the difference of any two sides is less than the third side.

Cor.-The shortest distance between any two points on the SUl''' face Ot'11 sphere is the arc which passes through these points.

PRoP(H!ITION XVI.

, 37. '.the greater side or any spherical triangle ill opposite to the greater angle, and. the less side to the less angle.

And, in a similar manner, it may be shown that the leas side is op-

posite to the less angle, and the less angle to the less side. ,

- -

PKOl'08l'l'lO!f XVII.

38. The sum of the three sides of any 8'ph~ triangle is less thAn the circumference of a circle, or 860Q; and the dift'eteQCt of uy two Biles is less than 180".

PROP08ITlON XVIII.

-89.~he SBPl of the -three angles of every spheriaal tria~le is Feater dlan. two right-a.ngles, or 180°, and leas t&an six, or 54()".

Cor.-The sum of any two angles of a spherical triangle is greater than the supplement of tlte third angle.

For the aDgles A+B+C, being greater than two right-angles, or than ACB+ACG, if ACB or C be take. away, the sum oftbe re.. ~ ..g1ft,4+B, IWiU he gr .... tlw1 AcP •

•• I ""

PROPOSITION XIX.

40. If thl! sum of any -two 1i4es 9f a spherical triangle be equal W, greater, or Iess than a semicircle, the 8um of their opposite aogtes·.m, accordingly, be equal to, gteater, fit lese than tw!) right-angles; and conversely.

And, in a similar manner, it may be shown, that If the sum of the two angles Band C be equal t.9, g1'88$er, • less than 1000, the sum of the oppolrite sides AD and AC, will also be equal to, greater, or less than 1000.

Cor. I.-If each side of a spherieal triangl~ be eqnal to, greater; or less than 180", each of the angles will, accordingly, be right, obtuse, or acute; and conversely.

Cor. !a.-Half the sum of any two sides ot a spherical triangle is or the same kind as half the sum of their opposite angles.

PaoPOSITIoN XX.

41. In any right.angled or quadrantal spherical trIang1e, the legs ur sides are of the same kind or afFection as their opposite angles, and conversely. .

The same will also hold if the triangle be quadrantal; for its sides and angles being the supplements of the angles and legs of the po1at' triangle, which in this case is right-angled, the similarity wID be

the same as before. . ,

PROPOSITION xX1.

49. In any right.8ft~ spberieal trianaie the hyJ*enuee itt less r br greater than 90", according as the two T~~, or the two angle-, or a leg and its adjacent angle, are alike or unlike,

Digitized by Google

SPHERICAL TJUomroJdETB.Y.

II

SBCTIO)l II~

Solution qf Spherical Triangles.

IUVING given a view of the general principles and propertiee- ot spherical triangles, .the solution of the various problems in spherical trigonometry ought necessarily to follow. These problems mal be resolved eitlier by geometrical construction or by arithmetical calculation. There are various methods of construction.' but the most simple, and generally employed, ia the .tereographic, in which all the circles of the sphere are represented by straigbt lines or c:ir_.

Of the Stereogrllpkic Prc;ection 'If t!ae Sphere.

Dl:PlNITIOJi&.

l. To ,rojlCl all objeclt, .. it ia eoaunoaly CIlledI i. to ftP~nt evPIrf POlDt of that .o~jed upob the I8IDl' plane, B8 it appears to ~

eye lD a certain pGtltWn. .

n. That plane upon which the object il projeewd ia<called.. pllu.e qf prqJection, and the point where the eye is situated, the projecti"E poi"'.

III. The 8tereogra~ic ~ectilm of tile sphere ia that._ ~ a great circle is assumed as the plane of projection, and one of Ita pole. as the projecting point.

IV. The great circle, upon the plane of which the projectlOQ is

made, is called the primitive. .

V. By the semitangent of any arc is meant the ~ent of.~f that arc.

VI. The Ii.e of meUflre8 of any circle of the sphere ts that diameter of the primitive, produced i .. definiwl¥, which is perpendicular to the lioe of common section of the circle and the primitive.

VII. The ~ecfi8tt, or representaticn of any point in the sphere, is u.e pe11lt m which the straight'Une dra1m fr()m it to the projecting point intersects the plane of projeetion. .

. THEOREM I.

Every great circle of the sphere, which passes through the projecting point, is pl'ojected in a itZaight line, pasUng througa ~e eentre of the ~mltive; and evfE141l'e'" ~'recKoDtld frota the otbet· )tOle of dle pnmitive} is projected into it» semitangent.·

Cor. I_Every small circle, which pa8ie8 througb the projecting point, is projected into that straisht line which is its co~on section

1Ji\ll the primitive. ,

Cor. 2.-Every straight line in. the plane of the )rililitive, -.d produced indefinitely, is the projection of some circle on the sphere passing through the projecting poiDt.

Cor. B.-The stereographic projection of any point on the surface

• For the jnv~tigatioD ?f the properties of this method of proj~n, lee GIf""lJ'" or Keith's 'tteatiseS of Ttlg'<ltlODlell'1, alid·Weat'll1\bthmlatics. .. . Google

Digitized by

of the sphere, is distant froIIl the eentre of the primUtive by tile lie .. , mitangent of the distance of that point frGlll the pole oppoaite die. '.

projecting point. I' J ., I

THEOREM 11. ' ':

. • ' ... 1,..- r

Bftl'y drele of the sphere, which does DOt put through the ,pfOo: •

jeeting point, is projected iDto a cirde. ." " , "

Cor. I.-The centree and poles of all circles parallel to the p~-'

tive, have their projections in its centre. ',; "

Cor. 2. The centre and poles of every circle, inclined to the pri-

mitive, have their projections in the line of measures. .

Cor. 3.-All projected circles cut the primitive in two poiD_ cJi;.

ametrically opposite. '

THBOREH III.

ne eentre of~ projection ofa great circle i. distaat &om th~" centre of the primitive by the tangent of the inelinaQea_ of the ~ I circle to the primitive, and its radius is die eeeant of the aame.

THEOBBIII IV.

The centre of projection of a sinall circle, pet'peildieular .0 1he. primitive, is distant trom the centre of tile primitivebythe~! the distance of the circle from its nearestpo1e, andthe'tadiaatJfpfD-'

jection is the tangent of the same. .' .. : ....

THEOB .. V.

!'be projections of the poles of any circle inclined to ~ .prif8- tive, &'re in the line af measures distant &om the centre of the pr~_. tiye by the tangent and cotangent of half its inclinaQon.· F-;'

'. T'

Any two circles upon the sphere, p&8singtbrough tbepoiesof ...

~eat circles. intercept equal arcs upon them. . ...... ~

!

THEOR.BH VII. . j .',

II, trom either pole of a projected great circle, two ~ ... ' be drawn to meet the primitive and the projection, they "IIViU;i • .: capt corresponding arcs of these circles. . . . _··L •....

THBORBX VI.

Sol.tion qf Rig1at-,Attgletl SpkericlI~ T"~le. ...

~ ',,~ .!. .} 11

The dation of right-angled spherical triangles may be . . ' .. }.

~ed by"'naule inveetigated expreuly for that purpose. _~ •

l~btecl to Napier~ however. for a comprehensive rule or ~e~t, ftRtage'" tile memory, br reducing all the theorems emp10red l~ sulution _1'ight-angJ.ed maoglea to two. This is called the role qf ~ circular pam, and is perhaps one of the happiest examples'of

fleW: IIleDlOl'Y that is known. _ .. ...

. DJlPINITIOJf8. '.. ".

r. If in 'a right-angled spherical triangle the r;e.~angle· be ·set aside.. and the five remainin~ parta of tbe tn.asle .be ~ered, consiating of the three SIdes, and the two oblique 'angl&s .... the two sides containing the right-angle. and the cotnpl~ref

Digitized by Google

SPHERICAL T8IGOROIIETRY.

.'

.. a6er dIIiee •..... il Gf,the, t,.. ..... , ~:olthe h~'-i

~~~;Ce~cular 'p.m, ~y"on~ i~ ~eD ~~ ~

middle part, then, Iff ~ remaining four; the two which' ate im'me_ diately acljaeent to it OD the right and left 'are calledtUljllCstpart, .~ and the oiher-two,' _ell'O' ..... i. ~ frem .,III .. ~Il .. ,.a .. by lID adjilceat part, are called ~,.',.... " ",I'. «; _. "' .. :,.,

This arrangement beiD~....u, the Hltrti_ '.obeIIia .. :t.,_4be M.,

lowing " , ' , ~'

TBEOBBJ(. ' . ,,:\.

Is my right-angled spherical triangle, the rectangle under' ~~'r~ dius, and the sine of the middle part, is equal to the rectangle' under the tangenU of the adjacent parts j or to the rectangle under the co. BINBB of the OPPOBITB parts. _ -

Thi",tbeoJlelll, or rule, may be easily reaellllMred, ~ ~~¥.tr, that -the llrst vowel. in line, "'''gat, cwiu; are reapectively \he flafqe

tl8 the fi1'IIt in fffidIIUe"G4iacM, oppoMe, . " :

or, Rxsin. mid'=rect. tan adj. = rect. cos. op .• ' "

It is usual to convert the equai.o under consideration into .1\ analogy haviBg the unknown '1Wl1ltity for the last term, tboughl to thoee acquainted with algebra, It would be more convenient to mAke it Mimethe first WrJn of 8.11 ~quation, and the remaining terms, .~_ bitted properly according to the rules of algebra, the 188~ , ' ',

PRoaL •• I.

Given three of dle six~, .. , tOr ~pJ.e, ~e ~~",.. one of the angles of a nglit.&ngled spherJcal trlangI~ to find the aides md the remaiDiDg angle.

On the mst OfMBii826, the sun's longitude was I' 10° 32' l~', and the obliquity of the ecliptic ilIo 21' 40"; required the right ....

ceD8ioo and declination it .

An.r.-R. A. 2" 32'" 27'.3 j dec. 14059' 47" N. ' ':'~

'Construclima.- With the chord of 000 describe p

the l_)rimitive circle EPQP' on the plane f1f the sobDtial eolure, and draw the diameters EQ e and PP' 'at right. angles to one another, then nj -,t;J:~~~J will EQ represent the equator, and pp. the :E.r

polar axis. Layoff fro. m the 8lUDe line of chords r .J.!~.l'I-~~

E e=23° 27' 40", the obliquity of the ecliptic, and draw tile dmm.eter e I ~.ti~. the; ~c, at right-angles to whIch draw l' p', :I' J ~ ,;'p 'are the poles of the eclipdc .. }ljQ~ the,li.,of .. mit¥l~iJt8, (Theon:m I.), lay oft' the sun's longttude ~'"O°.Ji' .l~'t or," 32'.2 on the ecliptic, from A to C, then C wjll-be- the 'place ot'~e lun~ and "c m a parallel of declination. Throu~ tbe-pOln .... PCP' .draw a circle of right ascension, catting die ell-tor .~

> .-

• ,l •• ,

• Should either of the oblique ..... or "~nuse, be one of the pvts, then, ..... of the word in the formUla, Ule tJiat derived from ita complement, that ii, for

AM n.d costne, fot" eoftU read 'BiDe, ani 10 OIl. - '. ,,', I

-t'fWtIIe estt otaIaeN,tera. die lIWoIl ~ OIJ ... tzon~J.wbF.FOn.

__ :Go ........ !!"ate4 with &he use of the globes, correct iaeas relative to'dlele ~., be readily obtained. It may be added, tlilt the rin's 10llgitude,'Mld the obllqui£y of the ecliptic, are eomputeli ftoom .. tronomical tabla.· , , .

Digitized by ~~ogle

1'0 sine BC 14 59 47 .

9.412891

..

. ·IJn'lIOM1C'1'ION •

....... , • .., .. in B, mADbetbit _

decUilation, and DCA the remaining angle or -.de' of poIit.ion, .. it III lODletimes ailled, which, in &StrOncmy. is seldom of mo. e •

. ~In the triangle ABC there are 8iven'AO:=fO'» 8i( .lW', ud the angle BAC::23° ~' 40'/, to fiail BO •. _ di8uoee tl ..... ·from tlae equator EQ. CII' the decliaatUm. as it is usually ealled. . Now, since in apherical tripaMeby the .nrar"of'tbe· ...

... ~ to the~. of their "I'f'Uite iIfIglu.,. ...

Therefore,

As sine ABC or radius

Is to aiDe.BAC ~~' 40" . So is sine AC 40 32 12

lO.()OOOO() D.6OOO2l 1J.81~

·T.·tDd·AB lfe ... y.esapl~.t1t. method ot'the eireular }tarts. . JD ~, triangle ABC ere Miven AC and the _gle BAO. to fuld AB the right ascension. Now, since the side CA, the aagle CAB, JlIld the side AD Me all cooneeted, that which stands in the middle or &lte anr1t' A 18 called the middle part, and the .idee Ac aad AB ad~t wit on each side ,are called. the adjacent parts.. . .

<::Onsequently R X COB. A = cot. AC X Ian. AB j and resoiva.

.tAiJ into an analogy, as is frequently done in this counIIry. we bavf!,

As cot. AC 4()" 3~' l~" 10.067939 .. :

Is to radius:. . 10.000000

So is cos. A 23 ~ 40 9.962626

.. ··To tan. AB. ~32'" 27'.3 . .1i.89458"/

· . l' '.' ,

~,.sincecot.: R:: R: tan., or tan.ee cot. toradiJltllll'rity(§ S5,.~ell.~

• . AI radius '. 10.000000 .

Is to tan. AC . 40'>32'l2,f ." 9.932061""

So is cos. A 23 . ~. 40 9.962526

To tan. AB . 2h 32'" 27'.3 .' . 9.894587

the same as before.' .' .•

To those acquainted with algebra, it is better, after the uiapneqif f'oreign mathematicians, still to retain the form of an equatiotdhuc

r Rx coe. A . . .. - :.

. tan. AB = --AC- = cos. A X tan. AC, the radius ~g~

. cot,. . - .'

re:~d by unity; it which cue ten. mlUt 1;M, rejee~ 'in:tbe'~

To lQg. eGf. .A ~ 27' 40" 9.~

add log. tIIP. AC 40 32 12 9.93206l ,

Sant. taD. AD 2b 39'" 27,'3 . .Q.8P4.5a7.·;

. .

· To find th:c:te ACB, since the· paris "Under consideratloii ~ atlll all eoDD AC BtaDdiug in the middle ia asanwed as the middle part, and the angles A and C are the adjacent partt, whence

· • It ., he retnarftdi that'lf the ~rta !lre aU ~~l !bat wtJldI _da 11f !h~ middle la Called the midcne vart. and the other tWit are CII1Ied tlw .t;aemo.parte. ·If ""0 Mly !lre connected1 and one stands by ittoeIf, then this ill called tbe mtcI&IIe part, IIJId

the ot~Ct. two are callc«:the oPT'osite parts. . .•

• _. Digitized by Google

SPHER1£AL TBJQONOME'l'RY.

, .' . • '. t· .. ..' . eos • .6.(: . . •

• x ~.~ 7"' cot. A.. X cot, ~ and cot. C = cot. A = COS; Ae X.·

A, bBnee·· .

TQJog. ~ AC 49'>.·19" Add log .. tan. A 23 Z/ 40

, "

S.am ;: Clbt. C 71 44 42 .2 • • . • 9.518304

Or the eemp, 16 15 17 .8, is called properly the angleof~. tioll, JOIDetimee useful in computing the parallaie8 in solar ecliplea ad ...vtMionl of the fixed stara and pIaJ1ets by the 1IlOOP., • B}' aasoming dift'erent parte of the triaragle ABC £01' the llliddle

~ lila, b~ resolved the following , ,

"

. .

. '

Ezamples for Bsereiee.

1.' On the first of June, 1827, at noon on the meridia1a of Grdrn; wieh, Ute sun', longitude will be 2' 10° 9' 4ft', u.e oWictuity. of .the ecliJjtic 23" 2'1' 36" ; required the right; ll8eeBiion and declinatioa ? .Ans.-R. A. 4b 34m 7'.6; Dec. 21°59'34" N. ':

2. August 12th, 1827. the obliquity of the ecUptic being 93\' 87' 36", the sun's right ascension will- be gb ~1D 29'.3; required hit

luDgitude and declination? : ", ,~

.Ans.-..Lobgitllcle 4' 18" .Q6' 28", D~ 15° 0' 32" S. . . .

. a., ()n thct 10th November, 1828, on the m.ndian of Greenwkh, .. ewt. right aacenaiop will be 15b 2'" 32:.7 and declinatioa l1Q 14'- 12" S.; required the sun's longitude and the obliquity.o£ the

ecli tie~ , .

. 7118. Longi~ude 7' ISO 61' 1", and obliqttity of theec:liptio .. sr 34".

'4. 'en the 2d of March, 1828, when the sun's declination' waa 7° f¥ ~8" ~, an~ obliquity of the ecliptic 23° 27' 35" ; required )lis 1000000tude_ and nght ascension ?

~ngitude 11' 11° 66' 34" J R. A. 22b, sa- 24'. ~,

PBOBLBJI( II .

. When thti oeleItial object is not upon tlwt ~ptic, as. 'the mOoD, or the Jm.met8, and some of the fiDel stars, the right aseension and d.e;. ~ are fonttd by the solution of two rigllHngledtriangles.

1. On the 17th of January, 1826, at noon, on the meridian of GueaWieb,.the raeon'e Iengitude, was I' llQ 5' 14". &ad 1ler latitude 2"'~ 3" N. j required her right ascension .ad declination, the oMlttWty of the ecliptic being 23° 27' 40"? To resolve this eumple it is necessary to employ two right-angled spherical triangles.

~-:ae t'Gregoing figure, the longitude of 'the moon or any star S,

• AD, the lstitutl., DB, the obliquity of the ecliptic BAC, 1ibe ~ht ~on AB and declination BS. Now, supposing a line drawn u..'A.to S, there would be formedthe right-angled spherical trian. gIe ADS, ri,hwngled at D, of which AD and DB are given to nnll • argle DA8 and the side AS. If the position S of the star is ... .., du~ecltptie, tBen'to the obliquity of the ecliptic BAC, a-dd CIIIe aagle DAS, the- ... will be die angle BAS j but if S is within the ecliptic, that is between it and the equator, mbtract the angle DA8 .from the obliquity BAC, and the remainder will be the 'angle BAS. Sinee the side AS, and the angle BAS, are now known, AB

·the~aicinllion; and'BS the d~ _y be fuund. .. ,

Cit~ulation.-By the rule of the cirQUlar' partsh Drst AD and ~

Digitized by Google

are given to find~, and since the last is separated froIIl11Ht·twa first by the oblique angles, it will be the midaJ.e pittt, '&tltt' AD 'etHI US ~e ,the, opposite pa~; th~refore, R X «:08. A~ = cos. 'DS ~~s.

AD', or COs. AS = cos. DS X cos. AD toTadi1l1! nmty.. .'

To log. cOS. DS 2" 34' 3" ·9.900664

Add·log. cos, AD 41 5 14 9.8772~

, . Log. cos. AS 41 9 11 , 9.816768

Again, to find DASI since the right angle does not separate t4~ parts, DA standing in the middle is called ~e middle part, IlIlcl the.

;.side DS and tJae anA'le DAB'8I!e the lU\i~nt ~arts, lie~ R x sin.

DA h' A' ,Ibn. DA,

= tan. DS X cot. DAS, and, t erefore, cot. D S = D8'~

. . tan" ,

ain. DA X coc. DS, c.nsequently ,

To log. cot. :DS,· 2" 34' 3" • . ll~

Add log'''no DJ\ ,.61 i 14 9.8l1~

1 to ~ ~ ,.' • I "

Sum=l~cot.DAS 3 5414 .. ;ro~. Ob -. Ec. 23 ~ 40

Sum = angie BAS ~ 21 54" r , ,

Hence AS and BAs' are now known, to fim{ AB and BS. '

: First to find::.MJ::.1n $his (!Rife the ~ are COIlIleeted'$- ther~re BAS is the middle part,. and AB, ana AS are the ad~ent parts,

whence . , " ., ' ',' ',," "

, cos. BAS

R x cos. BAS = tan. AB X cot. AS, or tan; - AB = cot. AS-' ~d

~. AD = eos; BAS X tan. AS, hence

To log. cos. 'BAS 27° 21' 54" 9.948460

: Add'log. tan. AS 41 9 II fl.~1505

, Sum = log. tan. AB 37Q 49' 5" 9.889965

Or in time R. A. 2h 31"'16'.3-.

" 'T6 ftml _ .. cb&ugle BAS -and aUie ASian! ~ <4II1d .JIB- is 'tti';oined,whenee R x. am. :OS::tt siD. AS X mIL BAS;, 0« tiBce ~ • . UneI1 ohhe;idea arft,p1'opoecioDal to the sines of their- ~lil:e an ...

giesl", ,: . ", ;'.. ':"

. As sine ABS or radius ]9'.000000

'1 tIs ~ sifteiA8 "'1° . 9i 1" 9.8.18274 ,

J (~,,, 6i1uJ BA:8 'ill' 1U '6,(, ;. ~~

• I I I .,'

" To sine Dec. BS 1:7 36 26 N. li48Q708

0)' The fof~oing' ~eiliod il;' general and I1pplieablettnuty J*tt!ofthe ecliptic, l,l:rofidM l'itoper .attention he pm' to th~ 'IIitd~on 4)f--the '~le!lthtl o~ect'wlth re"ect' to I:heec1iptic'and equllt(iti) ,:AlJ'thisopro.bIem~d its converse rs of frequerrt occurrence inpramtAl' ~

, > fXl:y. hiles and. forrrrulee, andeven tables, h .. 'e·b~ttl femted , .. the , .p. ut'Jl0~e of f1Jcilit~tillg'the com~litations. The roHO'ftfnf:.Ns;tR"~

By ttre ldf:e Dr M'llskelyne,wi11 be found vet1"'~ve t· fOr thIS

putpo!e. . . ,) ',f) 11, I

.,PROBLEM II. '

0' " .', Ift·'r·'l ... I", I • .' .1 " '.. Ir• I • • I , ~ U} ,', " ,

./' ~i,:en flie' l'igbt' asC'tln~ort; tb'~'d2e~in'ation.'and the ~ritr'~f.Wr

ecliptiC. to: Ilnd,the.Joognude·lUtd.latitlldew . . , .. , " • -

, DigltizedbyGoogle

SPHERI9A1o T:IIOONOMETllY .

. Let.RA denote the right ~ 0 theobUquit1 or the ediptic,

_I),~ 4ecUnation. " .

': TIJ). . .P--.in..ltA = tan. A, North or South as the declination i..

Cell 0 in the u,t eix ligns of RA South or S. and in the last six .

ilgnB North or N. .

Then A+O = B, regard beiug bad to the. alFwaic signs,. A being less than45°, and using logarithml.

See. A+coa. B+tan. RA = tan. Ion. of the lllUUe kind as RA. unless B be more than 00", when the quantity found of'the 88.IIl8 kind as RA must be taken from twelve signs.

. A being more than 45°.

. TNl- A + cosec. A + eoe, B+tan. RA = tan. Ion. or tlie iame kind as'RA, unless B be Dlore than W, when the quautiiy (QUnd ofdae ..me kind as itA must be taken from twelve signa.

, ~~. being nearer In. and IX. signs than 0 ami VI. aipw. 5bi. Ion, + tan. B = tan. lat. of the same lUUDe al B.

r Lon: nearer 0 and VI. signs, than In ... d I'X. sip.

,Tan. Lon. + COl. Ion. + tan. B =tan.1at. of the IIIUUe name lUI B.

ELUlPLB.'

. On Monday the 12th of June, 1826, the moon's R A at noon, was found by observation to be 1(1' 39'" 31· and her decHnation ~

51' 58" N. j required ber longitude and latitude? .

D= 2" 51' 58" N.' tan. 8.699&38

~1()1i 30'" 31" sine 9.536560 tan. 9.563008

A SO 16' 50" N. tan. 9.162973 sec. 0.004G51

0' 23 Z1 40 S.

'.

9.984575 tan. 9.433407':

11 15 10 50 S. COl.

--

LoB.l60 20 17 tat. 5 12 59 S.

tan.

9.553034 sine 9.5B46 tan. 8.960443'

PROBLE. III.

Given tit. longitlld. and latitude of & celeatial object;. and the ob-

liquity of the ecJlipti.; to find the right ascension .00 dec1i.DatiGD.

Tau.lA.t_.m. Loo.=tan. A, NoRa or So.nh .. U1elatitude is. Call 0 North in the six first sipi, and s..tb in the six lastsigns. A+O=B, as before.

A being Iese than 4,50, see. A+cos. B + tan. Ion. =T.an. RAafthe iIame kind as the longitude, unless B be more than 000, when the ~ found of the 8UDe kind as the longitude m:ust be subtracted

ftrOm tWelve sips. . "

.A being more than 45°, tan. A+cosecant A+cos. B+tan. Ion. = tan. RA of the same kind as the longitude, unless B be more than AID" J .... hen the quantity fo1lJld of the same kind as the longitude must be subtracted from twelve signs.

. ' If RA. be. Qeal'er, Ill. signs and IX. signs, than 0 and VI. sigrui, ., aitJ. ~+ao. B::c taB. Dec. of the same name as B.'

-: " And.BA being ~er 0 and VI. signs, than III and IX. signs, tan. RA + co&. RA+ tan. B = tan. Dec. of the same name .,. ~ .•

• These rulet tna1, in pnetal, be depended iapop. except in peeuliar eireumstaJlces, ", Wblc;b .. ~D of the Jfgiue will enable tJie co1Dputer UI correct, as when die Ion.

gitude or'llA~ r.n upon PP', or pp', &c. ' ,

See lh Abram Roberta;on's Jlllper in the fldl. TT.ns. flit I8H, JIII'Ie 138, yhlck fer

_t oCmom caDJlj)t be liven bi!l'8. . .. Google

Digitized by .

INTRODl1CTION,

.."., • 4 ~ • • BxAMPi..L

On_the 18\ of Janual'J~ 1820 .. th~ mea1;llongitud'e at ihe'~tar'P9-': matflaiit W&SU" t" 10' 34" .. the mean 1iltitude-2Ie 6' 45" S.; reqUired

~e ~~~~846~,8~on and declina~fon, the obliquity ~t the, e~Ptfc ~ •.

~t. 210 6' 45" 8. tan. 9.586721 .'

Lon. 331 19 34 sine 9.681082 tan.9.73790l

..... ' .

: A= 38 49 26 S. tan. 9.905639 sec. 0.10842()

0=23 ~ 46 S. "

: ~=f:'tyJ 17 12' s, cosine 9:667498 tan. '10,.219185,

RA::S4l 65 l~ tangent 9.513819 sine 9.~183r

Dec. 30 34 21 S. tan, '9.771~6

.' , ~a7ltf1lu for ~.rerci8" '.

1. The mean longitude of. Arietis, on the 1st January, 1820, was l' 5° 8' 48", aad mean.1atitude 9° 57' 34" N. when the obliquity of the ecliptic was 23° !¥I' 46" ; what was the right ascension and decli-

nation? '

An.r.-R. A. 11> 5'r 3'; Dec. 2~ 36' 24" N. _ .

2. Required the right ascension and declination of Pollux, when the longitude was 3"200 43' 5~", th~ latitude 6° 40' 17'" N\ the ob-

liquity of the ecliptic being 230 27riJE1If . -." - ,'- - - I

An8.-R. A . .,. 34'" 17.5'; declination 28° 27' 8N ?S" -,

- 3. 'The mean longitude of Spica Virginis is 6' 21" 19' 50", latitude

2s> 9' 24" $. and the obliquity of the ecliptic 23° '21"t6',.; .required

the right ascension and declination?' , , , , '

: An.r.-R.,A. 13h 15m 43.5'; declination 100 13:' 4" S. . _ _

4. The mean right ascension of" Aquilre is 19b 42m, and"itecIinatWm 8° 24' 4" N. the obliquity of the ecliptic being 23° 27' ,46" ; ,r~

quired the longitude and latitude? " . - .,' " .

. An8.-Longi~ude 9' ~9 14' 14"; Latitude 29° 18' 36" N.

• ~. Required the longitude and latitude of Ii "Pegasi,' or which the right ascension is .. 55'" 48", declination 14° ]4'21", the ohliq,ujij

of the ecliptic being 2S" ~, 4&' ? .

. ' An.t.-Longitude n' 20" 6W~H, Latitude 190 24' 88" N. '

• • • • " PaoBLElIl IV. " ' .

. Given the latitude of theplace, and the sun's declination, tofirid his altitude and azimuth at ~ o'clock. ',' ; , 1. At Edinburgh, in latitude 55° 57' 20" N. on the 21st or ,.ru~, 18Si6. the sun's declination was 23° 27' 36" N.; required bis altitude pnd azimuth at 6 o'clock in the morning or evening, his declinaeion

being supposed to remain the same. . -::

· Conalruction.-Describe the primitive HPON on the plane of'the

meridiau. Let no represent the horison, ZN' z .

'the prime vertical at right angles to the former. P ,

Make OP .-from a scale of chords equal to the Ia'titude of the pl,,;ce, ~ort~ in thepresentinstence ; , l' draw PP', the SIX 0 clock hour dtcle in this case, 1I:!t:~~.~~'J:: and at right angles to it draw toe equator EQ; . 'describe the small circle 11m at the distance of

23° 27' 36" from the eql1ator,'representiilg the

'parallel of decImatioh, and it will cut th~ six ~ J)II

o'clock hour circle PP' in F, the sun's pla_liteizatb,t:heC'g.t~ktinre.

SPHERlCAL TIUOO~ETRY.

ThJ'ough Z. F. and N dMribe tbe azipluth cir.cle_ZFN c.tti.JIg the hod.on'in t)..z._!hen;?iS:b the altitll4e, .J!Z,~e' .eDit~ distance, and tile mgle Fz.t', or-lU meaaure, the arc DO, IS the asimuth ] eonsequently, the thing!l given and req1lited talUo eit1ier of the triangles FZP. 01' FDA. which ..... supp}.eaenW \Deach cather. For, .mee OP i& the latitude, PZ is the co18titude, AP 'is the declinadon; ,Consequently,' FP i. ~ polar distance, DF being the .Ititnde, FZ muit lit the .enith distanee.' , .', _. . Calculalion.-In the rigbt-aJigled apherical. triana'le FP~ righ~ aDs!ed atP, FP and PZ are given, to fiDd the angle jlizp and FZ; qt in tbe. triangle ADF. ridIt-angied at D, there are. given the aape lIAD, ~ual to the IatituCle of the place" and AF, the sun's tIeeliaa.. _, •• find DF, the altitude, and the side AD the .. imatb~ .

, By the rule of the circulal' parta FP, PZ, and PZF, aft aU' eo~ ~ therefore PZ is the middle pet" and PZF _d l'F Ret u..

adjacent parta, where - -

_ - R X sine ZP = tar. PF X cw. PmJ', or

•. ' ' B X C08. lat. = 008 dec. X CM. azimuth, therefore

.' COLI.l.

eoI. GZlJIlutb = d - cos. lat. X tan. dec.

, CDII. ec.

To log. cos. lat. 55° 57' 00"

Add log. _tan. -dee. sa 27 M

Sum = log. cos. az. 76 10 38 " . 9.385533

, A.piD, .to- find PZ th~ coaltitude, the aame -tbinas bcin_g given. 1t X C08~ FZ = cos. zp X cos. FP, or sine alt. = sine lat. X sme dec. -

, To lo&-,..m.e!at. 05° 57' 20" 9.918347

I •• -Add fog. sine dec. 23 2'1 36 g.OOOOO~"

.. ~~ ~_log. sine all. 19 15 _ 4p : PBOlI~V.

- Ginn the latitude ~f the plaee, and the 8UO'S d~Ji~tion, to find the altitude ~d hour wW!n ~~ sun i. du,e E~lt ~. Welt.

9.518.149 -~

" -4~ ~dinburgh, on the 21st J,une, 1826,'wbat ~a. ~e su~·s altiiu4e .ad hour when due East Or West,. the declination ~mg -23:' 'fl1'3fS" N. _ ' - - . - . .. -'

, lh the last fLzure, let ZAN meet the parallel n m in K, ana s~

pose a circle to be drawn through the points PKP; fonning the tri~le ZKP. right-angled at Z, then ZK is the ccaltitude, and Z~lt the hour from noon; hence " , - Rxcos. PK = cos. Zp x cos. ZK~ or- '. " - ..•.

cOs. PK ' .

"Cf)S. ZK = ZP = cos. PK )('1Jee. PO, or . . - cos.

,_ &ine alt. :::; sine dec-x sec. lat. .

, Dec. 23° 91'/' 36" sine 9.6OOOQ2

Lat. 55 57 20 sec. 008] 653.

, ~ ': - ~J!--' ,'_Alt." ~ :-~ J,Ss "eine .9.68J.6M

. -- .. R X eee. ZPK = tan. ZP)( eos, PK, 6l" .

.v , ,',:_: " ..... cps. '1' =009. lat. X tan. dec. - Digitized by Google,

INTRODUCTION.

'L8t.' 56· 5'/' 26" cos. 9.899114 . Dec. ss: ~ 38 tan. 9'.637472'

Time 4b' 51" 48' cos. 9.467186

From noon, that is. at r 8'" 12' A •••• and 4b 51" 48" P •••

. This problem is of considerable utility to the navigator arid prae,;. tical aatronomer.fOr the purpose of determining time accurately when an altitude instrument is used. As the change of altitude, OIl whiCh the accuracy of the deterinination' of the time depends, is \uickest when the objeet is on the prime vertical, the most proper time for observing an altitude for that purpose is, therefore, wJien the object is due East .or West, as any BD_Iall error in the observation has then the least possible effect' on the time. Other errors at'e also in this' case in a great degree avoided, or at least considerably lessened. particularly that arilling from any small error in the estimated latitude at the time of observation. To facilitate its application, tables, cprresponding to the latitude and declination (which must be of the same name with the latitude), have been given in books on Nautical Astronomy, such as those of Mendoza Rios, Mackay, and Lax. When the latitude and declination are of different names, the altitude must be as near the horizon as is consistent with accuracy. so far 811' depends upon the uncertainty of the horizontal ref\'action. Altitudee '\lnder 5° Should not be used when great accUracy is required.

PROBLEH VI. ,

Given the latitude ot the place and the sun's declination, req.ir~

his amplitude and ascensional difference. • .' . ' .

At Edinburgh, on the 21st ofJune, 1826, from the data given, on

"hat point, and at what time, did the sun rise and set ? .

In the triangle ABC, in the last figure. there are given the ,angle BAC, equal to the colatitude, and BC the sun's declination; to find ACandAB.

R X sine BC = sine AC X sine BAC, or

• A£ sine BC • BC BAC

sine = sine BAC = sine X cosec.' • ' )

DC, or dee. 23° 27' 36" N. sine 9.600002

Latitude, 55 57 20 sec. 10.251939

AC, 45 19 33 aine 9.851941.

. CO, 44 40 ~, in which case AC is the ~1i-

tude zeekened from the East or West, to the North and South# ~:. cording to th~ name of the declination, and,CO is that reckoned froni t'lae meridian, or from the North or South, according to the nanie of

the declination. . .

Again, in the same triangle AD is the aacensional difference, and R X sine AB = cot. BAC X tan. BC, or sine AB = tan. lat. X tan. dec.

Lat. 55° 57' 20" tangent, 10.170286

..pec. 23 27 36 tangent, 9.637472

----

A. D. 21' 39'" 52" sine . . 9.807758

6

·8 39 62 = time or setting.

S 00 8 = time··or rising. the latitude and de.

• By the ascensional dureience is meant the time before or after~41 o'clock the IUD riles or seta. By this problem, therefore, the ltmgths of *z~ly~'" ~ __ - lIined, and the uri.tion of the mariner', compass.

13

clination being of ~,~ name,~, i£instead of sine we read eoeine, then we would gef t11e UJQe of ri8i~g M' the Jatit~ and declination are of the same name, and the time of' setting it of different Ilamea. Thi:;! however, is ou1y tJie~pproximate time, as no allowance is made

. for the effects ofra change of declinuticn, the horizontal refraction and plll'iiUal( in the case of the sun ,and planets. _Fqr tlie'l>esee Ma ai' on the longitude, or they may be found by the following m'te.. Bt'. let the appl'O~inH,te time b. e fI.~und., . T? this time let ilie liee· . bon of the object be reduced. Wlth It frnd the ns-

ri ion!11diffe!i'ence as fo.rmerly. Now~ :lind t~e~Lml a'ml llifference of'the natural eosine of the reduced declinatlOn and natural ine of the'latitude, which may be carried to £01.11" places of figjJ!'e only~ .these being lJuffici~ntly accurate for this pllrpQse, and take half tbe sum ot'the logarithms of these quantities, to which add the con-

ant 10garit1lm 7.1761, and the proportional logarithm of tbe dllferehce between the horiaontal parallax and the sum of the horizontal . r fh" . u: .and dip oftbe horison, the sum, l'eJectit~g ~o in the Index, "WiJ). e't"he propol'tional1ogarithm of the correction which. is td be 8"14 tracte;Z from the time of rising, 01' {l(ld~d to the time of setting:. if the 'horizontal J_JaralJ.ax is less than the sum of hor-isoutal refnlCtioll .• 'Uu:i dl_P, otherwise tbe correction must he ridded in the first case ... and

,'UJ'ractlid in the second. '

• , ~MMJ.>LJl:. . .. • ... ,:.

Required the time' of ris~ ~d set\,i.ng of 'the sun on the 1st ~f Ap\'il, 1826, in latitude 33d 42" tq.~ and longitude 16° 20' W, . the

k'i8M en the eye, above the sea, being ~8 feet. ... r, ;, I ,.. ".

Dec. 4° is' N. COlI. 9900 .. , " " .,,:'

.Iv ,.' .:,' f i, ,J •. Lat. 83 42 N. sine ·5MB .' , .J. ..

l'

Dip to 28 feet ~ 5' 16" Hor. ,refr~. ,~ 1,7 ' Parallax" + "9

~. 15517 'log. 4,]900 .. Ditr.4421 log. 3.64~ ,

-

7.8363

.': (1

"r

con st. lo~. : ~39 24 P.L,

'3.9181 7.d761 0..0098

':" . .'_ 3"'10' P.L .. -. •. 1:"7540

"The tNJItreciAAt to- be sulltrKted irOl'll tL. time of .hsihg, or ad4led b)the~tl~ ohetting. As the moen's horiiontal pa1"alwi.i.an·~ .. \.o~e&cer tMn the effects of di" and refrlletioD, .. the:~oft!eOti_ thu "'tJlIiliM'W6akl hav8' ·beoen applied "ltba eontl'llry sign. Thi. IIUIoI thod of determining time may sometimes be of use 'wilen a betUIt _dC·b.:o1Jtai~ed,.·and-in the (:88",01 UIe·llun. fJI' moon,'·a.~ of t~ \iwe~f. a.l>J"!~,ance of, the "Pper IIhd lower limb may be>tak"

So_Jw;.an..qJ' Oblique.Angled SpherWal Tria •••

The differl!tlt eases of oblique-engled IIpherical a1tmg18s may be

solved by th~( '~ing theorems:- ,

. .~ '.' .' ,_ ,

. .

• To find the rising and Betting of a star or plan~e transit over the meridian mUlt be fint computed at! fOllows :-From l\. A. of the liar lulSVliIlt· Q!at of the 111ft fer· noon, the remainder is the approxiJDllle ~tne tIf ~. Redu,*tbe B. A. of both to this time 1UItt~~tQdll;~d.~",.~,~the ~ai~~r will be the true time o£~.lW:b. -propq apW~ to t!'j: . .senUw.QJ1UIl ~C;, wjUKt.!~J. .when ~on:~~ed for

c\i~ &11., iIu::true time II rbmg or setting" ., -, ' ... ' G r

' , •• H Digitized by '(®Pg e

14

INTRODUCTION.

TBEoa. I.

; In every spherical triangle A.

· the sines of the sides are pro- ~

· portional to the sines of the

ang~ op~site ~ them,·. :B D

· or, SID. AB : SID. AC:: sm. C

C: sin. B.

~c

D n

THEOREII II.

In oblique-angled spherical triangles a perpendicular arc being · drawn from any of the angles upon the opposite side, the cosines of .. the angles !It the base are proportional to ~e sines of the segments of the vertical angle, or cos. B: cos. C: : sm. BAD : SID. CAD. .

THEOD-EII III.

, The same things remaining, the cosines of the sides are proportional to the cosines of the segments of the base, or cos. A-n: cos. AC: : cos. BD : cos. CD.

THEOEEM IV.

The same construction remaining, the sines of the segments of the base are reci~rocally p~oportional to the tangents of the angles at the base, or sm. BD : sm. CD: : tan. C : tan. B.

THEOREII V.

The same construction remaining, the cosines of the segments of the vertical angles are reciprocally proportional to the tangents of the sides, or cos. BAD: cos. CAD: : tan. AC : tan. AB.

1'BlMmBIl VI.

If, from an angle of a spherical triangle, there be drawn a perpen_ dicular to the opposite side or base, the tangent of half the sum of the segments of the base is to the tangent ot"half the sum of the two sides of the triangle, as the tangent of half the difference of those sides to the tangent of half the difference of the segments of the base, or tan. ! (BD + CD) : tan. l (AB + BC) : : tan.! {AD en AC) : tan. i

(BDrIlCD). .

When the three sides or the three angles are not the given parts of the triangle, to have sufficient data for the solution of the problem, the peryendicular must be so drawn, that two of the given things in the oblique-angled triangle may be known in one of the resulting right-angled triangles.

THEORElIl VU.

If a perpendicular be drawn from an angle of a spherical triangle, to the opposite side or base, the sine of the sum of the angles at the base i8 to the sine of their difference, as the tangent of half. the base is to the tangent of half the difference of its segments: And the sine of the sum of the two sides is to the sille of their difference, as the cotangent of half the angle contained by the sides is to the tangent

• See Playt'air's Geometry. article Spherical TrigoIIometry, Prop. XXIV., or Legen_ dre .. Geometry, article LXXVI., and" the followlr.g in order.

Digitized by Google

SPHERICAL TRIGONOMETRY.

75

of half' the dift'erenee of the angles which the same sides make with ' tlte perpendicular,· or sin. (B+C): sin. (B en C): : tan. ~ BO: tan. t: (BD(J)CD). And sin. (AB+AC): sin. (AB(/.)AC): :cot.IA:tIln; . (UDCb CAD). -

TaBoBElII VIII.

The sine of hall the sum of any two angles of a spherical trilUlfSle, is to the sine of halt their difference, 88 the tangent of hall the SIde adjacent to these angles, is to the ~ent of lialt' the difference of the' sides oppotrite to them. And the cosine of half the sum of the IIB1Ile angles, is to the cosine of half their difference, as the tangent of half the .aide adjacent to them, is to the tangent of halt' the sum of the sides Oprosite, or sin. i1'A+ B): sin •• (A ill B) :: tan.! AB: tan. l (Be ell AC). And cos. i (A + B) : cos. i (A III B) : : tan. i AB : tan. ~

{Be (J) AC). . .

C~-The sine of half the 81UIl of any two lides of a spherieal triangle, is to the sine of hall their difFerence, 88 the co4IIngent of half the angle contained between them, is to the tangent of half the difference of the aDfIles opposite to them: And the. cosine of half the sum of these. sides 18 to the cosine of half their difference, 88 the cotangent of half the angle contained between them, is to theIlmgmt of half the sum of the angles opposite to them, t or sin, i (AB+AC) : sin. t (AB (/.) BC) : : cot, t A : tan. l (B \I) C) COlI. I {AD X AC): COB. ·HAB II) BC) : cot. I A: tan. I (B+ C).

TaBoBBI[ IX.

~t will be sometimes more e88y in practice to compute an angle from the three given sides by the following formula! and rules, than by any of those already gi,:en: thus,. suppose A, B, C, are the angles 88 before, and a, b, c, the sides ~ ; then

fiIiJl. i A = j sin. { I (a+b+c~-c} : sin. { l (a+b+c)-b } (I)

SlD. b SlD.C .

Cos. i A =J sin·l (a+h+c.> sin .. f I (a+b+c)-a 1 (2)

; SID. b SlD. C

~ 1A_jsin. {t (a+b+c)-b 1· sin. f l (a+b+c)-c} (3) an. ~ - _ sin. { t (a+b+c)--a } . sin. { i (a+b+c) }

Rilles in Words.

1. From half the sum of the three sides subtract each of the two siCles which contain the required angle. Then to the OOIIeCautS of tlIe sides which contain the required angle add the sines of the two remainders; half the sum of these foregoing logarithms will be the sine of half the required angle. ,

II. Find the difference between half the sum of the three sides, aDd the side opposite the required angle. Then to the cgeecan" of the two containmg sides add the sines of the half sum III1d difference ; ~f the sum of these four logarithms will be the coeWe of half the·

riuired angle. .

II. To the, cosecant of half the sum of the three sides add the

• Thla theorem form. Propoeitioa XXX. in Planair's Spherieal T~y, "here it is partly erroneous. It Is also given in Mr J. Wallace's edition of Brown's ~thmic Tables. Erroneous rules and impOl8ible triangles should &1waY8, ifp06SibJe; be lWOideid.~ the French Edition of C&gnoli's Trigonometry, § 1088, lUlU and nes,

t Legendre, § LXXXIII. Digitized by Google

IN'l'BOJ)UCTION.

'led",.9W1' ~~~ 8I&IIlf'l~Jliah~~ tJa.1lt'~ed \he~:\. _ _;

4Wg18J iWU w..IIJ ~ 0 _.-;ne .. QiIoU I1lm ·lJJIWI _ .

the side, containing the required angle i half the aum of fbw .

logarithms will be the tangent of half the required angle;' S. xe.. marks annexed to Cue UI •• Plane TrigollQmetry.

I . THEOREM :x. .

.; Given two sides and the contained angle, to find the side oPJMliite, that angle.

To twice the sine of half' the contained angle, add. the &ineI of th& two containing sides. and from half the sum of these three lofJu:ithms subtraCt the sme of balt the difference of the sideri 7 the remainder will be the ~ngent of an arcI the sine of wltich. being ~, t,racted from the halt sum of the three logarithms akea4, f01Ml'd~ leaves the Me of half the required side.

TJo:o ... XI. . I

t/ 'fte two sid .. and contained angle being given, tlie tflUod side w.ay

be tbund in the (ollowing manner, .

To twice the sine of h8lf the contaiued angle add the sines or the two containing sides; hall the sum of these three lo~thms, after J'e~ ~g 20 in the index, will be the eosine of an arc. Also find half' the difFerence of the two containing sides -.

To the sine of the limn of these two last arcs add the sloe of'theix difference j half the sum of these two logarithms will be the cosine ot

half the required side.. '.

It may be remarked, that when the side is not gl'ea~ .~ ~ .. theorem X. may be used; when it is greater than 90". theotQl.Xl.

may be employed when great accuracy ia required. .

T ...... XII.

The three angles of a spherical triangie being giV8n1 W And .tbd sides.

From half the sum of the three angles 81lbtract: eaclt of the analets next. the req,uired side, then to the cosecants of the adjacent' angIes add the. ~ of the twc' reroaindel"8; half the SUlll of these fow logarithms will be the COBine of half the required side.

TII1IOBml XIII .

. The same things being given; from half the sum of the' thtee angies sUMraet: the angle opposite the required side, then to the eMecaDts or tile ad'aeent angfes add the cosine of half the sum and the eosine of the difference; half the sum of these four logarith~s will be the cosine of half the required side.

Either of these theorems may be employed, which wiD give the more aoourate result.

Having stated the theorems on which the solutions in ob1i'}Ue-arl~ . gled spherical triangles depend, it is necessary to illustrate them by examples which will chiefly consist of those apflicable to the usual eases that occur in praetieal astronomy and navIgation.

PROBLEM I.

Ginm the latitude of the place, the sun's altitude and declination. to find the time and the azimuth.

At the observatory of Edinburgh, ~ the Calton-hill, in latitude 55e 57' 21" N., on the third of June, 1826, tile followwg. observaDigitized by Google

SPHEJIICU, 'r&l4Jft!lOMETRY.

'If.

tioaa ~.~ .'.a t.or.r u.b wj!~ blieD in the JIlOI'bing ;' uquirtd tile time and uimuth, the barometer. being at 29.56 in., atld 'Che

~at(U°F.,? (. .

X_ b!J Watck. .Altitudes. '

r 1m 20' . . '. "1 ._- 26" 51' 20"

2 18 26 59 30

3 25 _', . 27 7 15

4 30 27 15 40

D 27 272345

..!J. t -11 0

Meua.- .-'/ . ~ 24

0> ~tl!d z.n. , z. D. 6~ 52'.5 log. 3 , Thermometer 640 F. log. Barometer 29.56 Thermometer d4.0 F.

r = 106".5 = I' 46".5 log. Z. dist. = ~ 52' 30" ReUactiou + 1 46 .5

35 37 30

27 7 30 Lower limb. 62 52 30

. :._ .-.... 2.03692

9JJ87il 9.99358 9.99940

True Z. n, 62 54 16 .5 of the lower limb. Semidiameter - 15 47 .5 .

True Z. D. 62 38 29 01 the centre. Approximate time, June 2d, 19b 4m Longitude in time add-,-+ -12 West.

._ated (heenwich time 19 16 D • .L. ruJN08,

Daily variation of dec. 7~ (9" ,P. L. 1.36878

Prop. part. to 1']1' 18'" + 6,11 P. L. 1.46a8J

Dec., ·June 2d, 2i'> 9 38 N.

Reduced declination ~ -1& 49 ·N.

Polar distance 61 . '" 11 . .

L, Now in the figure, (page'1O), dtere- are given OP the latitude, ani eo~ent1y Z P the colatitude, PK the polar distance, and ZIC t1\e zenith distance, the place .of the sun being K De8l' the prime vertical, as being most advantageous to determine the time with aceuracy, or the three sides of the triangle KPL; to find the angle ZIJIC. the time, and the angle PZK the azimuth from the so1I&heni meridian PEP'. This, tllerefbre, is solved by means of theorem IX.

Digitized by Google

INTBODt1CI'ION •.

. Now the latitade beiag 550 57' ~1", the colatitude is 340 ~ 39"

Z.ll. 1M 38 ~

Colatitude 34 ~ 39 cosec. 0.251942

Polar dist. 67 44 II cosec. 0.003647

Sum 164 35 19
Half . 8i 12 39
First rem. 48 10 0 sine 9·872208
Second rem. 14 28 28 sine 9·397850
19.690647
ga CJr 2" •• :
2 : .aine 9·77789{
Time &om noon 3d 454 ti
1~
~ time, A. lII. 7 5 18
.by watch 7 3 24
Watch slow 1 54 for apparent time.
Again app. time 7 5 18
Equation of time 2 23
Mean time 7 ~ 55
Time by watch 7 3 24 Watcb fast . ~ for mean time.

~. To find the asimuth or the angle KZP, the point K beiDg tJaat in which the circles" .. and ZINcut each other, there are given 'the

three sides of the triangle KPZ. -

KP, or polar dist. 610 44' lI"

PZ, or colatitude 34 2 39 cosec.

ZK, or Z. diat. 62 38 ~ cosec.

0.25]949 0.051515

Sum 164 25 19
Half 82 12 39 sine 9.996974
DifFEftDCe 14 28 28 sine 9·391850
19.697281
46 741 cos. 9.8486*)
2
N.90 15 22 E.
44 5t 19 sin. or ..
2 8. 89 44 38 B. or reckened &eln the

fbItIr.in nGl'tb latitude, 01' &om. the North in !lOath latitude. . ...

This pMlem is very "setal ill na'figation, fur the p1Irpoee of'fiild~' iog the variation of tbe aIIDp&IB, wbich is the dil"eftnee betweeft't!le'

true and observed amplitude or azimuth. . .. Google

Digitized by

To determine this, let tbe- ebeerNr be ~ to look directly

· fr.om the c:ent~ of the card towards the ~nt representing the true azimuth ; then if the observed azimuth JS to the lift of the true uimuth,. the variation is ealterlfj, but if to the right it is westerly to tile

· amount of the difference between them.

Thus let the true azimuth be S. 890 44' 38" E.

~ed $~400

Variation. • • 24 20 0 West.

Or about 21 ~ts westerly.

. 'nlese results for time and nriation have been deduced strictly from the iiolution of the spherical triangle fonned by the data, but

· they may be found more readily by rules derived from it, as may be 8ee1i'in various books on navigation and nautical astronomy.

When tables which have proportional parts annexed to them are used, the following method may beadvantageoualy employed for detennining the time.

Rule.- When the latitude of the place and the declination are of the lame name, let their dijference, but, if of contrary names, let their sum, be taken. Under tliis difference or sum place the zenith distance, and let the half sum and h~lf difference of these be taken j then ad d together the secant of the latitude, the secant of the declination, the sine of the half sum, and the sine of the half difference j half the B1lDl of these four logarithms will be the sine of half the hour angle or time from noon, from which the apparent and mean time may be obtained as formerly ..

. Latitude 550 57' 21" N.

Declination 22 15 49 N.

Ditference 33 41 3SI Zenith dist. 62 38 29

secant . secant

Sum ·96 20 l·half48" 10' Ol"

Difference 28 56 57 half 14 ·28 2Bt

sine sine

1 9.872ia08 9.391891 233

!It i'/m 20'

1.05 p. P.

2 27 2J.05

2

sine

-

45 43

4,54 42 .10 24

June 2d, 19 £) ]7.90 P.M.,

, In the abovecompntation the several proportional parts are let down and sammed all together, wlaiOa renders the operatioD 1IOIIMtwJlat.-e ..., wbea OUI'~_ •• emplo~.

Several val'iatiOll8 may be made eo. the IlX daiogs'ben POP_IIII,,' ~ _y aonoe." a' aIef.l ... -'wlUda, by a 1 .. ..w to the ~ and .... ....,._'giftD, will be_Iy peri'onIIId.

Digitized by Google

'.80

'J:t ~. _,) ,., i. ~: .. Iff .. "J!\I" !1,.llaaIIItIIIrJI.· I'~ '" I,Ultlt1't;,,h, ... , ..

:', \ •• 'A.,;,. 6,,·(te or~ . ,L""~ <l~' ;: , '," .t~\iAA •.. ,." '''J:!,:.J ~ .. GIx~n ~ ~ tu ':L'f.,Jt~~,iW- ' ..• lJ!u.l~ .. ~" •. !H~f.t~,HPf'4

.).<~4\!,~e. ;'r~n.~~ilig,~.,!~gl~~~,e~ 1W;'. ~ " '=' ,'I. ,~, '." ~

Xt what time will twIlight be~M~!' "~t,,, ' ''6.~

51032' N., on the second ot:May, l827, s~!t .. ~ ,,~~, ~

15° 14' N.? , S'!'c It (~

In figure, (page,7Q). S)lPPose Ii parallel n m to the equatOr l':~ to be drawn ~ ~ distance oflS0 14' above it, while another,)MIIIillel to the'honz~nHO is ({fawn at the' distanCe of I&. below ii;''tltese ,~"'o woul~ ¢l!-t one. ago,th,r ~ew~e between f ~ ,t.!t/ip:~,Plug the, tqaugle ZPS~ m whiCh ~P.,'J.>S, and Z~ at~ 8JIl'e9,~~nd tbe?D,s:Ie ZP~, the ,angle between the m~idian. PEP and anoth"r mer",li~'pasSlllg through the sun at the time he 18 18° degrees below the,borizoll, hl~ situation wheJ;l twili&ht begins and ends.",,;J

Z ~ .q~ p:eni* distance 108° 0'

P 8 oi fbla.r. '. distance 7416, ., ,.. cosecant O.Ol~

PZ 'Or coIll,titutle 30 28 cosecant 0.206l68

swi ,.-: ", ,'f'
221 14
Halr' llO 37 sine 9.971256
D~ff'~.rt:nce, . 2 ~7. sine ~6S.!H75
18.852433
4h 58'" 6' cosine "9.42'62\6
2 448 Timl-! from noon Or at

9 56 12 in the eve ••.•. ; .' ,,'1 'Bl .2 4 48 in the mopti .. ". '" " .\":',

", : t • .' 1.'1

Giv~n the right AscensiOl!s and detii~ati~,pr iI\~lo~gitV4e;~imd latltudes of tw6 ~lestial obJect~ ;' to lind their angular dlstance. ,

In' this problem there are given two sides and the contained ~le to find its opposite side. The contained angle is the difference betweMl their right ascensions or longitudes, and the containing sides are*'emttplements of the declinations or latit\lde~. If. the s»n be on~.ef the· objects, I1S hill latitude.is very small, he may be sUllP.?Bed to bIlI"always in the ecliptic; then the 'triangle so formed Wi,ll ,be right angled if the longitudes and latitu.d~s are used, and the computation becomes more simple. By means of this problem th, 1qPA1'

distances in the nautical almanac are computed. .'

OR the 1st ot June, 1828, required t:b~'distanceDetween themoon and .. Pegasi, at noon, on the meridian 6fGreenwich, ~e moon'srigbt ascension being 295° 23' 46", and declination 16° '11' 4,5" S., the star's right ascension being 22h 5f)'" IS-'1m, or '3.«""3" 28"~ and north

polar distance 75° 43' 2", or -deelination 14° HY 58" N. '

344° 3' lB'~I95° 93' 46"=48° 39' 42" the mgle at the pole. lwttea~ however, tIS ~dut~dsiveddhJlllthe ~erical.triangle, a mor8s~e )JIRCtWnale ma"be ~,...u.ac-

~jo Uacar .... I!X.' " .. , .. '.!'. ' .1. ,.'ftIl"'.

,TQ,~~.t"ba sjpe.of ~ ~ ..... _ 081181. MId tile of

the QI()Oll' ud .. tAr:s,,~, .-d .. ~.wt' ....

Digitized byGoogle

81

.. :::idHu. J'rom tWa ball _ mbtract tIM aiDe of ball tile ,., .. of declinations if they are of cOfltra,,!/ fUJ.rN, or that of half their dif'erence if of tbellUlle Dame, the remainder will be the tan~ of an "arc, the sine of which being" subtracted from half the sam" of the thme logaritbms already loUd will give the aiDe of balr

the required distance. " "

DifF. of IL A. 48° 39' ~, " "

Half 24 19 51 sine xt=19.~
Moon's declination 16 11 45 S.cos. 9.982413
Star's deelination )4 )6 58 N.C08. 9.986964
_- 39.198581 "
Sum 302843
19.599291 (0)
HaIr 15 14 ~Il iline 9.419717
---
Arc 56 31 18 tan. 10.179874
Same arc sine 9'921216 (b)
Half distance 282729 sine 9.678076 (a-6)
2
True distance 565458
&a .. ple. for Ezerci.e. 1. Required the distance between the moon and sun en luly 2d. 1828, at noon on the meridian of Greenwich, the 10llfPtude of the SUD being 3' 10" 28' ,"', the longitude of the moon n- 17° 59' 39",

and latitude sao 51' 40" N. ? " "

.A,...-11sao fJ!1' 19" east of her.

" 2. Reqnired the distance between the moon and IIIR on the 20th January, 1828, at noon, the SUD'S longitude being 9' 29°2ir 39", that of the moon n- ] 7° 54' 42", and latitude 3° 24' 28" ?

3. Reqwred the distance between the mOOD aod .. A.t--. at noon on the 10th of May, 1828. the right ascension or &be moOa being 6" 56' 43", the declination 4° 44' 48" N., the right asoebsion Of .. Aquille in time, being 191' 42"'25".62, and north .polar dild:allee·81°

34' 41" r " :

.A" •• =1f1'" ~ 51" west of her .

.. "Required the distance between the mOOR and .AldebaIwI. at lllidnight on the 16th of December, the moon's R. A., being ~ 31' 30", die declination no 18' Il" N., the R. A. of Aldebaran being ~

~ 8".6'1. and N. P. D. 73° 50' 37".4? ,

..4tt8.-38° 21' 10". "

PROBLEM IV.

00 finding the latitude by observation.

The IIlG8t simple praetica1 meth9d of Gnding the latitude, i!l uom the meridian altitude of a celestial body whose declination is known.

I!AaeuW. die object be the 81111, moon, or some of the 3'1anets, the ............ cJiIIIaIIce of the Iftel' or upper Hmb~r both, are Digitized by GIDrc)gle

.82

d},~l ... U".. t .. Upp ...... tifol&eveal,oerrectialia ... ef!the

eentleislcJbt.ioed. ,., \ ", .. } \1'. j.1 • II ..... ' .11, .j, •.

. : Wbto reflecting instllalllents. nch as.tIle &eXtant, l'epeatiIig cim1e, &c.! With an .artificial horizon, are empl~ed. the arc; _~ 06 JDIlIt, . iWriUhe prinaples of optic., be halved lJemre the other,.omecti.oa.

are applied.· , . ..

A meridian altitude of the IUD,. DlOOn, or. a planet. .taketl> at land, mOBt: be corrected for refraction, parallax, and semidiameter, and at

sea foUhe.dip of the borizon.t . . . ;.

Having found the true altitudertake its complement to 90", which·gftet tile zenith distaJtce, denominated nonh or south, accord-

ing .... >ihe >observer is north or south of the object. . :

Now';~ zenith distance and declination are of the same name, their "'. iathe latitude; if of CfJIIIlrary names, their differenee is the

latitude-".tbe same name with the greater. . .

,Ez.IL~burgh Observatory, March 28th, 1825, with an artificial ~~OD and one of Troughton's best sextants, the verni.u- of which. ~ 10", Captain Pringle Stokes, R. N. found the meridian eltitucle"Gt'dte l!U1l'slower limb to be 73Q 32' 15", the index error being +2' 26", the barometer standing at 29.66 inches, and Fahrenheit's thermelRetel' 860 ; what was the latitude, employing the refractions in the.. table in the nauticel almanac?

Oblinetl altitude Index eriw

8um

.7W 3-2f Hi" + 2 26

73' 34 ~.

Half . . ;

RefnlCtion to 29.66 and S6<> F.

Parallax - . .

Semiiliameter \'

True altitude

36 4f'rs

+. ·8 "+ 16.2

31 215

.. ----

\ Zenit1l .iltanee ,Declination

52 57 46 N. 2 59 43 N.

. J ..... e 0" .. ' • • • : 55 51 20-

J "i_To deterlXliM from the observa~oDl of Captain BasilHall, ,& tN., taken.J une 4th and 6th. 182!1, the latitude 0( &at'. Blail, . that by eatim&tioo beiogabout 21° 39f N~ IBId .odgitudel~o 15',,*. =

1" }m in time. . f ,~ .

To compute the sun's dec1ination,.lune .t1i.·1 .... 1\,',

~thi1e in time 'r 1- . D. L.' i .,o.RMt8

'. !WI,.. iuiatlon' 6' 56" , .. P. L. l.41433

&op. part to 1" }m 2' 1".6 P. L. l.~l

Eq. to sec. difF.-23" and 7" + 2.4

Correct prop. part • 2 4.0

Declination at noon G. 22" 24 {l .0

SIUJ'S true dec,

22 26 45 .O-N.

r .

• "·~1tPadon ofTiIbJe xxv. J • ., '.. " •

..... NJei mil. and XIV. have ben eGIIlJ!lltecl, ft1'M11111 far. ... pupIIiIe. '" _

combiniDg the whole ill one.. . G I '

• Digitized by' oog e

SPHEB.U:ll ftJGOlIOIIETRY.



To ~ the ~efraedon. the barometer bein" at, .,. ~. and tile. tiMm6tlei£,. 86° Fahrenheit, to merid. alto L I •.. JiB'': :Dr

,.Z.~. ',:_"~:h""'_ 1"10' log. , =

'ther. ~ tab. . .

Bar. 29.7&' .,. . 9.9003

Ther. Sf ! " '. ~

r 1".1 .

, ,Pandlu 0'~.2' (table 16) 'I ,

. ,.- ,~ .. Pace Of the eirde '"8t.

Readings {~t V emier ~:- . .

. t ., (" r'

86".&0'- .(XI '" '

": ,oo,·lQ,

(Jbs: mend. alt. son's I. ,I;' ,Sun's .. idi .. eter

, 8efraet50n I

.ParallaX~ , .:

8& SO' &. .+, 15 47..2 .....:.. 'hI

+' I' iO-.i,

\ ............. .-. ..

o 5,i l&7 S . . f8. 2a 5~:N.

Latitude',with face west. . 21 3t 36.3 N.

t" • #0 I ." iJ

• r _ T? compute the sun's deeliaation, JUlIe 6th, 1892.~

~J&!t~e in time ' 7" 1m D. L. 0.53400 ,

6 9" ~"14. I '. },46640 .

Prop. part to 7" lID, , l' 48" P. L

~. 'WBeC. IIUf.-24" and 'f' + 2.5

2.00048'

•.•• f" ~

. Correct prop'r + 1 50.5

. ~~~r,pon,. 2'J SQ' 10.0 " .

( .... ~ .. , ,

~.~. at S. B. 2'J 40 0.5 N. ~

~ ~pute/the retraction, the barometer being 29.8 in~ and:

the tb_9Dleter 85° Fah., the meridian Z. D. being 1° ~,5 nearly.

*,'Biit;. ~023'.5Iog., 0.IQ26 Patallax·()".!l'

Ther. 85° . log. 9.9004 '

Bar. 29.8 .' J. 9.997r

,~':~o 0.9085

~ .....

(·:'.·'r'A4

'.1 .• ,

I"

. . 0.1576

Face of the circle east.

Beadings{ 1st vern!er

t . ": 2d vermer

" ,

1023' 30" 25

Qbs . .zenith dist. sons's I. I. ~'8: semidiameter

~addbn .

Plrallax .

Troe'iner. Z. D.

')'~'ns'l "":"15 '47~~; - ,14 . + . ,'()~~t

1 '1 39'.3 '5.

Digitized by Google

Sf

/ ~ ~,,! r Z • .D. y~~on .

. ~tua,. face east

• 0;' t'ac.e west

Meaq latitude by sum 21 82 28.75 ' . II .

1Vhea the la\itude is determined by an astronomical cit&, :111\' Obsenation.ia DOt...IIUpposed to be complete, tin the observer h-.t.,ev~ the circle, by tbi. means combining two sets of ob~s, with the face or. graduat.ed limb of the iDttru.en' ~1 .. in this example, towards the eut and west. .' . . : '

San Blas, iOth May, 1822, the barometer beiPg.at ~.1a.~~; F&brefiheit'l} thennometer 83°, the chronometer. tQct ....... f .. fJIieut time 41\ 41D 45", Polaris on the meridian below the -pole by"a.oauii ter al:.lh 8'" 41' ~ its true apparent N.P. D; 1° 38' 28".46.·:' ,'.lI.)

. I

I' • .l ~"'I' 1

s
iface~~ ~l'OII~. ~:'~: IReII=~ to alii, ~,u. &041 . AIU .... ,.:.1 ~
All&.
- ' J
h 1ft I m. I I. ~ , n .. r . " . J
i tJ 5 i 86 13".27 70 3345 } 1.9 ,i6. 8&5\
::l 1 . '1.51 .050 1.36 334.0 h • .Ii&1ifb9i1 "
1_ 841 o 0 0.00 3350 66 ~.o I
114' 3 522 56.55 19 56 19.0 56 .l9J)._,
I f: M 730 110.44 56 18.0 , fitS'l8J)
'654 19~ .41 55 20.5
,·1 : 196629.83
. _.- ~ 6 .S7t .63 I
. !
.J '.
i . 1 62.84 I . , .) ~ "'r.", TR ~uiP._J.re the correction of altitude on account of the diBWu:e

of the star from the meridian. ' '. .

.J<' ~o $' 30" cosine 9.9681~

.,' 28 21 30 cosine 8.461111

, Alt 19 56 22 secant 0.~14

~, :-. G".34 log. l/fNfIl"l.

.' 'tI'" It

(Jor.-'l.77. 0 log.,. .,.: •. . •

. The correction fgr »'t~ I.L is in this cue insensible.

'lhdo~tite thenti-action.

7(1' 3'.6 !og. ,

m F. ' ,. log.

19 /18 , log .

89. log.

().1411O'

Z.D. '.I'iIu . . Ba.."

Ther.

to

rI4Ir'.M

Or 2' j'f" •• Observed altitude Refraction

s • €dft'ecdon '

log.

./ I_

True altitude

. , "

.19- 53 53 .M Digitized ~y Google

8P11EB1CkL TlUOONOMETRY.

Mean' .

CaptaUl.lIa1l __ it. .

'19' 5S'''S:f':54N.

l' 38' ~L46N.

21 32 22 .OON. 21' 3i 518 .76

21 32 25.81

··fn· .·.·i6'f

Trut!' altHIade below the pole PolDdiMan*·. .

Latitude &om Polaris from Sun

I ~ #."

J>itFeQQD . . I: .?ft.,.

Which.appMh to 1M! OCCAsioned by neglecting the appliciitioh ot the equation of second difference in reducing the sun's deClination to

the place of observation. . .

,·It .... ~i'y to 'extend our remarks rarth~r' ~th regli!4 '!' iheee obtetvtttions,' more especially if the examples 10 the ~xplana&iuD._' 1ile ,table XXVDI. be consulted. If tile . observations are taken at Ilea ,nth a refteeting instrument, on the principles of nadle_y's q_11II!h'_apt~ a correetion JIUI8t be made for the «Ii" in addition tp tfese already given. ,This may be taken from 'tIifile XI.; {II' tIie Uue_ altitude may be still more readity found from table' XlII;''dr

JUV. sufticient1~ correct for all the usual purposes at sea, :

: ~. IUy 1st, 18!5, in longitude 64° 25'W .• the observed, mt~Ikltikucfe or the sun's L I. was W 34' S&"~.~e zenith' being ... l8fthesun; aild the height of the eye 14 feet; what.wasthe 14-

titude.? 'f :

I""]t!':~ ship, time c)!> om . Dee. 1st 15°,4' 19"lf.

:-I-. Hriime ' ". 4 '18 P. P. + 8 14 '

.:~~~~' I" '!. I

I Gr. time, Mal 1st '4 ' 18 R. D. 15 7 as If.

,Observed Altitude • 48" 34'.5

l£or. ~,~o, 14 r~ aDd.llay + II .0 i

. an.ealt. I • , ' " 48 46.0'

I . ~ , " "".f

---

".

• !! " r 'e ~ ').

'41 14.0N. 15, 7.6,N.·

.: ..

~ . . . .' '. 56 21.6:r1. Id. DimMfI"ry to p1ltlb the calculations nearer than tentbs of a milJ*j .. 8IIy obeer:vation taken at sea is, from -the indilltinctriel8' ot the bori.on aDd tha.~ of the horizbltta'l'refraction, unless a dip eect;ion be used, liable to aD error of at lltlst one minute.

Ezamples for Ezercise." .

l,~ On the 1st of September; 1'824, in longitude 54° W" . the aeridi.titt~de of the sun's lower limb was 79044' 15" 8., the height

or ~e 'f1e being 24 feet ; what was the latitude? ' : .

N~ltr 30'.9 N.

_ ,t::~.O~l'tfle 1st of January, 1826 the meridian altitude oF tbe star ~"Wa8 60" 41' S., the height of the eye being 94 feet; .. bat

wte latitude?

, ~"'29':8. '

':<¥ the 14th September, 1827, in longitude 103" 18' Eo, ·let.1be meridian altitude of the moon's lower limb be 51° 4' N., and the ~ or ltieeye 20 feet; required tbe latitude?

~ ,', ,. ..4111._190 48'.4 S.

Digitized by Google



1laWD1JOC'DIOlI.: : .

4. 0b1the".2Idl September, 1897, in longU_ ~ .'w.., if. the ob..-vt!lCi Jbl!ridian altitude of the moon's. upper lilJlb. he- 839 6', N., and tba.Jaeig~ the eye 16 feet; required the latitude?

, . AIII.-21° 25/.7 S. ' '. ·,I ..• ·.f

.1\$. the meri&n altitude may, by the inter- . position of clouds, or other causes, be lost at sea when a knowledge of the latitude is neces.sary for the safety of the ship, recourse must be badrto otheemethcds, particularly to that of Hj--nff.1~~--l double o.ltitudes, andrbe .time between them, \J.

.~ beipg. the. m.o~t p~actU:able. tI :nm' , ~e,""

1;$.04 l'eqllll'e~ aolutJOn.s,lll Wee. spberl.~l NJ;a.P~ _ _. _ t

gles. In tIle triangle 'ZPS there aJ'cgiven ~. ,~ "

PS the .sun's. polar ill~tan~ lit :the time ·of' t40-.fu~ ob§ellvatio~ PS' that at the. .second, and tbe. angle SipS D}:casu.rcd,. 4J' thtct ~.'1.ee~ap.~ed. ~ti ~e; to lihd - the. si~. c. $IS-anc,h ~~~ aJll:!~e lP;S.(8. t 'Agllint J:: the tyat:!gJe .ZS'Stl!e!~ ~e glyen t;,he ~en!~h distallc . ~. aJ;tb!!t time of the fiJSt observatIOn; Z,S7 that, ~t, the second. and t,);re, Ide &la iili~~y ~tlml to determine die angte.ZS'S: But; 1?~S. ll~g .;il..t r,e.a~'y' C4?Ul;p,utea, Z~p ma~ be obta.!»ed,Wbenc,,: there-ope lll,tb~ trumgle ZS P, the sides .ZSJ , .. :mll p,s:, and ~ £Qnta.Jt;l~d an.gle Z ~; ~ .lind theaiae ZP the· colatitude. This is ~e il'egQl.IU- method 'by snbemca'l trigonD.lDel:ll1y j. but if the polaedistance, PS be supposed tor remain the sa.mfil, that. at the middle. time, beb"'leen the observation!! .. or, as 'Professor Lax seems to think preferable, the same as at. ~ "~~~'(l'eate~ aWtude;.a.nd; by ~biniJlg.~8;ad~~·tbe sevel:¥ t:rt!iLQg1es 10 one, ~ ~peration I:!ecom.ea ~"lUlple.·. In ordft. .~, 1'fiD~er 'tIus methQd @ll more elllY to practical seamen, ~~; prop~ ~ ap~D¥Jte method by i~-tn&. ;tbe. ladtade- b~,~unt, which, when properly reatrictell ailorA:diDg:-tiO'. rul~ «fr)~~kelyne or the tables of ~ -m. ~l1P",tW.Qe.. aired ~sul~ ~ufficieDtly correct. for IUi.'utical purpose8, and the eempuQl!i~ ","y be very readily performed by ~ tables of L,... " '

~ th~ common tables are used, Mr I vorr's solution is the best, P!I'f\ic~ly: in the form that Mr Riddle has given U, which 1PQ.slNUI

~1~1 SjlIl'S declination for ilie time of the greater' 'altitu~e;~d the tfq~·al~des, red~cing the less if neceBBRy for the ship's ron to w,ru.t ~JrQlIlcl:~vt! ~ ~ jt ~ takea.at ~.""'P~":Ith the gt'eateJ:. ~,18,~qq»1Ml1ed by ob_vlll« tbetlW\" . bearing· by com~, ~ ~e time of takiD« the leis altitude, and, findillg the angle colltained betw~ that an.d tlle ship's oouree .b,..~ <corrected for leeway if shemakes &Il¥, iJl. ~ inW,rval between the 'observatioQs. With tbis angle 4Il a·~seept.er a ~v~te<)~j ttPd th~ d,ilfer~f1ce. ~ la~tud,? answ~rh~g to t?~ d~stanc~' r~n. ~~g,th~ I!lApsed time, W!U ~ the reductio!! of altitude_. , ., . _ . : 4

If the less altitude be observed In the forenoon,. t~e reduction of altitude mu~t be 1I4ded to i~, ifth~ ~gle ¥.~een tIi~··~hip·s. c'P#,se and the sun s bearmg be less thari e1gh. fOlDts; b.ut 1f that ang1e be greater than eight points, the reduction rs to be eubtracted frQ.JP the. less altitude. If the less altitude be observed in the a~~loon; the

, ,

• On the a~thority ~. ~. ,. ;""'~r~l ~~~:~t

double altitudes are not of sv::i importance &8 ~j .auppollOd. . ". '

t A circle is suppeeed to pass through P8' Pi similar 10 ~'P'.

Digitized by Google

SPREBIeItL ~JIfMIETRY.

~ci&it; it to;tie.l~ from it,· if dae ~ between the efp'1 cbune and tbe sun's beuin« is 'less than eipr points; but if gr~ter, the reduction is to be added t61:1le leu altitude. WjJ;b the 'eoftected altitudes, ~e elapsed clme, and the declination, the 1 .. !ftV,tfe lit the tin:ul of the observation of the gre&teat altitude will be found; which may be redueed to noon by meaua oC the deed

reckoning. •

1. Take half the interval behveea the obaervatiool, and call it

the "all elapletl time. '

. -9.' T«Jfhe line oftlaelulf ela}*d tilDe add the nile of the 1hID·. poW: distance, the SUlll, rejecting always ten in the index, will be

.n:~I" ., .'

3. To the 11#:11111 of arc firat add the COline of the polar distance • ." .am ...m be the ~ine of lire MIOtUl. which will be .r the same

a6ctien ..... eMracter as the polar distance. -

II ••• To the ccIftcmIt of arc firllt, add the eMile of half the JRUIl of the tnMf altitudes. and the .. of Ulf their dHFerence; the sum will

be the Me of lire tItinL .

&..AM together the I~n' of arc irat, the nile of b~ the 3llUl of the 4iu altitudee, the conu of half tlNir ditFerenee,· and the _at of UO __ , the IUJa will be the com.e of orefourtll.

flx!flle.rl#fn_ee of ore HctJiad and on: fOtlrlla is tWe .mlt, when the senith.d the elevated pole are on ehe same Bide eetbe gnatCUde. pe."1hrou~h the places of the SUD at the times 01 obsenatien,

edt..". th8ll' ,.. is 4"e .Rlth. . . - .

7. To the CMiIie of ore awl add ·the Coatle·olllf'C jf/M,· .. a the 111m win be the mae of the laIiteule.

, &Do I.-On the 6th of 1UDe, 1828, in latitu.IieUO N.,and~_ 480W., by acc:ount, at 10"53-. A. M. r:; ..... tch, tile altitu __ tlte ... ·.lower limb wu oSlO 2&, aod at 1 17m 8', the alatude bf·the Slime limb was liSio 64', and the bearing per oompa81 S. W. by W. The ship's eourse during the eIapeed time wa s.~ the wind E.S.B •• and. homy nte of 8IIiliDg 8 knots, and the ship making 1. pta • .. way. ~ the true lati,ade at the time of ob8erftldon of ... greateIIII altitude, tit. heish~ of the eye being 16 feet?

Ship's apparent course S. or ()I*

l'M-way. Ii

• ,I • .--.-

.1·'8litp·stnie course . s. byW.i w. = Ii pta S. w.

81111's bearing at SId obs. So W. by W. ==:6pbi S. W.

: Contmed angle . .". ' ··It· .

Interval between the obseruti8lll'::::: ~ • .(80 := 5P>.4

Dilrtanee l'1I1'l::i:! ::dl" . .fx8::::: )9~ mile. . ' ,

" . l" . 'w' ,

'Now to coUrse '8! pointli and distance 19'.'!~ the dIffertmce of lati.

,iu~ i. ~".84, ima since tbtf least aleitude was observed in tile . '~«:i6ft. ~ ~'1te.4ngl.e .between the ship's course, imd sun's bear-

Wg U l~ tb~n ,~jght pbints~ this red~etion is subtrd~e. ..".,

.. .

",

" ". Sbeuld ~ be any doub~ lfhetbtr th& ~~th qd elevated ~e are on the same --Meeflie pti~ p8fting tllmuab the plaCeS of the sun. the JatftudelUl11 be com • . pi" .. bI&h ~tt.a, whidJ, biln8 ClllDpared witll that by -t, the true l.tI • .. -wa. .... {.·.....-I._~1~.1Iith;liWe~tiooaJ...., _if. only - an: roiarih anll ili_ine that will require alteratton.

Digitized by Google

.as

Fir~ob8erv_ alto Cer. table nu.

1. True alt.

1. True alt 2.

Sum

P~op. pUt

D~. at noon or &;h

Reduced dec.

Pol4U" dist.,

63°00- + 11.2

, Secood ~elt • .Q. ~ +11.2

o 40.8 Time Long. W.

- U.s 52 50.4

. '_

half 0 20.4= 0 iO ~ 1()b 53'" iO' A. 1\1. 3' 12"

on 6th at

D.L. P.L.

o 31

220 41' 17" N ..

22 41 48 N. 67 18 12

~ aiD. Ih 11m", B. E. T.

9J166066 lin. 67 12 12 pol. dist. cot. 9.~ ,

~ sin. IS lit 111 arc lit see. 0.013382 cosec.

, 66 16 62 arc 2d cos. 9.004784

53 31.2 Reduction 2 True alt. 53°31'.2

52 50.4

106 21.6 half 63" 10'.8=i3'? 10'48"

Difference .

. {10" 53'" 20'

Times 13 17 8

Elapsed t. 2 23 48 H.E.T. 1 11 54

App. time 2h 5m 20'

Daily variation 5' 55"

14 6 2O,A."M. ~, 6 ~,P.Il.

','

1.06030, 1.483j()' '.

-

'J .! t . t,

_,f

sec. sn:]"

aiD. 61" 19' .... _.

CIOIoo 0., M ..

Bee. Jil 0 42 118 sin.

.,018382 9.110337' 9.99SII9B

O.fMI08IU

33 22 8 arc 4. COlI. 9.921763

32 112 44 arc II. cos.

latitude 67 6 61 N. are 6. aine

3d

In this example the computation i. carried to 8eCOftd" but sm:h a

degree of accuracy i8 unnecessary' at sea. '

2. On tbe 6th of March, 1827, in latitude ..,.. N. hy ~t, and -longitude Uy'jO E., tbe altitude of the 8un'8 lower limb was ob~ed to be 19° 42' at 40h 4m 20' in the forenoon, hi8 centre bearing'S. S. E. by COJDpU" and at 1 h 32m 36' afternoon ,it W88 210 6'. The ship'.

, course during the elapsed time W88 N. W. by N., sailmg at the "ale of9 knota per hour, aDd the height of the 'eye 16 feet. Required the &hip's latitude at the time of taking the greater altitude i" ' ..4118.-60" 37' N.

3. AQgIlst 31,1827, in latitude 12" .fO'S. by account and 1~tude 165° E. at lIh 13'" 30' A. M., the altitude of the sun's 10.nab was 000 9' 30", and at Ih 16" 12' P. M. it was 6P 0' -JGIF/ ~ .. _

... - ..

Digitized by Google

You might also like